Sport Orthopedics Mcqs online Bank

(SAE12SN.35) What is the most commonly involved level for brachial plexus stretch injuries or "stingers" in younger athletes involved in collision sports? Review Topic

 

1 C3-4

2 C4-5

3 C5-6

4 C6-7

5 C7-T1

 

PREFERRED RESPONSE 3

 

The most commonly involved level for brachial plexus traction injuries in young athletes is C5-6, ostensibly due to the greater mobility in the midcervical spine. "Stingers" in older athletes may be due to foraminal stenosis in combination with extension and nerve root compression.

 

 

(SAE08OS.83) A 32-year-old man has groin pain that is aggravated by sitting. A frog leg lateral and an axial MR arthrogram with gadolinium are shown in Figures 26a and 26b. Nonsurgical management has failed to provide relief and it has been decided to proceed with surgical intervention. The most appropriate surgical approach should include Review Topic

 

 

 

  1. total hip arthroplasty.

  2. arthroscopic labral debridement alone.

  3. arthroscopic labral repair alone.

  4. posterior hip dislocation with labral repair and femoral osteoplasty.

  5. anterior transtrochanteric hip dislocation with labral repair and femoral osteoplasty.

PREFERRED RESPONSE 5

 

An anterior labral tear is present in the setting of cam-type acetabulofemoral impingement. Although labral tears of the hip can occur spontaneously or secondary to a traumatic episode, many labral tears are related to an underlying predisposing anatomy such as dysplasia or impingement. Labral tears are often treated successfully with nonsurgical management. When surgical treatment is indicated, it is generally recommended to address the predisposing anatomy in addition to management of the labrum. Cam-type impingement is addressed by removal of the prominent bone at the junction of the femoral head and neck to prevent abnormal contact between the neck and the rim of the acetabulum through a normal range of motion. Most of the data supporting this procedure show that it has been performed with a trochanteric osteotomy, allowing for an anterior dislocation of the femoral head without interrupting the blood supply to the femoral head (branches of the medial circumflex artery). Some authors have reported success with an all-arthroscopic approach. In general, the outcome of labral repair has been more favorable than labral resection.

 

 

(OBQ14.51) A healthy, active collegiate soccer player returns to your office approximately 10 months after returning to full play and 18 months after undergoing ACL reconstruction with bone-patellar tendon-bone (BTB) autograft. The patient reports landing awkwardly after a jumping for a ball and felt his knee give way. He presents with pain, worse with weight bearing. On physical exam, there is a mild effusion and a grade 2B Lachman. Radiographs are shown in Figure A. What is the likely underlying cause of his current diagnosis? Review Topic

 

 

 

  1. Inappropriately early return to play

  2. Improper rehabilitation and conditioning

  3. Unstable meniscal tear

  4. Malpositioned tunnel

  5. Inadequate graft sizing

 

PREFERRED RESPONSE 4

 

The most common cause for early failure following ACL reconstruction is a malpositioned tunnel.

 

Ideal tunnel placement on the femoral side should be at the approximately 2 o'clock (for a left knee) or 10 o'clock (for a right knee) position on the lateral wall, which facilitates a more horizontal, anatomic graft. On the tibial side, the tunnel trajectory in the coronal plane should be about 60-75 degrees from the horizontal and the tunnel entrance should be approximately 10-11mm from the anterior border of the PCL.

 

Noyes et al. emphasize the importance of anatomic reconstruction. They recommended against using a transtibial tunnel to make the femoral tunnel because it will result in a vertical orientation. The authors summarized and recommended the use of individual drilling of each tunnel, and using a anteromedial portal to obtain the ideal femoral tunnel.

 

Driscoll et al. compared the rotational properties of a BTB graft placed centrally in the tibial footprint in both groups, but on the femoral side, placed in the anteromedial aspect versus central portion of the ACL femoral origin. They noted a significantly stronger resistance to rotational failure when placed centrally. Thus, noting the importance of placing the graft anatomically, within the central areas of both the tibial footprint and femoral origin.

 

Figure A exhibits malpositioned tunnels, both of which are too vertical. Illustration A exhibits well-placed tunnels, with the horizontality exhibited on the femoral side and approximately 75 degrees from the horizontal on the tibial side.

 

Incorrect answers:

Answer 1: Return to play remains a controversial topic where there is no high-level evidence to decide the ideal time frame. This patient, however, is seemingly out long enough to not sustain this tear. Answer 2: Improper rehabilitation and conditioning may place the patient at risk for re-injury. However, the most common cause for ACL graft failure is still malpositioned tunnels, as noted in Figure A. Answer 3: An unstable meniscal tear, even if it occurred, is not likely the correct diagnosis simply because of the instability noted on exam. Answer 5: It is difficult to assess the graft, however, the follow-up is too long for this to be the cause. Smaller graft sizes would have been evident earlier and likely would also have shown screw loosening/position change.

(SAE08OS.199) When comparing gait parameters between a patient with an anterior cruciate ligament (ACL) deficiency to a patient with a normal knee, the patient with an ACL deficiency has which of the following? Review Topic

 

  1. An absence of the normal internal rotation of the femur during the terminal swing phase

  2. An absence of the normal external rotation of the femur during the terminal swing phase

  3. Greater posterior translation of the femur during late swing

  4. Greater knee flexion during midstance

  5. Greater adduction of the hip

 

PREFERRED RESPONSE 1

 

During normal gait kinematics, the knee is in near full extension at heel strike with relative internal rotation of the femur relative to the tibia. During midstance (swing phase of the contralateral extremity), there is flexion of the knee and external rotation of the femur relative to the tibia. In addition, in the normal knee there is relative anterior translation of the tibia during late swing with contraction of the quadriceps. Between heel strike and midstance there is posterior translation of the tibia relative to the femur. In an ACL-deficient knee, there is abnormal kinematics characterized by absence of the normal femoral internal rotation during the terminal swing phase. Furthermore, there is decreased anterior translation of the tibia in late swing, presumably an adaptive response with decreased quadriceps contraction and/or increased hamstring contraction.

 

 

(OBQ14.201) The findings from the MRI depicted in Figure A have been shown to predict which of the following outcomes after anterior cruciate ligament (ACL) reconstruction? Review Topic

 

 

 

  1. Increased risk of re-rupture

  2. Higher quality of life scores

  3. Increased antero-posterior laxity

  4. Delayed recovery of range of motion

  5. Increased risk of infection

 

PREFERRED RESPONSE 4

 

A prolonged range of motion deficit following ACL reconstruction has been demonstrated in patients who have a classic bone bruise pattern on MRI (Figure A) after ACL rupture.

 

Females, acute surgery, limited preoperative range of motion and the classic bone bruising pattern seen on MRI are all predictors of post-operative loss of motion. Strategies to prevent development of motion deficits and arthrofibrosis include preoperative rehabilitation to improve motion, waiting until the inflammatory phase has subsided to operate, proper tunnel position, and aggressive post-op cryotherapy.

 

Quelard et al. conducted a retrospective cohort study of 217 patients who underwent ACL reconstruction to determine the preoperative factors associated with prolonged range of motion deficit postoperatively. Limited preoperative range of motion, typical bone contusions of the lateral compartment, operative delay less than 45 days, and female sex were found to be significantly correlated with delayed recovery.

 

Johnson et al. prospectively evaluated 40 patients who had knee inflammation after isolated anterior cruciate ligament rupture. Patients with a bone bruise had increased size and duration of effusion, increased number of days required to nonantalgic gait without external aids, increased days to achieve normal range of motion, and increased pain scores at measured time intervals.

 

Figure A demonstrates an MRI with the classic bone bruising pattern associated with an ACL rupture. As seen on the MRI, the classic pattern involves the middle third of the lateral femoral condyle and the posterior third of the lateral tibial plateau.

 

Incorrect Answers:

Answer 1, 2, 3 and 5: The classic bone bruise pattern seen on MRI after ACL rupture has not been associated with an increased risk of re-rupture, higher quality of life scores, increased AP laxity or infection

 

 

(OBQ06.235) A 25-year-old marathon runner presents with pain, coolness, and tingling in her lower leg and foot which are exacerbated with walking, but relieved once she starts running for a few minutes. Compartment pressures are normal at rest and with exercise. Her physical exam is significant for pain with passive dorsiflexion and plantar flexion of the ankle. These symptoms are most consistent with which of the following conditions? Review Topic

 

  1. Lumbar radiculopathy

  2. Piriformis syndrome

  3. Exertional compartment syndrome

  4. Popliteal artery entrapment syndrome

  5. Tibial stress fracture

 

PREFERRED RESPONSE 4

 

The clinical presentation is most consistent with popliteal artery entrapment syndrome.

 

Patients with popliteal artery entrapment syndrome typically present with intermittent claudication and decreased pulses. The pathoanatomy involves compression of the popliteal artery by the medial head of the gastrocnmius. Therefore, dorsiflexing and plantarflexing the ankle are provocative maneuvers that would worsen the claudication as a result of the compressive effect of the medial gastrocnemius on the popliteal artery. An arteriogram would demonstrate the popliteal artery compression by the medial head of the gastrocnemius.

 

Gokkus et al. present a case and systematic review of popliteal artery entrapment syndrome. They state the differential diagnosis for patients with lower leg pain from exercise includes chronic exertional compartment syndrome, medial tibia stress syndrome, fibular and tibial stress fractures, fascial defects, nerve entrapment syndrome, vascular claudication (artherosclerotic or popliteal artery entrapment syndrome) and lumbar disc herniation.

 

Illustration A shows MR angiogram images demonstrating a segmental occlusion of the right popliteal artery.

 

Incorrect Answers:

Answer 1&2: Since the patient is not exhibiting signs of a lower extremity nerve entrapment syndrome (neuropathy would not explain the cool crampy leg), lumbar radiculopathy and piriformis syndrome should not be the 1st choice (in this case the parasthesias are a result of the claudication). Answer 3: Elevated post exercise compartment pressure measurements are essential to diagnose exertional compartment syndrome where vigorous athletic exercise leads to swelling of myofascial compartments in the lower leg leading to pain which is relieved with rest. Answer 5: A stress fracture would not be relieved with running and would not have exhibited signs of claudication like the cool leg, cramping and parasthesias.

 

(SAE10SM.74) Which of the following types of intra-articular pathology is associated with lateral meniscal cysts? Review Topic

 

  1. Discoid meniscus

  2. Posterolateral corner injury

  3. Vertical meniscal tears

  4. Middle third lateral meniscal tears

  5. Popliteus tendon tears

PREFERRED RESPONSE 4

 

Lateral meniscal cysts often arise from myxoid degeneration that progresses from the meniscal center and then outside the meniscus. Horizontal cleavage tears are commonly associated with the condition. Cysts of the lateral meniscus are most commonly the consequence of a tear located in the medial third. If the tear communicates with the joint, arthroscopic partial meniscectomy and cyst decompression are indicated. If the tear does not open into the joint, arthroscopy should be followed by an open cystectomy.

 

 

(SAE08OS.198) In assessing the knee after acute injury, increased tibial external rotation at 30 degrees of knee flexion indicates injury to what structure? Review Topic

 

  1. Medial collateral ligament

  2. Anterior cruciate ligament

  3. Posterior cruciate ligament

  4. Posterolateral corner

  5. Posteromedial capsule

 

PREFERRED RESPONSE 4

 

Increased external rotation at 30 degrees indicates injury to the posterolateral corner of the knee. Increased external rotation at both 30 degrees and 90 degrees indicates injury to both the posterolateral corner and the posterior cruciate ligament.

 

 

(OBQ15.120) A 39-year-old male with chronic renal disease and type 2 diabetes mellitus fell 1 week ago after slipping on ice. He is unable to bear weight on the right lower extremity or perform active knee extension. He reports no prior history of knee pain or instability. Lachman, posterior drawer, posterolateral recurvatum testing are deferred secondary to patient's pain. He has a palpable dorsalis pedis pulse but does have neuropathy as determined by Semmes-Weinstein filament testing. His radiograph is shown in Figure A and MR images in Figures B and C. What is the most appropriate initial plan for management? Review Topic

 

 

  1. Primary repair of injured structure

  2. Primary repair of injured structure combined with anterior and medial tibial tubercle transfer

  3. Immobilization of knee in full extension with a progressive weight-bearing exercise program

  4. Semitendinosus or gracilis tendon autograft reconstruction of the injured structure

  5. CT angiography followed by primary repair of injured structure if the limb has vascular stability

 

 

PREFERRED RESPONSE 1

 

The clinical presentation, exam, and images are consistent with an acute patellar tendon rupture.

 

Primary surgical repair within 2 weeks of injury is recommended to prevent extensor mechanism contracture. Patellar tendon ruptures typically occur in patients younger than 40 years old. Most ruptures occur at the junction of the tendon and distal pole of the patella.

 

Matava et al. presents a level 5 review on patellar tendon ruptures and states that active knee extension is permitted at 3 weeks postoperatively. Non-weightbearing movement exercises like heel slides are encouraged. This can incorporate active knee flexion with passive extension. Alternatively, active knee flexion in the prone position with passive knee extension can be performed. Open chain strengthening exercises such as leg extensions are started later, as are weight bearing resistance exercises like squats, lunges and leg presses.

 

Volk et al. discuss potential complications and pitfalls of patients with the management of extensor mechanism injuries. They warn that complications can consist of misdiagnosis, delayed surgery, failed repair due to poor surgical planning of injury site, or wound infection.

 

Figure A demonstrates patella alta which in this case is indicative of complete patellar tendon rupture. Patella alta can be quantified by using the Insall-Salvati ratio (patellar tendon length / patellar bone length): PTL/PBL normal =1, >1.2 is patella alta, <0.8 is patella baja) with the knee flexed to 30 degrees. Figure B and C are sagittal T1 and T2 images showing complete patellar tendon rupture.

 

Incorrect answers:

Answer 2: The patient has a ruptured patellar tendon and does not have a history of patella lateral instability that would warrant a tibial tubercle anteriorizing and medializing osteotomy (Fulkerson osteotomy). Answer 3: Immobilization in a hinged knee brace locked in full extension is only indicated for partial patellar tendon tears where patients have intact extensor mechanisms.

Answer 4: Semitendinosus or gracilis tendon autograft reconstruction of the Patella tendon is only reserved for cases where primary repair is not sufficient to restore the extensor mechanism and augmentation with other tissues isn't required. This surgical option may be needed intra-operatively however should not be the preferred initial management option.

Answer 5: There is no indication from the history and examination that the patient has sustained a knee dislocation that warrants further vascular evaluation. The patient has palpable pulses so CT angiography is not indicated in this case.

 

 

(OBQ15.125) A 35-year-old skiier presents with pain in the left buttock and proximal posterior thigh after a fall. His clinical appearance is shown in Figure A. He is enrolled in 8 weeks of physical therapy after 2 weeks of rest, icing and NSAIDS. He returns for follow-up 6 months after his injury and has persistent ischial tuberosity pain with running. Examination confirms focal ischial tuberosity tenderness. MRI images are seen in Figures B and C. Which surgical option is most appropriate? Review Topic

 

 

 

  1. Sciatic nerve decompression

  2. Arthroscopic labral repair

  3. Repair to the intertrochanteric crest

  4. Repair to the femoral shaft

  5. Repair to the ischial tuberosity

 

PREFERRED RESPONSE 5

 

This patient has a partial hamstring avulsion injury. If symptoms persist after a period of therapy and rest, operative repair to the ischial tuberosity is indicated.

 

Untreated partial hamstring ruptures may present with residual pain, weakness and hamstring dysfunction. The mechanism is eccentric lengthening (sprinting or cutting) A proposed treatment algorithm is: (1) Nonoperative management for single tendon avulsion with <2cm retraction. The ruptured tendon scars to intact tendons. (2) Repair for acute 3-tendon rupture (semitendinosus, semimembranosus, biceps femoris) with retraction >= 2cm. (3) Surgery for young (<50y) patients with 2 tendon avulsion and retraction >= 2cm.

 

Bowman et al. examined the outcomes of operative management of partial hamstring tears in 17 patients. They found no postoperative difficulties with ADLs, and no recurrent surgery was required. All patients returned to their preoperative level of activity. They concluded that surgery can lead to good function with low complications and is reserved for patients who have failed nonoperative management.

 

Hofmann et al. retrospectively reviewed 19 patients with nonoperatively managed complete hamstring avulsions. They found diminished SF-12 scores, diminished hamstring strength at 45° and 90° of flexion (62% and 66%, respectively) compared with the normal side. They concluded that nonsurgical management leads to both subjective functional and objective strength deficits.

 

Figure A shows pronounced bruising from hematoma tracking following the injury. Figures B and C are coronal and axial images showing partial avulsion of the right hamstring insertion. The images correspond with Illustration B, with arrows pointing to the "sickle sign" . Illustration A shows the origins of the hamstring tendons. Illustration C shows the origins of the hamstring group (bf, biceps femoris; st, semitendinosus; sm, semimembranosus; qf, quadratus femoris; am, adductor magnus)

 

Incorrect Answers:

Answer 1: Sciatic nerve decompression is indicated when patients have hamstring syndrome (posterior buttock pain and discomfort over the ischial tuberosity, worse when sitting and during stretching, sprinting, hurdling, kicking) because of hamstring tendons scarring to the sciatic nerve. Answer 2: There is no labral injury. Answers 3-4: Nonanatomic repair of the avulsed hamstrings is not recommended.

(SAE13SM.10) A 17-year-old basketball player has a soft-tissue abscess over the anterior aspect of his left knee. The team physician prescribes amoxicillin and the infection resolves. The next week the patient develops fevers and significantly increased pain at the site of the previous infection. What is the most likely diagnosis? Review Topic

 

  1. Community-acquired methicillin-resistant Staphylococcus aureus (CA-MRSA)

  2. Tinea corporis

  3. Herpes simplex virus

  4. Group A Streptococcus

 

PREFERRED RESPONSE 1

 

Skin and soft-tissue abscesses should be drained and cultured by the treating physician whenever possible. Antibiotic therapy should be guided by antibiotic sensitivities derived from the cultures to identify cases of CA-MRSA and prevent severe recurrent infections. These infections have been associated with significant morbidity, with up to 70% of players requiring hospitalization. A high index of suspicion in at-risk populations is necessary, and empiric treatment with an antibiotic effective against MRSA should be considered until sensitivity results are available. Tinea corporis is a general term for a cutaneous fungal infection. The lesion appears as a well-demarcated erythematous plaque with a raised border and central hypopigmentation, giving it a ring-like appearance. Primary infection with herpes simplex virus can produce constitutional symptoms with burning, tingling, or stinging at the site. Grouped vesicles with clear fluid 1 mm to 2 mm in size form on an erythematous base and then rupture, leaving moist ulcers or crusted plaques. Amoxicillin is appropriate empiric antibiotic therapy for group A Streptococcus, so a recurrent infection is less likely with this organism.

 

 

(SAE07SM.43) Which of the following best describes heat stroke? Review Topic

 

  1. Transient loss of consciousness with peripheral vasodilation and decreased cardiac output with normal body temperature

  2. A condition involving painful contractions of large muscle groups because of decreased hydration and a decrease of serum sodium and chloride

  3. Hypernatremia in poorly conditioned athletes, manifested by thirst and oliguria with a core temperature of less than 102.2 degrees F (39 degrees C)

  4. Hyperthermia, central nervous system dysfunction, and loss of thermoregulatory function

  5. A transient condition that responds to glucose administration

PREFERRED RESPONSE 4

 

Heat stroke consists of hyperthermia (greater than 105.8 degrees F [41 degrees C]), central nervous system dysfunction, and cessation of sweating with hot, dry skin. It is a medical emergency that results from failure of the thermoregulatory mechanisms of the body. It has a high death rate and requires rapid reduction in body core temperature. Heat syncope is characterized by a transient loss of consciousness with peripheral vasodilation and decreased cardiac output with normal body temperature. Heat cramps involve painful contractions of large muscle groups because of decreased hydration and a decrease of serum sodium and chloride. Heat exhaustion is distinguished by a core temperature of less than 102.2 degrees F (39 degrees C) and an absence of central nervous system dysfunction. Hypernatremic heat exhaustion results from inadequate water replacement.

 

 

(SAE13SM.12) What factor highly correlates with poor outcomes after surgery for femoroacetabular impingement? Review Topic

 

  1. Age younger than 20

  2. Degenerative arthritis

  3. Prominence of the femoral head in cam impingement

  4. The patient is a professional athlete

 

PREFERRED RESPONSE 2

 

A systematic review of case studies looking at the results of surgical treatment for femoroacetabular impingement showed good results for most patients, with the exception of those with preoperative radiographs showing osteoarthritis or Outerbridge grade III or grade IV cartilage damage noted intraoperatively. Both Byrd and Jones and Philippon and associates have shown good surgical results for this condition among professional athletes. Likewise, Fabricant and associates demonstrated good surgical results among adolescent patients with an average age of

17.6 years.

 

(SAE10SM.92) Which of the following physical examination findings is most likely present in the condition producing the MRI findings shown in Figure 92? Review Topic

 

 

 

  1. Valgus laxity at 30 degrees of knee flexion

  2. Varus laxity at 30 degrees of knee flexion

  3. Posterior drawer

  4. Pivot shift

  5. Patellar apprehension

 

PREFERRED RESPONSE 4

 

The T2-weighted sagittal MRI scan shows the classic "bone bruise" pattern seen with an anterior cruciate ligament (ACL) tear. These lesions are thought to represent subcortical trabecular hemorrhages and are manifested as an increase in signal intensity on T2-weighted images and diminished signal intensity on T1-weighted images. They are classically located in the mid-portion of the lateral femoral condyle and posterior aspect of the lateral tibial plateau. This is due to the fact that an ACL tear typically is the result of a valgus-external rotation of the femur on the fixed tibia. This places most of the weight-bearing stress on the lateral femoral condyle, which rotates laterally and impacts the posterior lip of the lateral tibial plateau. This may result in an impaction fracture if the force is great enough, but more frequently causes merely a microfracture of the involved subcortical trabeculae.

 

 

(OBQ15.181) You are evaluating a patient with recurrent patellar instability who has failed conservative management. MRI demonstrates articular cartilage loss in the patella. In addition to a cartilage restoration procedure, you recommend the procedure depicted in Figures A and B. Which of the following imaging findings best supports your decision? Review Topic

 

 

  1. Open tibial apophysis

  2. Medial patellar cartilage defect

  3. Femoral trochlear sulcus angle = 165°

  4. Tibial tubercle-trochlear groove distance = 25mm

  5. Patella tilt = 25°

 

 

PREFERRED RESPONSE 4

 

A tibial tubercle-trochlear groove (TT-TG) distance > 20mm is highly associated with patellar instability secondary to rotational malalignment and is an indication for anteromedialization of the tibial tubercle (AMZ), pictured in Figures A and B, to enhance patellofemoral stability and decrease patellofemoral contact pressures in the setting of cartilage restoration procedures.

 

Recurrent patellar instability is often associated with chondral lesions of the patellofemoral joint. In addition to managing the cartilage injury, the underlying etiology of the instability must also be addressed to ensure a successful outcome. Patellar instability may arise from coronal or rotational malalignment, patella alta, trochlear dysplasia or damage to soft tissue restraints including the medial patellofemoral ligament (MPFL). Rotational malalignment, defined by a TT-TG distance > 20mm, can be addressed by AMZ. Medialization of the tubercle restores a normal TT-TG distance while anteriorization of the tubercle unloads patellofemoral contact forces.

 

Beck et al performed a cadaveric study measuring patellofemoral contact pressures on the trochlear surface following AMZ. They found that anteromedialization decreased the mean contact pressures and shifted contact pressures to the medial trochlea. The authors concluded that while this procedure successfully unloads the lateral trochlea, it likely has minimal benefit for central chondral defects and may be detrimental for medial chondral defects where contact pressures are increased.

 

Strauss et al authored a review article detailing the surgical treatment options for patellofemoral cartilage lesions, including concomitant realignment/unloading procedures. AMZ is designed to correct rotational malalignment while unloading the patellofemoral compartment in the setting of cartilage restoration surgery. While AMZ has demonstrated good outcomes for lesions located in the lateral facet or distal

 

pole of the patella, poor outcomes have been seen with medial facet or central trochlear lesions.

 

Figures A and B are the AP and lateral radiographs, respectively, of a knee s/p AMZ, also know as a Fulkerson procedure. Illustration A is an algorithm detailing the approach to recurrent patellar instability in both skeletally immature and mature patients. Illustration B demonstrates how to measure the TT-TG distance, the distance between two lines drawn perpendicular to the posterior condylar axis to the tibial tubercle and deepest portion of the trochlear groove (normal 8-10mm). Illustration C depicts the measurement of the femoral trochlear angle or sulcus angle (normal 137°

+/- 8°). Illustration D shows how to measure lateral patellar tilt (normal < 5°). Illustration E is a postoperative radiograph after AMZ. A long (> 5cm) and thick (> 0.75cm) osteotomy cut is required to allow adequate fixation with two screws.

 

Incorrect Responses:

Answer 1: Open tibial apophysis is an absolute contraindication to AMZ, as there is a high risk of growth arrest and subsequent recurvatum deformity. Answer 2: Medial patellar cartilage defect is a relative contraindication to AMZ, as the procedure increases the contact forces in the medial patellofemoral joint and has been associated with worse outcomes as compared to lateral and distal lesions. Answer 3: Trochlear dysplasia, defined as a sulcus angle > 145°, is not an indication for AMZ. While controversial, some authors advocate for trochleoplasty in cases of patellar instability with significant trochlear dysplasia. Answer 5: Patellar tilt > 20° is not an indication for AMZ. It is often indicative of a tight lateral retinaculum that may benefit from lateral release in the setting of other surgical procedures.

 

(OBQ14.125) A 25-year-old patent underwent anterior cruciate reconstruction (ACL) surgery 6 months ago. He returns to clinic with persistent instability. Physical examination reveals full range of motion of the knee. Additional tests show a 1A Lachman, 2+ pivot shift, negative external rotation dial, negative reverse pivot shift and negative McMurray. His radiographs are shown in Figure A. What is the likely cause of his persistent symptoms? Review Topic

 

 

 

  1. Missed posterolateral corner injury

  2. Femoral tunnel placement did not restore the anteromedial bundle

  3. Tibial tunnel was positioned too anterior

  4. Femoral tunnel placement did not restore the posterolateral bundle

  5. Tibial tunnel was positioned too posterior

 

 

PREFERRED RESPONSE 4

 

This patient has a vertical femoral tunnel position with rotatory instability. The most likely cause of his symptoms is a femoral tunnel placement that did not restore the posterolateral bundle.

 

An anterior cruciate reconstruction (ACL) surgery with vertically placed grafts will result in persistent knee instability. Complaints will be mainly related to activities that require twisting or cutting movements. Physical examination will likely reveal a positive pivot shift exam due to the failure to reconstruct the posterolateral bundle of the ACL. Current standards for anatomic ACL reconstruction stress the importance of more horizontal graft placement (for example, 10:30 in a right knee vs 1:30 in the left knee). This allows for more rotational stability, while maintaining anterior stability. Improper femoral graft placement is one of the most common reasons for ACL revision surgery.

 

Noyes et al. looked at patients undergoing revision ACL surgery with the use of patellar bone-tendon-bone (BTB) autograft. They showed the rate of graft failure to be three times higher than their reported failure rate after primary ACL reconstructions.

 

Driscoll et al. performed a biomechanical study comparing 2 femoral tunnel locations in anatomic single-bundle anterior cruciate ligament reconstruction. They found that a femoral tunnel positioned in the anatomic center of the femoral origin of the ACL, as opposed to the anteromedial position, provides the greatest amount of rotatory and anterior stability.

 

Figure A shows AP and lateral radiographs of the knee. Note there is a autograft ACL reconstruction with vertical placement of the femoral and tibial tunnel.

 

Incorrect Answers:

Answer 1: Physical examination revealed a negative external rotation dial test which would likely rule out a posterolateral corner injury. Answer 2: Femoral tunnel placement that did not restore the anteromedial ACL bundle would result in greater anteroposterior instability. Answer 3: A tibial tunnel that was positioned too anterior would cause the knee to be tight in flexion with impingement in extension. Answer 5: A tibial tunnel that was positioned too posterior would cause significant knee instability with a grossly positive Lachman and Pivot shift.

 

(SAE08OS.179) What is the primary mechanism by which anabolic steroids increase muscle tissue? Review Topic

 

  1. Increased production of cortisol

  2. Increased production of messenger RNA

  3. Increased secretion of vascular endothelial factor

  4. Decreased production of HDL

  5. Attenuation of the effects of cortisol

 

PREFERRED RESPONSE 2

 

Anabolic steroids have many effects on the body. Increased muscle mass occurs specifically through increased production of messenger RNA. HDL levels usually decrease but do not affect muscle. Also, steroids act to change the effects of cortisol to decrease catabolism.

 

 

(SAE11AN.54) A 28-year-old man reports an episode of buckling and giving-way of his right knee. Figure 54a and 54b show a radiograph and sagittal MRI scan. What is the most likely diagnosis? Review Topic

 

 

 

  1. Anterior cruciate ligament rupture

  2. Posterior cruciate ligament rupture

  3. Quadriceps tendon rupture

  4. Patellar tendon rupture

  5. Displaced bucket-handle meniscus tear

PREFERRED RESPONSE 4

 

The radiograph shows patella alta consistent with a rupture of the patellar tendon. The MRI scan confirms disruption of the patellar tendon from the inferior pole of the patella. The cruciate ligaments are not visualized in this image, and would not result in patella alta. Quadriceps tendon rupture would result in patella baja. There is no evidence of meniscal tearing on these images.

 

 

(SAE08OS.190) The term anorexia athletica refers to a problem whose criteria include all of the following, EXCEPT Review Topic

 

  1. gastrointestinal complaints.

  2. restriction of caloric intake.

  3. presence of an affective disorder.

  4. weight loss.

  5. menstrual dysfunction.

 

 

PREFERRED RESPONSE 3

 

Anorexia athletica is a newer term that may generally replace the "triad" of disordered eating, menstrual dysfunction, and osteoporosis. True osteoporosis is actually relatively uncommon in the female athlete, and thus the traditional diagnostic criteria are rarely met. In anorexia athletica, multiple signs and symptoms are used to reach the diagnosis. Specifically, however, a diagnosis of anorexia athletica requires the absence of any affective disorder, such as depression.

 

 

(SAE10SM.45) Which of the following best describes the pathologic anatomy of cam impingement of the hip? Review Topic

 

  1. Retroversion of the acetabulum

  2. Posteroinferior labral tears

  3. Morphologic abnormality of the femoral head

  4. Femoral anteversion

  5. Femoral head osteonecrosis

 

PREFERRED RESPONSE 3

 

Cam impingement creates shearing forces that result in an outside-in directed detachment of the labrum in the anterosuperior quadrant. Retroversion of the acetabulum is associated with pincer impingement. The impingement is exhibited with hip flexion. Cam impingement involves a morphologic abnormality of the femoral head. Pincer lesions result from stresses of a normal femoral neck against an abnormal acetabular rim. Cam impingement is not associated with osteonecrosis.

 

 

(SAE07PE.30) A 12½-year-old boy reports intermittent knee pain and limping that interferes with his ability to participate in sports. He actively participates in football, basketball, and baseball. He denies any history of injury. Examination shows full range of motion without effusion. Radiographs reveal an osteochondritis dissecans (OCD) lesion on the lateral aspect of the medial femoral condyle. MRI scans are shown in Figures 14a and 14b. Initial treatment should consist of Review Topic

 

 

 

  1. activity modification

  2. arthroscopic evaluation of fragment stability.

  3. transarticular drilling of the lesion with 0.045 Kirschner wire.

  4. arthroscopic excision of the fragment and microfracture of underlying cancellous bone.

  5. excision of the fragment and mosaicplasty.

 

 

PREFERRED RESPONSE 1

 

This skeletally immature patient has a small OCD lesion that appears stable, and he has not undergone any treatment. Therefore, a trial of activity modification and limited weight bearing until pain resolves is the best initial choice.

 

Cessation of sport activities for 4 to 6 months may allow healing of the lesion. Surgical treatment of juvenile OCD lesions is reserved for unstable lesions, patients

 

who have not shown radiographic evidence of healing and are still symptomatic after

6 months of nonsurgical management, or patients who are approaching skeletal maturity. Good results with stable in situ lesions that have failed to respond to nonsurgical management have been reported with both transarticular and retroarticular drilling. Results after excision alone are poor at 5-year follow-up, and it is unclear if microfracture will improve the long-term outcome. Mosaicplasty may be the next best option for patients who remain or become symptomatic after excision of the fragment and microfracture.

 

Wall et al. reviewed juvenile OCD. They state that JOCD has better potential for healing than adult OCD, but several series have shown up to a 50% failure to heal with nonsurgical techniques. The presence of a loose body is an indication for surgical fixation, drilling or regenerative procedures, depending on the presence/extent of subchondral bone sclerosis and the surgeon's experience.

 

Figure A and B are coronal MRI images showing a stable appearing JOCD lesion of the medial femoral condyle.

Incorrect

Answer 2-5: The most suitable treatment conservative, including activity modification.

in the first

Answers: stages of the disease is

 

 

(SBQ13PE.9) A 6-year-old boy complains of a 'clunking' sensation in his left knee. He has no associated pain and denies trauma. He can elicit the sensation when moving his knee from flexion into full extension. He is otherwise healthy, with no birth or developmental issues. On examination, there is a palpable clunk felt over the anterior knee through range of motion. There is no obvious instability or tenderness and he had normal patellar tracking. An AP radiograph of the knee is shown in Figure A. What would be the most likely diagnosis? Review Topic

 

 

 

  1. Agenesis of the anterior cruciate ligament

  2. Thickened medial plica

  3. Grade IV chondral flap

  4. Pigmented villonodular synovitis

  5. Abnormal meniscal morphology

 

 

PREFERRED RESPONSE 5

 

This child presents with an asymptomatic click in the knee. It is associated with widening of the lateral joint space of the knee on X-ray. These features are highly suggestive of a discoid lateral meniscus in this age category.

 

The principal diagnostic feature of a discoid meniscus is the complaint of snapping or clicking in the knee. Children are usually asymptomatic. Although, less frequently, children may present with pain that is largely secondary to an underlying meniscal tear. MRI scans of the knee have show to have the greatest sensitivity for identifying discoid menisci. The presence of a contiguous central meniscus on three consecutive slices is usually indicative of the diagnosis. Treatment is mostly focused on conservative modalities. Surgical intervention is reserved for symptomatic cases with recurrent locking, swelling or persistent pain is present in older children.

 

Kramer et al. looked at the presentation of pediatric knee pain. They showed that the lateral meniscus is more commonly affected than the medial meniscus. The majority of discoid tears occur in the posterior or middle aspect of the discoid meniscus.

 

Figure A shows an AP radiograph of a pediatric knee. There is an increased lateral joint space suggestive of a discoid meniscus.

 

Illustrations A-C show an MRI of the knee with 3 consecutive coronal cuts showing an abnormal appearing discoid meniscus.

 

Incorrect Answers:

Answer 1: Agenesis of the ACL is rare. Symptoms may include knee pain and instability. It may also be associated with hypoplasia of the lateral femoral condyle, shallow intercondylar notch and agenesis of the tibial spine. Answer 2: Thickened medial plica is uncommon in children. These usually occur in older athletes and develop slowly over time Answer 3: A grade IV chondral flap is a full thickness lesion. This usually occur secondary to knee trauma, although not exclusively. Children rarely have sponateous and severe osteochondral lesions under the age of 9. Moreover, unlike adult OCD, it is well documented that spontaneous healing with good results can often be obtained by conservative measures and observation. Answer 4: Pigmented villonodular synovitis is an uncommon disease. Presenting complaints commonly involve one joint, most often the knee or hip. Symptoms include pain and swelling. The physical examination is usually completely normal.

 

(OBQ15.236) A 26-year-old female twists her knee while playing soccer and sustains the injury shown in Figure A. She undergoes a reconstructive surgery and is started on an accelerated rehabilitation program. Which of the following is true regarding this type of rehabilitation protocol? Review Topic

 

 

 

  1. Increased knee laxity compared with nonaccelerated rehabilitation programs

  2. No weightbearing allowed for 2-weeks post-operatively

  3. Accelerated rehabiliation program duration is 32-weeks

  4. Equal outcomes compared with nonaccelerated rehabilitation programs

  5. Improved range of motion compared with nonaccelerated rehabilitation programs

 

PREFERRED RESPONSE 4

 

The literature indicates that there is no difference in outcomes of accelerated rehabilitation programs compared with nonaccelerated rehabilitation programs after anterior cruciate ligament (ACL) reconstruction.

 

Rehabilitation after ACL reconstruction is necessary for a successful surgical outcome and remains an important topic of research interest. Accelerated programs allow for earlier weightbearing and certain strengthening exercises, rather than waiting several weeks before performing certain activities which put strain on the graft. Despite initial concerns that accelerated rehabilitation programs may lead to worse outcomes, many studies have demonstrated comparable outcomes between accelerated and nonaccelerated rehabilitation programs. No significant long-term differences have been reported regarding ACL laxity, range of motion, strength, or return to sports. Use of an accelerated program does not imply earlier return to sports. The optimal time to return to sports has not yet been elucidated.

 

Beynnon et al. performed a randomized controlled trial of patients who underwent ACL reconstruction with bone-patellar tendon-bone autograft comparing rehabilitiaton with either accelerated (19 week) or nonaccelerated (32 week) programs. At final evaluation, patients in both programs had the same increase in the envelope of knee laxity, clinical assessment, functional performance, proprioception,

 

and

thigh

muscle

strength.

Kruse et al. presented a systematic review evaluating studies on rehabilitation following ACL reconstruction. The authors found that accelerated rehabilitation does not appear to be harmful. It is likely safe for patients to begin immediate postoperative weight bearing, range of motion from 0-90 degrees of flexion, and perform closed-chain exercises. Bracing following ACL reconstruction is neither necessary nor beneficial and often adds to the cost of the procedure.

 

Figure A is a sagittal MRI demonstrating a complete ACL tear.

Incorrect

answers:

Answer 1: Accelerated and nonaccelerated rehab programs have been shown in recent studies to result in the same increase in envelope of the knee laxity. Answer 2: Accelerated rehab programs allow for immediate weightbearing as tolerated with crutches. Answer 3: The duration of the nonaccelerated rehab program is 32-weeks. The accelerated rehab program used in prior studies is 19-weeks. Answer 5: No significant long-term differences in range of motion exist between accelerated and nonaccelerated rehab programs.

 

 

(SAE10SM.84) Endurance training stimulates which of the following physiologic adaptations in the athlete? Review Topic

 

  1. Selective hypertrophy of type II muscle fibers

  2. Decreased concentration of Krebs cycle enzymes

  3. Increased rate of glycogen depletion

  4. Increased sympathetic nervous system activity

  5. Increased storage and utilization of intramuscular lipids

 

PREFERRED RESPONSE 5

 

Endurance training causes selective hypertrophy of type I muscle fibers. It stimulates an increase in the enzymes of the Krebs cycle which increases the capacity for aerobic ATP resynthesis during exercise. There is a decrease in the rate of glycogen depletion. Depletion of glycogen has been linked to fatigue during endurance exercise. Endurance training blunts the catecholamine response and may contribute to the reduction in heart rate observed for the same exercise intensity following training. The greater use of lipid reduces the contribution of carbohydrate to ATP resynthesis and preserves muscle glycogen.

 

(SAE10SM.65) During preparation for the NCAA wrestling championships, a participant reports the development of vesicular lesions on his right chest wall that are mildly painful; however, they have not affected his ability to wrestle. How should this athlete be managed? Review Topic

 

  1. He may wrestle if his lesions are covered.

  2. He may wrestle if he is on oral antiviral agents for 48 hours.

  3. He may wrestle immediately with no other treatment.

  4. He cannot wrestle until the lesions are scabbed over and there are no new lesions for at least 72 hours.

  5. He cannot wrestle for 2 weeks.

 

PREFERRED RESPONSE 4

 

Herpes simplex virus (HSV) can cause serious outbreaks on athletic teams, especially wrestling. HSV is highly contagious; it is secreted from active blisters, saliva, and mucous membranes. For wrestlers, the NCAA states that the athlete must be free from systemic symptoms and any new blisters for 72 hours before being allowed to participate. Also, all lesions must be dry and crusted and at least 120 hours of antiviral therapy should have been instituted.

 

 

(OBQ15.171) Which of the following is more likely to occur with use of a bone patellar bone allograft instead of a bone patellar bone autograft for anterior cruciate ligament (ACL) reconstruction in an 18-year-old high school or collegiate athlete? Review Topic

 

  1. Septic arthritis

  2. Early onset osteoarthritis

  3. ACL graft re-rupture

  4. Cyclops lesion formation

  5. Femoral and tibial tunnel osteolysis

 

PREFERRED RESPONSE 3

 

The best reason to use an autograft (rather than an allograft) for anterior cruciate ligament (ACL) reconstruction in a young athlete is lower graft rupture rate.

 

Many factors can potentially contribute to the failure of ACL reconstructions, including the surgical technique, the selection of graft material, the integrity of the secondary restraints, the condition of the articular and meniscal cartilage, and postoperative rehabilitation. Early failure, usually within the first 6 months, most often is the result of technical errors, incorrect or overly aggressive rehabilitation,

 

 

premature return to sports, or failure of graft incorporation. Later failure, usually after one year, is more typically the result of recurrent injury.

 

Kaeding et al. report data from the MOON multicenter research consortium. They present Level 2 evidence that the odds of graft rupture with an allograft reconstruction are 4 times higher than those of autograft reconstruction in athletes aged 10-19 years old. For each age, the number of autograft ACL reconstructions (ACLRs) performed to prevent one failure is as follows: 14 years, 7 ACLRs; 18 years, 8 ACLRs; 22 years,

11 ACLRs; 30 years, 25 ACLRs; 40 years, 50 ACLRs.

 

Krych et al. present a systematic review of prospective trials using BTB autograft and BTB allograft tissue for ACL reconstruction with a minimum 2-year follow-up. They found that BTB allograft patients were more likely to rupture their graft than BTB autograft patients (OR, 5.03; P = .01), however once irradiated and chemically processed allografts were excluded there was no statistical difference in graft re-rupture between the groups.

 

Greenberg et al. conducted a study of nearly 1300 patients and found no increased clinical risk of infection with the use of allograft tissue compared with autologous tissue for primary anterior cruciate ligament reconstruction.

 

Incorrect Answers:

1: There is no proven increased clinical risk of postoperative superficial or deep bacterial infection with the use of allograft tissue compared with autologous tissue for primary anterior cruciate ligament reconstruction. 2: There is no definitive data suggesting different arthritis progression rates based on autograft versus allograft. 4: Cyclops lesion formation is not related to graft choice. 5: There is some literature that tunnel osteolysis and enlargement is more common and greater with hamstring soft tissue grafts, however it does not appear to affect the clinical outcome in the first 2 postoperative years. There is no definitive data comparing BTB auto vs allograft in regards to tunnel osteolysis.

 

(OBQ15.155) Which of the following structures is predominantly composed of Type 1 collagen? Review Topic

 

  1. Epithelial Basement Membrane

  2. L4 disc nucleus pulposus

  3. Anterior cruciate ligament

  4. Medial femoral condyle articular cartilage

  5. Dupuytren's contracture tissue

PREFERRED RESPONSE 3

 

The most common fiber type in the Anterior cruciate ligament is Type 1 collagen.

 

Type 1 collagen accounts for more than 90% of the total collagen content in the body. Type 1 collagen is found in bone, ligament, tendon, meniscus, annulus of intervertebral discs, skin, healed cartilage, scar tissue, and nerves.

 

Duthon et al present a review of the ACL and its histologic and mechanical properties. The ACL has a mean length of 32 mm and a width of 7-12 mm. They state that the ACL has a microstructure of collagen bundles of multiple types (mostly type I) and a matrix made of a network of proteins, glycoproteins, elastic systems, and glycosaminoglycans with multiple functional interactions.

Incorrect

1: Epithelial

2: L4 disc

Basement

Membrane is made up of Type

nucleus pulposus is composed of mostly Type

Answers:

4 collagen.

2 collagen

4: Medial femoral condyle articular cartilage is comprised mostly of Type 2 collagen. 5: Dupuytren's contracture tissue is mostly Type 3 collagen.

 

 

(SAE10SM.72) What is the most common physical finding in a patient with femoroacetabular impingement (FAI)? Review Topic

 

  1. Increased external rotation

  2. Increased abduction

  3. Decreased external rotation

  4. Decreased flexion and internal rotation

  5. Decreased adduction

 

PREFERRED RESPONSE 4

 

A loss of flexion and internal rotation are hallmarks of FAI. With the hip flexed 90 degrees, maximal internal rotation testing is also known as the anterior impingement test, causing deep groin pain and reproduction of symptoms. Occasionally, a posterior impingement test will be positive with extension and external rotation. There are a variety of causes of FAI; however, the pathology limits motion as the femur (cam) and acetabulum (pincer) contact one another. Also, only one location needs to be present, such as cam-type or pincer-type versus both cam-pincer lesions to cause symptoms.

 

(SAE07HK.35) A patient reports pain in the hip with functional positioning. With the patient supine, pain in which of the following positions would be typical for femoral acetabular impingement? Review Topic

 

  1. Hip is internally rotated, passively flexed to 90 degrees, and adducted

  2. Hip is internally rotated, passively flexed to 90 degrees, and abducted

  3. Hip is externally rotated, maximally flexed to 90 degrees, and adducted

  4. Hip is externally rotated, passively flexed to 90 degrees, and abducted

  5. Hip is externally rotated, maximally flexed, and abducted

 

PREFERRED RESPONSE 1

 

Patients with dysplasia often have a hypertrophic labrum. Abnormal contact between the femoral neck and the acetabular rim leads to labral injury, especially in the anterior-superior acetabular zone. Typically, young patients with the condition report pain with activity or long periods of sitting or driving. The hips often have limited motion, in particular in internal rotation and flexion. Forceful adduction with the maneuver causes pain.

 

 

(SBQ13PE.10) Which statement is true regarding discoid menisci? Review Topic

 

  1. Most commonly involves the medial meniscus

  2. Bilateral in >75% of cases

  3. Asymptomatic discoid meniscus should undergo saucerization

  4. Radiographs will commonly show a hyperplastic lateral intercondylar spine

  5. Radiographs will commonly show squaring of affected condyle with cupping of tibial plateau

 

PREFERRED RESPONSE 5

 

Radiographs of knees with discoid menisci will commonly show squaring of affected condyle (lateral>medial) with cupping of tibial plateau.

 

Discoid meniscus refers to the abnormal development of a hypertrophic and discoid shaped meniscus. It occurs in 3-5% of the population and it is considered the most common cause of a symptomatic clicking or clunking in a childs knee. The lateral meniscus is most commonly affected and it will occur bilaterally in 25% of affected

 

 

people. The Watanabe Classification describes the 3 types of discoid menisci. Type 1

= Incomplete, Type 2 = Complete, Type 3 = Wrisberg (lack of posterior meniscotibial attachment to tibia)

 

Kramer et al. looked at the presentation of pediatric knee pain. They showed that the lateral meniscus is more commonly affected than the medial meniscus. The majority of discoid tears occur in the posterior or middle aspect of the discoid meniscus.

 

Lee et al. retrospectively reviewed 36 patients aged less than 15 years who underwent arthroscopic procedures for torn discoid menisci. The mean patient age at the time of surgery was 9.5 years. They showed that partial meniscectomy yielded better radiologic results than subtotal/total meniscectomy for torn discoid menisci in this population.

 

Illustration A shows the 3 classifications of discoid menisus as originally described by Watanabe. Type 4 is a ring type discoid that was not originally described by Watanabe in his 1978 paper. Illustration B shows an AP and lateral radiograph of a discoid meniscus knee. Note squaring of affected lateral condyle in the presence of a lateral discoid meniscus. Illustration C shows 4 consecutive sagittal MRI images with meniscus continuity. It is important to note that the diagnosis of discoid menisci can be made when 3 or more 5mm sagittal images show meniscal continuity.

 

Incorrect Answers:

Answer 1: Most commonly involves the lateral meniscus Answer 2: Bilateral in ~25% of cases Answer 3: Asymptomatic discoid meniscus should be observed. Saucerization is considered with symptomatic discoid meniscus or associated tears. Answer 4: Radiographs will commonly show a HYPOplastic lateral intercondylar spine.

 

(SAE07HK.82) Figure 50 shows the cross table lateral radiograph of a 31-year-old paratrooper who has recalcitrant groin pain. The pain is worse after activities such as standing or sitting (driving). Examination reveals that pain can be reproduced by internal rotation of the leg with the hip and knee in 90 degrees of flexion. Extensive nonsurgical management has failed to provide relief. What is the treatment of choice? Review Topic

 

 

 

  1. Periacetabular osteotomy

  2. Femoral neck osteotomy

  3. Femoroacetabular osteoplasty

  4. Hip arthroscopy and labral debridement

  5. Hip arthrodesis

 

PREFERRED RESPONSE 3

 

The radiograph reveals the classic “bump” that is seen in patients with femoroacetabular impingement (FAI). Ganz and associates described two types of FAI. This patient has cam impingement, which describes a nonspherical femoral head being forced into the acetabulum during hip motion and resulting in labral and chondral injury. Hip arthroscopy and labral debridement is unlikely to control the symptoms because the underlying anatomic abnormality is often difficult to address with arthroscopy. The treatment involves surgical dislocation of the hip with preservation of the blood supply to the femoral head, removal of the asphericity on the femoral side (femoral osteoplasty), and removal of the acetabular rim (acetabular osteoplasty) if the latter is found to contribute to impingement.

 

 

(SAE10BS.50) A previously healthy 20-year-old male wrestler is seen for evaluation and treatment of draining sores of the forearm. Empiric treatment for cellulitis was started with oral clindamycin with improvement. Culture of the drainage reveals methicillin-resistant staphylococcus aureus (MRSA). Sensitivities at 48 hours demonstrate additional resistance to erythromycin and a positive D-zone test. Definitive antibiotic treatment until resolution should consist of which of the following? Review Topic

 

  1. Change to oral doxycycline

  2. Change to IV cefazolin

  3. Change to IV vancomycin

  4. Change to oral rifampin

  5. Continuation of oral clindamycin

 

PREFERRED RESPONSE 1

 

Based on the description of the infection and the history of close contact, the clinical scenario is most consistent with community-acquired MRSA (CA-MRSA). It is important to distinguish CA-MRSA and hospital-acquired MRSA (HA-MRSA) as the two display different sensitivities to antibiotics. Antibiotic selection is based on sensitivity and severity of infection. Because this infection is superficial and without

 

any signs of systemic illness, an oral antibiotic regimen is appropriate. When culture results reveal resistance to erythromycin, then a D-zone test should be performed to check for inducible clindamycin resistance. The D-zone test is performed by plating the sample on an agar and placing antibiotic disks made of clindamycin and erythromycin on the agar. A zone of inhibition in the shape of the letter D is seen with an inducible strain. If the D-zone test is positive, then clindamycin should not be used because the strain of MRSA can become resistant to the treatment. Therefore, because of the positive D-zone test, the antibiotic should be changed to oral doxycycline. IV antibiotics are not indicated for this infection. Oral rifampin should never be used as a single agent as resistance rapidly develops.

 

 

(SAE11OS.95) A 29-year-old man sustained an injury when he was playing basketball, landing on his left knee while jumping for a rebound. He had vague pain in the anterior aspect of the knee for several weeks. The initial radiographs were negative with the exception of a large traumatic effusion. Examination reveals no apparent ligament instability but a significant extension lag of 30 degrees. There was a palpable defect above the superior pole of the patella. What is the most appropriate management? Review Topic

 

  1. MRI scan

  2. Diagnostic arthroscopy

  3. Surgical repair of a ruptured quadriceps tendon

  4. Knee immobilizer for 6 weeks, followed by a sport brace

  5. Limited weight bearing for 3 weeks, followed by physical therapy

 

PREFERRED RESPONSE 3

 

This is the classic presentation of a tendon disruption in an active athlete that may represent chronic strain or weakening of the tendon insertion. The factors that lead to this condition are multitude, including biomechanic and cytologic, but there is little evidence that inflammation is an active factor. Surgical treatment is straightforward and logical. Suture anchors have been compared with simple holes made in the patella for suturing the tendon, with no apparent biomechanic advantage.

 

(SAE10SM.78) An 18-year-old high school football player exits the field after making a tackle on the opening kickoff. He reports "feeling out of it" and states that he has a headache. He does not recall any loss of consciousness and has no amnesia. He is unable to list the months of the year in reverse order on questioning. He does not return to the game and feels normal at the completion of the game. What is the most sensitive test in assessing deficits after mild traumatic brain injury? Review Topic

 

  1. Head CT

  2. MRI of the head

  3. Neuropsychologic testing

  4. Radiographs of the skull

  5. Sideline assessment

 

PREFERRED RESPONSE 3

 

Most imaging studies in mild traumatic brain injury will be normal. Neuropsychologic testing is the most sensitive test in assessing mild deficits after traumatic brain injury. Sideline assessment is important but less sensitive in assessing deficits. The precise role of neuropsychologic testing in determining return to play has not been fully defined.

 

 

(SAE10SM.14) A 24-year-old professional male soccer player has lower abdominal pain on exertion. He has pain with resisted hip adduction and with sit-ups. There is no palpable inguinal hernia with a Valsalva maneuver. Nonsurgical management has failed to provide relief. After ruling out malignancies, what is the next most appropriate step in management? Review Topic

 

  1. Additional nonsurgical management

  2. Referral to a sports hernia surgeon

  3. Ultrasound of the scrotum

  4. CT of the pelvis

  5. Cortisone injection

 

PREFERRED RESPONSE 2

 

The patient has a sports hernia or athletic pubalgia. The exact nature of this pathology is not well understood. MRI scans are not very helpful in making a diagnosis. In high-performance athletes who have failed to respond to nonsurgical management, surgical intervention is needed to strengthen the anterior pelvic floor. Additional nonsurgical management at this point will not improve symptoms. Cortisone injection will not strengthen the pelvic floor. CT scan of the pelvis is valuable to rule out bony injuries such as osteitis pubis. Ultrasound of the scrotum will have no additional diagnostic use in management of this patient.

 

(SAE11AN.48) An 18-year-old man sustains a twisting injury to the left knee while playing football. An MRI scan is shown in Figure 48. What is the most likely diagnosis? Review Topic

 

 

 

  1. Anterior cruciate ligament rupture

  2. Posterior cruciate ligament rupture

  3. Medial meniscus tear

  4. Lateral meniscus tear

  5. Osteochondral lesion

 

PREFERRED RESPONSE 4

 

The MRI scan shows a displaced, bucket-handle lateral meniscus tear. The sagittal view shows the typical "large anterior horn" sign, or "double meniscus" sign in which the displaced bucket-handle fragment appears just anterior to the native anterior horn of the lateral meniscus. The presence of the fibula on the sagittal view confirms this as the lateral compartment. The image is lateral and the cruciate ligaments are not visualized. The articular cartilage shown does not demonstrate an osteochondral lesion.

 

 

(OBQ15.275) Two football players present with knee pain and instability after being tackled in a game. Figures A and B are injury radiographs for Player A and B, respectively. Which of the following is correct regarding the radiographic findings of these 2 players? Review Topic

 

 

  1. Player A sustained an avulsion fracture of the anterolateral ligament. Player B sustained an avulsion fracture of the arcuate complex.

  2. Player A sustained an avulsion fracture of the arcuate complex. Player B sustained an avulsion fracture of the anterolateral ligament

  3. Both players sustained avulsion fractures of anterolateral ligament.

  4. Player A sustained an avulsion fracture of the iliotibial band. Player B sustained an avulsion fracture of the biceps femoris.

  5. Player A sustained a Segond fracture. Player B sustained a reverse Segond fracture.

 

PREFERRED RESPONSE 1

 

Player A sustained an avulsion fracture of the anterolateral ligament (ALL). Its tibial attachment is midway bewteen Gerdy's tubercle and the fibular head. Player B sustained an avulsion fracture of the arcuate complex.

 

A Segond fracture is commonly associated with ACL tear (75%). It is caused by internal rotation and varus load (unlike the more common cause of ACL tear, which is valgus stress). The arcuate sign is an avulsion fracture of the proximal fibula at the site of insertion of the arcuate ligament complex, and is usually associated with cruciate ligament injury (~90% of cases). The fracture fragment is attached to the LCL or biceps femoris tendon.

 

Dodds et al. reviewed the anatomy of the ALL in 4 cadaveric knees. They found that the ligament was isometric from 0 to 60° of flexion and slackened with the knee flexed to 90°. It passed superficial to the LCL and knee capsule. Mean length was 59 mm, mean width was 6 mm. The femoral attachment was slightly posterior to both the lateral epicondyle and the femoral attachment of the LCL. The tibial attachment just distal to the anterolateral rim of the plateau, mid-way between the head of the fibula and Gerdy’s tubercle. An arcuate complex avulsion fracture is an avulsion of the fibular styloid attachment of the arcuate complex.

 

 

Claes et al. (2014) compared the distance between the tibial ALL footprint to the center of Gerdy's tubercle (GT, in cadavers) vs the distance between GY and a Segond fracture (on CT). They found the former to be 22mm, and the latter to be

22.4mm, and this was not statistically significant. They concluded that the Segond fracture is a bony avulsion of the ALL.

 

Claes et al. (2013) investigated the ALL in 41 cadaveric knees. They found that the ALL ran obliquely to the anterolateral proximal tibia with attachments to the lateral meniscus. The insertion on the tibia was midway between Gerdy's tubercle and the tip of the fibula head

 

Figure A shows a Segond fracture (ALL avulsion fracture). Figure B shows the arcuate sign (arcuate complex avulsion fracture). Illustration A shows the ALL (arrowhead) overlying the LCL (star). Illustration B is a diagrammatic representation of of the ALL (59mm long, femoral attachment 8mm behind lateral epicondyle, tibial attachment 11mm below tibial joint line, 18mm behind Gerdy's tubercle, 17mm anterior to fibular head. Illustrations C and D are CT and MRI imaging reconciliation with anatomical specimens showing that a Segond fracture corresponds to the ALL attachment. Illustration E highlights the arcuate sign from Figure B.

 

Incorrect Answers:

Answers 2, 3: Figure A shows a Segond fracture (ALL avulsion fracture). Figure B shows the arcuate sign (arcuate complex avulsion fracture). Answer 4: The ITB attaches at Gerdy's tubercle. Answer 5: A reverse Segond fracture is an avulsion of the deep capsular component of the medial collateral ligament.

 

(SAE08OS.133) A 9-year-old boy has lateral right knee pain. An MRI scan shows a discoid lateral meniscus with a partial tear in its central portion. Treatment should consist of Review Topic

 

  1. arthroscopic saucerization of the meniscus.

  2. lateral total menisectomy.

  3. meniscal transplant.

  4. arthroscopy and repair of the central tear.

  5. casting for 6 weeks followed by physical therapy.

 

PREFERRED RESPONSE 1

 

A tear of the mid portion of a stable discoid lateral meniscus should be treated with a partial menisectomy with saucerization. Lateral total menisectomy is contraindicated because of the poor long-term results following this procedure. Meniscal transplant and casting do not have a role in this scenario, although meniscal repairs may be needed for peripheral meniscal instability.

 

(SAE10SM.34) Which of the following cohorts of patients is at highest risk of a future anterior cruciate ligament (ACL) tear? Review Topic

 

  1. Men with a hip abduction moment during landing

  2. Men with a neutral hip abduction-adduction moment during landing

  3. Men with varus knee abduction moment during landing

  4. Women with a hip adduction moment during landing

  5. Women with a knee valgus moment during landing

 

PREFERRED RESPONSE 5

 

Hewett and associates reported in a study of 205 female athletes that female athletes, with increased dynamic valgus and high abduction loads, were at increased risk of ACL injury. The same investigators in an earlier study of 81 high school basketball players reported that female athletes landed with greater total valgus knee motion and a greater maximum valgus knee angle than male athletes. Female athletes were also found to have significant differences between their dominant and nondominant side in maximum valgus knee angle. Lephart and associates reported that in single-leg landing and forward hop tasks that female athletes had significantly less knee flexion and lower leg internal rotation maximum angular displacement, and less knee flexion time to maximum angular displacement than males. Females with an adduction moment during landing should have a lower incidence of ACL tears. Males in general have a lower incidence of ACL tears.

 

 

(SAE10BS.9) What is the most important muscle adaptation resulting from endurance training? Review Topic

 

  1. Increased recruitment of motor units

  2. Formation of additional myofibers (hyperplasia)

  3. Hypertrophy of individual myofibrils

  4. Increase in capillary density

  5. Increased activity of glycolytic enzyme

 

PREFERRED RESPONSE 4

 

The most important muscle adaptation resulting from endurance training is an increase in capillary density. Increased capillary density improves blood and nutrient delivery and elimination of metabolic waste. Increased recruitment of motor units, hyperplasia, and hypertrophy are all adaptations that occur with resistance training or strength training. There is no significant change in glycolytic enzyme activity.

 

(SAE10SM.10) A 32-year-old man underwent a total medial meniscectomy 2 years ago. He now reports pain and recurrent swelling for the past 3 months. Work-up includes full standing hip-knee-ankle radiographs, standing AP radiographs of both knees in full extension, an axial view of the patellofemoral joint, and a 45-degree flexion AP radiograph. Contraindication to meniscus allograft transplantation includes which of the following? Review Topic

 

  1. 4 mm of tibiofemoral joint space on a 45-degree weight-bearing AP radiograph

  2. Intact anterior cruciate ligament on MRI and physical examination

  3. Recurrent effusions

  4. Flattening of the femoral condyles

  5. Healed high tibial osteotomy

 

PREFERRED RESPONSE 4

 

Flattening of the femoral condyles indicates the onset of significant arthritis of the joint and is a contraindication to meniscus allograft transplantation. Criteria to proceed with allograft transplantation includes prior total meniscectomy, age of 50 years or younger, BMI of less than 30, clinical symptoms of pain in the involved tibiofemoral compartment, 2 mm or more of tibiofemoral joint space on a 45-degree weight-bearing AP radiograph, ligamentous stability, normal alignment, and no radiographic evidence of advanced arthrosis. Recurrent effusions are associated with chronic meniscus deficiency, and is one criteria for meniscal transplantation. High tibial osteotomy is often considered in conjunction with meniscal transplantation to correct tibiofemoral malalignment.

 

 

(OBQ13.152) A 25-year-old patient undergoes the procedure seen in Figure A. Which of the following statements best describes the incorporation of the graft and biopsy results of the graft at one year? Review Topic

 

 

 

  1. The transplanted chondrocytes are viable and articular cartilage heals. Biopsy shows type I collagen.

  2. The transplanted chondrocytes are viable and articular cartilage heals. Biopsy shows type II collagen.

  3. The transplanted chondrocytes are nonviable and cartilage is used as a scaffold for growth of new articular cartilage. Biopsy shows type II collagen.

  4. The transplanted chondrocytes are nonviable and articular cartilage is gradually replaced by fibrocartilage. Biopsy shows type I collagen.

  5. The transplanted chondrocytes are nonviable and articular cartilage is gradually replaced by fibrocartilage. Biopsy shows mixture of type I and II collagen.

 

 

PREFERRED RESPONSE 2

 

The patient underwent an osteochondral autograft transfer (OAT) with multiple plugs (also known as mosaicplasty) for a full-thickness chondral defect of the medial femoral condyle. The chondrocytes in the graft remain viable, the transferred cartilage heals, and biopsy reveals articular cartilage composed primarily of type II collagen.

 

Articular cartilage defects can be treated by a variety of methods including debridement, fixation of unstable osteochondral fragments, marrow stimulation techniques (microfracture, abrasion chondroplasty), cartilage replacement techniques (osteochondral autograft and allograft) and cellular techniques (autologous chondrocyte implantation). Osteochondral autograft transfer is performed by harvesting normal articular cartilage with underlying bone from lesser weightbearing areas (e.g. intercondylar notch) and transferring the graft to a recipient socket at the site of the chondral defect. Graft incorporation occurs by integration of the bony graft into the subchondral bone and healing of the overlying cartilage layer.

 

Hangody et al. reviewed the outcomes of autologous osteochondral mosaicplasty in professional athletes. They found successful outcomes similar to that of less athletic patients, despite a higher rate of preoperative osteoarthritic changes in the athletic population. The authors noted that histological evaluation revealed good graft incorporation in all 11 cases.

 

Alford et al. authored a two part Current Concepts article on cartilage restoration. They constructed an algorithm (Illustration A) highlighting many factors that impact treatment choice, including patient activity level and defect characteristics such as location and size. This algorithm also illustrates comorbidities (malalignment, ligament insufficiency) that warrant correction prior to addressing the chondral defect.

 

Figure A shows a full-thickness chondral defect (left) and subsequent osteochondral autograft transfer (right). Illustration A is a treatment algorithm for the management of chondral defects, as discussed above. Illustration B shows a microfracture procedure, a marrow stimulation technique resulting in fibrocartilage filling of the chondral defect. Illustration C shows an osteochondral allograft transplant, a cartilage replacement technique useful for large defects in which donor graft is obtained from a cadaver hemicondyle and transferred to a recipient socket at the site of the chondral defect. Illustration D shows the autologous chondrocyte implantation technique, a two-stage procedure consisting of 1. Cartilage biopsy for growth of autologous

chondrocytes, and 2. Subsequent injection of autologous chondrocytes beneath a periosteal patch.

Incorrect

Answers:

Answer 1: Type I collagen is more commonly found in reparative fibrocartilage (following microfracture), bone, ligaments and tendons. Biopsy in this patient would show articular cartilage composed of predominately type II collagen. Answers 3, 4, 5: The transplanted chondrocytes and overlying articular cartilage remain viable. The cartilage is not used as a scaffold for new growth, nor is it replaced by fibrocartilage.

 

 

(OBQ13.248) Figure A shows the 2 bundles of the ACL dissected from a cadaveric knee off their bony attachments. They are labeled Bundle A and Bundle B, respectively. Which of the following is true? Review Topic

 

 

 

  1. The tibial attachment of Bundle A is anterior to Bundle B. In extension, Bundle B is loose and Bundle A is tight.

  2. The tibial attachment of Bundle A is anterior to Bundle B. In flexion, Bundle B is loose and Bundle A is tight.

  3. The tibial attachment of Bundle B is anterior to Bundle A. In flexion, Bundle B is loose and Bundle A is tight.

  4. The tibial attachment of Bundle B is anterior to Bundle A. In flexion, Bundle A is loose and Bundle B is tight.

  5. The tibial attachment of Bundle B is anterior to Bundle A. In extension, Bundle A is loose and Bundle B is tight.

 

PREFERRED RESPONSE 2

 

Bundle A is the anteromedial (AM) bundle, which is longer, and is tight in flexion. Bundle B is the posterolateral (PL) bundle, which is shorter, and is loose in flexion. The AM bundle is attached anterior to the PL bundle on the tibia.

 

The ACL is comprised of 2 bundles. The AM bundle is longer than the PL bundle.

 

Their names reflect their relative anatomic positions on the tibial insertion site. On the femur, the AM bundle begins at the proximal-anterior aspect of the femoral insertion site, while the PL bundle begins at the posterior-inferior part. In flexion, the AM bundle is tight and the PL bundle is loose. In extension, the AM bundle is loose and the PL bundle is tight.

 

Bicer et al. reviewed the anatomy of the ACL. They found that the AM bundle was longer (32mm) compared with the PL bundle (18mm). PL bundle carries greater force near full extension, and the AM bundle carries greater force after 15-45° of flexion. Under combined rotatory loads (valgus and internal tibial torque at knee flexion

>30°), the AM bundle bore more force than the PL bundle.

 

Figure A shows the 2 bundles of the ACL. The AM bundle is longer than the PL bundle. The oft referred to length of ACL refers mainly to the length of the AM bundle. Illustrations A and B show the spatial relationships of the AM and PL bundles in a cadaveric knee. Illustration C shows the relative positions of the attachments of each bundle.

Incorrect

Answers:

Answer 1: In extension, Bundle B (PL) is tight, and Bundle A (AM) is loose. Answers 3 to 5: The tibial attachment of Bundle A (AM) is anterior to Bundle B (PL).

 

 

(OBQ13.99) A 35-year-old man presents with mechanical knee pain after a fall. An arthroscopic picture taken during diagnostic arthroscopy is shown in Figure A. His surgeon considers treatment with Technique B and Technique C, which are shown in Figures B and C, respectively. Which of the following statement is true with respect to Technique B and C? Review Topic

 

 

 

  1. A diminished immune response to transplanted chondrocytes is seen in Technique C because the dense cartilage matrix acts as a barrier that limits antigen exposure.

  2. In Technique C, healing is initiated by mesenchymal stem cell migration from subchondral bone. In Technique B, healing is initiated by allogeneic chondrocytes reimplanted beneath a periosteal patch.

  3. Grafts in Technique C are transferred to an antibiotic solution to kill microorganisms and stored at 4°C until use.

  4. A biopsy of the repair site at 3 months will reveal more Type I collagen in Technique B than in Technique C.

  5. Technique C is a 2-stage procedure. Technique B is a single-stage procedure.

 

PREFERRED RESPONSE 4

 

Microfracture (Technique B) heals with fibrocartilage, predominantly Type I collagen. Mosaicplasty/OAT (Technique C) transfers articular cartilage, containing predominantly Type II collagen.

 

The initial treatment for small articular cartilage defects is rest, NSAIDS and bracing. For larger defects (<2-4cm2), options include marrow stimulation procedures (abrasion chondroplasty, microfracture, osteochondral drilling) and autograft procedures (Mosaicplasty, a technique of osteochondral autograft transfer (OAT).

 

Alford et al. reviewed cartilage restoration. They defined a spectrum of treatment option ranging from (1) palliative (debridement/lavage), intended to reduce irritation

 

 

and inflammation, (2) reparative (marrow stimulation techniques), intended to recruit stem cells to proliferate fibrocartilage repair tissue, to (3) restorative (osteochondral grafting), designed to replace articular cartilage and subchondral bone as a single unit.

 

Magnussen et al. reviewed the treatment of cartilage defects in the knee. They compared autologous chondrocyte implantation (ACI), OATs, matrix-induced ACI, and microfracture. No one technique was superior to the rest. Outcomes for microfracture tended to be worse in larger lesions.

 

Figures A is an arthroscopic picture showing a grade IV chondral defect. Figure B shows microfracture. Figure C shows the multiple cylindrical plugs of mosaicplasty, a technique of osteochondral autograft transfer (OAT). Illustration A shows donor and recipient sites in mosaicplasty. Illustration B shows the spectrum of treatment options defined by Alford et al (MST = marrow stimulation; ACI = autologous chondrocyte implantation; OCG = osteochondral grafting with autograft/allograft).

 

Incorrect Answers:

Answer 1: An immune reaction is of concern only with osteochondral allograft transplantation.

Answer 2: Healing in Technique C (OATs) is by transplanted autologous chondrocytes. Healing in Technique B is by stem cells migrating from subchondral bone. ACI involves autologous (not allogeneic) chondrocytes expanded and reimplanted beneath a periosteal patch. Answer 3: Osteochondral allografts are cleansed in antibiotics prior to implantation. Answer 5: Both treatments are single-stage procedures. Only ACI involves 2 stages, with the interim period necessary for chondrocyte expansion.

 

 

(SAE13SM.17) When reconstructing the anterior cruciate ligament (ACL), what is the most common source of potential autograft failure? Review Topic

 

  1. Graft choice

  2. Tunnel position

  3. Tibial fixation

  4. Femoral fixation

 

PREFERRED RESPONSE 2

 

Technical failure is the most common reason for ACL reconstruction failure. Tunnel position is the most frequent cause for technical failure. Malpositioning of the tunnel affects the length of the graft, causing either decreased range of motion or increased graft laxity. Although graft choice is an important factor when planning an ACL reconstruction, overall outcomes with autograft tissues are fairly similar. Fixation of the graft at the femoral or tibial end is not as important as tunnel position.

 

(SAE13SM.82) Figure 82 is the MRI scan of a 15-year-old boy who has had knee pain with running for 5 months. Radiographs show an osteochondritis dissecans (OCD) lesion of the medial femoral condyle. What is the most appropriate treatment? Review Topic

 

 

 

  1. Arthroscopic or open reduction and internal fixation

  2. Arthroscopic loose body removal

  3. Activity restriction for up to 9 months

  4. Subchondral drilling

 

PREFERRED RESPONSE 1

 

OCD is an acquired lesion of the subchondral bone. Patients with OCD initially report nonspecific pain and variable amounts of swelling. Initial radiographs help identify the lesion and establish the status of the physes. An MRI scan is useful for assessing the potential for the lesion to heal with nonsurgical treatment. Nonsurgical treatment is appropriate for small, stable lesions in patients with open physes and focuses on activity restriction for 3 to 9 months. Surgical treatment is necessary for unstable or detached lesions. Stable lesions with intact articular cartilage can be treated with subchondral drilling to stimulate vascular ingrowth, with radiographic healing at an average of 4.4 months. Fixation is indicated for unstable or hinged lesions, and stabilization of the fragment can be achieved using a variety of implants through an arthroscopic or open approach. The fragment should be salvaged and the normal articular surface restored whenever possible.

 

 

(SAE10SM.60) A patient competing in a professional motocross race sustained a direct blow to the knee after falling off his bike at high speed. He sustained several lacerations as shown in Figure 60. He is able to actively extend his knee painlessly and his Lachman examination is negative. What is the most likely injury? Review Topic

 

 

  1. Anterior cruciate ligament tear

  2. Patella fracture

  3. Patellar tendon tear

  4. Tibial tubercle avulsion

  5. Posterior cruciate ligament tear

 

PREFERRED RESPONSE 5

 

It is important to recognize the injury pattern sustained by this motocross rider by inspection of his traumatic scars present anteriorly over the proximal tibia and the dorsum of the ankle and dorsum of the forefoot, indicating that his foot was in a plantar flexed position with a concomitant blow to the anterior tibia. This is a classic mechanism for a posterior cruciate ligament injury, and external clues (the scars) should not be overlooked when examining the knee. Occasionally, a posterior cruciate ligament injury is overlooked; however, putting together the patient's history, the examination (especially the posterior drawer and quadriceps active tests) provide a reliable diagnosis. Additional pathology should also be ruled out, such as a posterolateral corner injury and intra-articular pathology. Patella fracture, tibial tubercle avulsion, and patella tendon tears are unlikely because the patient can actively extend the knee. An anterior cruciate ligament tear is unlikely with a negative Lachman examination.

 

 

(SAE11AN.52) Figures 52a through 52c show the axial, coronal, and sagittal T2-weighted MRI scans respectively of a knee. The highlighted structure represents what anatomic finding? Review Topic

 

 

  1. Lateral meniscus tear

  2. Medial meniscus tear

  3. Anterior meniscofemoral ligament

  4. Ligamentum mucosum (infrapatellar plica)

  5. Transverse meniscal ligament

 

PREFERRED RESPONSE 5

 

The images show a transverse meniscal ligament, which connects the anterior horns of the medial and lateral menisci. On sagittal images, the interface of this structure with the anterior horn of the lateral meniscus often simulates a tear. Following this structure over several successive images is helpful in identifying it as a normal structure. There is no abnormal signal within the menisci to suggest a tear. A meniscofemoral ligament is a posterior structure. A ligamentum mucosum or infrapatellar plica is best seen on the sagittal image and runs from the intercondylar notch to the anterior fat pad.

 

(SAE13SM.2) A 19-year-old running back lands directly on his anterior knee after being tackled. He has mild anterior knee pain, a trace effusion, a 2+ posterior drawer, a grade 1+ stable Lachman, no valgus laxity, and negative dial tests at 30 degrees and 90 degrees. What is the best treatment strategy at this time? Review Topic

 

  1. Physical therapy with a focus on quadriceps strengthening

  2. Physical therapy and delayed posterior cruciate ligament (PCL) reconstruction

  3. PCL reconstruction

  4. PCL and posterolateral corner reconstruction

 

PREFERRED RESPONSE 1

 

This patient has likely sustained an isolated PCL injury. The examination is consistent with a grade II injury to the PCL. In this scenario, the best initial option is nonsurgical treatment and return to play as symptoms subside and strength improves. Physical therapy with a focus on quadriceps strengthening and delayed PCL reconstruction is not the answer because this patient can likely be treated without surgery. The absence of valgus laxity and negative dial testing findings suggest that an injury to the posteromedial and posterolateral corners has not occurred. Initial nonsurgical treatment is indicated for this patient. If he completes rehabilitation and experiences persistent disability with anterior and/or medial knee discomfort or senses the knee is "loose," PCL reconstruction should be considered at that time.

 

 

(SAE10SM.70) A high school football player asks you about an oral supplement that increases body mass and improves sprint times. He would like to use it to improve performance. What is the most likely agent? Review Topic

 

  1. Creatine

  2. Caffeine

  3. Testosterone

  4. Human growth hormone

  5. Ephedrine

 

PREFERRED RESPONSE 1

 

The supplement is creatine. Approximately 17% of high school athletes and about 30% of high school football players use creatine. Creatine is a protein synthesized in the liver and the kidney, circulates in the bloodstream, and is incorporated into muscle. Its use is associated with increased muscle mass, short-term improvement in sprinting, and may allow for increased anaerobic resistance performance. Caffeine and ephedrine are taken orally but do not increase muscle mass. Testosterone and human growth hormone are both associated with increased body mass but must be injected.

 

(SAE07SM.39) A 17-year-old basketball player and pole vaulter who has had anterior knee pain for the past 18 months now reports a recent inability to jump. Based on the MRI scan shown in Figure 11, management should consist of Review Topic

 

 

 

  1. debridement and repair.

  2. cast immobilization.

  3. aggressive overload eccentric strengthening.

  4. ice massage and continued athletic participation.

  5. steroid injection.

 

PREFERRED RESPONSE 1

 

The MRI scan reveals a partial patellar tendon rupture in conjunction with chronic patellar tendinitis. Mild and moderate patellar tendinitis may be treated nonsurgically with rest, stretching, strengthening, and anti-inflammatory drugs. Severe tendinopathy or extensor mechanism disruption is best treated surgically with tendon debridement and repair.

 

 

(SAE13PE.83) At the time of arthroscopy, a 9-year-old boy was found to have a Watanabe type II discoid lateral meniscus. What is the most appropriate treatment? Review Topic

 

  1. Saucerization of the meniscus only

  2. Saucerization and stabilization of the meniscus

  3. Stabilization of the meniscus only

  4. Complete menisectomy

PREFERRED RESPONSE 1

 

The Watanabe type II meniscus should only require saucerization for treatment because it is not unstable. The Watanabe classification defines 3 types of discoid mensici. In type I (stable, complete), the block-shaped lateral meniscus covers the entire lateral tibial plateau, whereas in type II (stable, partial), the lateral meniscus covers less than or equal to 80% of the tibial plateau. Type III discoid menisci (unstable, ligament of Wrisberg) appear to be normal except for a thickened posterior horn, but they lack posterior meniscal attachments, including the meniscotibial (ie, coronary) ligament. The type III discoid meniscus is stabilized only by the meniscofemoral ligament of Wrisberg. This results in hypermobility of the lateral meniscus at the posterior horn, which pulls into the intercondylar notch with knee extension, resulting in snapping knee syndrome. Complete menisectomy should be avoided if possible.

 

 

(OBQ14.36) Which of the following medications has been shown to improve skeletal muscle regeneration and decrease fibrosis following muscle injury in an animal model? Review Topic

 

  1. Aspirin

  2. Allopurinol

  3. Acetaminophen

  4. Losartan

  5. Bupropion

 

PREFERRED RESPONSE 4

 

Angiotensin II receptor blockade (e.g. losartan) administered after injury has been shown to improve muscle regeneration and decrease fibrosis in normal skeletal muscle.

 

Skeletal muscle undergoes a natural process of healing and regeneration after injury. The formation of fibrous tissue in place of normal muscle is also part of this process. However, fibrous tissue in place of muscle can predispose the area to re-injury and impaired function. Administration of angiotensin II receptor blockade medications (e.g. losartan) after skeletal muscle injury has been shown to decrease the apoptotic cascade response and the formation of fibrous tissue. The mechanism of benefit is thought to be associated with blockade of insulin-like growth factor.

 

Terada et al. looked at the affect of platelet-rich plasma (PRP) and losartan on muscle healing after contusion injuries. They showed that PRP plus losartan combination

 

therapy improved overall skeletal muscle healing by enhancing angiogenesis and follistatin expression as well as reducing the expression of phosphorylated Smad2/3 and the development of fibrosis.

 

Bedair et al. used a rat model to investigate the effect of angiotensin receptor blockade on muscle fibrosis after injury. They found that angiotensin receptor blockade therapy significantly reduced fibrosis and led to an increase in the number of regenerating myofibers in acutely injured skeletal muscle.

Incorrect

Answers:

Answers 1,2,3,5: None of these medications have shown to affect the muscle regeneration cascade.

 

 

 

(OBQ14.52) Following a partial muscle laceration, keeping the affected extremity immobilized for 2 weeks before starting an exercise program will likely lead to: Review Topic

 

  1. Greater area of fibrous scar tissue than immediate mobilization

  2. Less fatty infiltration than immediate mobilization

  3. Atrophy limiting final functional outcome

  4. Better penetration of ingrowing muscle fibers than a shorter period of immobilization

  5. More uniform orientation of ingrowing muscle fibers than a shorter period of immobilization

 

PREFERRED RESPONSE 3

 

Muscle lacerations immobilized for 2 weeks would likely experience significant muscle atrophy affecting final functional outcome. For muscle injuries, including partial lacerations, current literature suggests the affected extremity should be immobilized no more than 3-5 days, followed by a progressive strengthening and stretching program.

 

Healing a muscle injury involves two competing processes: regeneration of muscle fibers and formation of granulation tissue. Starting motion too soon after injury may increase the area of fibrous scar, and limit the ability of new muscle fibers to penetrate this area. Prolonged immobilization limits scar production but penetrating muscle fibers will lack appropriate orientation, and the muscle as whole begins to atrophy. Three to 5 days of immobilization has been shown to limit scar tissue production

 

 

while the early motion helps generate appropriately organized muscle fibers and maintain strength and range of motion.

 

Järvinen et al. present a review of muscle injury and healing. They found that immediate mobilization led to a large area of dense scar tissue that regenerating muscle fibers could not adequately penetrate. Prolonged immobilization allowed muscle fibers to regenerate but they lacked appropriate morphology. Following 3-5 days of immobilization they found less scar tissue, better penetration of regenerated muscle fibers, and the regenerated fibers were more well aligned with the uninjured fibers surrounding them.

 

Menetrey et al. present a mouse model of muscle laceration comparing a short period of immobilization (5 days) to suture repair of the muscle. They found quicker healing and greater strength in the suture repair group. Tetanus strength at one month after injury was (compared to an uninjured control) 81% for sutured muscle, 35% for the early mobilization only group, and 18% for the prolonged immobilization group.

 

Illustration A is a histology section of lacerated muscle after 7 days of immobilization. The area is infiltrated with granulation tissue and few regenerating myofibers. Illustration B is a histology section of lacerated muscle after 14 days of immobilization. Further infiltration with granulation tissue and mononuclear cells can be seen with regenerating myofibers only at the periphery.

 

Incorrect answers:

Answer 1: Prolonged immobilization leads to less scar tissue formation. Answer 2: Prolonged immobilization is associated with greater atrophy and fatty infiltration.

Answer 4: Prolonged immobilization shows less regenerative growth of muscle fibers. Answer 5: Prolonged immobilization produces regenerative mucle fibers with a more disorganized orientation.

 

(OBQ14.129) A 14-year-old soccer player injures his right hip during a game. He reportedly missed the ball and struck his foot against the ground, with an immediate "pop" and worsening pain in the right hip. A radiograph of his pelvis is shown in Figure A. This injury most commonly results from eccentric contraction of which muscle labelled in Figure B? Review Topic

 

 

  1. A

  2. B

  3. C

  4. D

  5. E

 

 

PREFERRED RESPONSE 3

 

The patient has sustained an anterior inferior iliac spine (AIIS) apophyseal avulsion injury. The rectus femoris muscle (C) inserts at the AIIS and eccentric contraction of this muscle can cause an avulsion injury.

 

Anterior inferior iliac spine (AIIS) apophyseal avulsion injuries are most commonly seen in adolescent male athletes. Patients often present with hip pain and hip flexion weakness. Treatment is conservative and includes rest, ice and protected weight bearing.

 

Rossi et al performed a retrospective case review of 198 adolescent competitive athletes with avulsion fractures of the pelvic apophyses. They concluded that these injuries are most common in soccer and gymnastics, and are usually the consequence of sudden and forceful muscle-tendon contraction during sport activities. The ischial tuberosity was involved in approximately 50% of the cases followed by the AIIS (25%), ASIS (20%), superior corner of pubic symphysis (4%) and the iliac crest (1%).

 

Reina et al presented a case report of twin brothers who each sustained an AIIS avulsion fracture 1 year apart via the same soccer-related injury. The authors conclude that AIIS avulsion fractures are likely multifactorial, with factors including genetics, age, maturation and sports all playing a role.

 

Figure A shows an AIIS avulsion injury. Figure B is a T1 axial MR image of the thigh. Illustration A is a T1 axial MR image of the thigh with labelled structures.

 

Incorrect

 

 

 

 

 

 

Responses:

Answer

1:

A

 

=

Vastus

lateralis

m.

Answer

2:

B

=

 

Vastus

intermedius

m.

Answer

4:

D

 

=

Vastus

medialis

m.

Answer 5: E = Sartorius m.

(SAE07SM.22) Which of the following complications is more likely with an inside-out repair technique compared to an all-inside techniques for a medial meniscus tear? Review Topic

 

  1. Failure

  2. Intra-articular synovitis

  3. Peroneal nerve injury

  4. Saphenous nerve injury

  5. Arthrofibrosis

 

PREFERRED RESPONSE 4

 

All of the answers are possible complications of meniscal repair. There are large volumes of literature evaluating the results of meniscal repair, both for the all-inside technique, as well as the inside-out technique. Failure rates are similar. Intra-articular synovitis occurs with absorbable sutures and absorbable implants. Peroneal nerve injuries are more common with the lateral-sided repairs. Saphenous nerve injuries are more common with medial-sided tears. Because of the incision required and the technique of tying over soft tissue, the risk of a saphenous nerve injury is greater with an inside-out technique than with an all-inside technique.

 

 

(SAE08AN.82) A 28-year-old man has left knee pain after a snow skiing accident. The MRI scan shown in Figure 47 reveals which of the following? Review Topic

 

 

 

  1. Osteosarcoma

  2. Bucket-handle medial meniscal tear

  3. Lateral collateral ligament tear

  4. Bone bruise

  5. Tibial spine avulsion

PREFERRED RESPONSE 4

 

Bone bruises are often noted on MRI after anterior cruciate and medial collateral ligament injuries. The significance of these injuries awaits long-term follow-up studies. The areas of increased signal on T2-weighted images represent areas of acute hemorrhage and are secondary to microfractures of the adjacent medullary trabeculae.

 

 

(SAE13SM.36) A 49-year-old man is seeking a second opinion for continued knee pain and swelling. He went to his primary doctor for swelling "on top of his knee," and he says his doctor drained some clear fluid. He noted that his condition improved for about 1 week before the swelling returned. He now has increasing pain and redness around his kneecap. Examination reveals significant swelling of his prepatellar bursa, with erythema over the bursa that extends to the surrounding skin. His temperature in the office is 101.7°F. What is the next step in treatment for this patient? Review Topic

 

  1. Initiate oral antibiotics for 7 days.

  2. Reaspirate the bursa and inject a corticosteroid.

  3. Recommend padding the patella for kneeling and ice.

  4. Perform an open bursectomy and start intravenous antibiotics.

 

PREFERRED RESPONSE 4

 

This patient has septic prepatellar bursitis. Padding, cold therapy, compression, nonsteroidal anti-inflammatory drugs, and aspiration are thought to be acceptable treatments for aseptic prepatellar bursitis; they have little role in septic bursitis. Antibiotics along with aspiration or placement of a percutaneous drain have been associated with success in some cases of septic prepatellar bursitis, but the standard treatment is complete bursectomy with systemic antibiotics.

 

(SAE10SM.16) A 17-year-old male football player is seen 1 week after developing symptoms of infectious mononucleosis in the middle of the season. Examination reveals evidence of splenomegaly. He and his parents want to know if he can play in a game the following day. What is the most appropriate recommendation? Review Topic

 

  1. It is safe to return to play right away.

  2. It is safe to return to play if there is no airway obstruction from adenopathy.

  3. It is safe to play due to the low risk of disease transmission to other players.

  4. It is safe to play after 3 to 4 weeks.

  5. It is unsafe to play the next season.

 

PREFERRED RESPONSE 4

 

Infectious mononucleosis (IMN) is a self-limiting viral (Epstein-Barr virus) infection that affects mostly adolescents. One of the clinical findings in IMN is splenomegaly. Unfortunately, the splenomegaly is palpable only 50% of the time. The risk for spontaneous splenic rupture is highest 3 weeks after the onset of symptoms. Thus, most clinicians recommend return to contact sports after 4 weeks from the onset of symptoms. This patient presented 1 week after the onset of symptoms, so he can return to play in 3-4 weeks from the time he was examined. The athlete should be afebrile, well hydrated, and asymptomatic. Airway obstruction is usually not of concern. Disease transmission to teammates is possible in the acute phases.

 

 

(OBQ15.188) Which of the following is not considered to be a part of the constellation of the clinical entities known as the female athlete triad? Review Topic

 

  1. Decreased bone mineral density

  2. Menstrual dysfunction

  3. Low energy availability with an eating disorder

  4. Low energy availability without an eating disorder

  5. Thyroid dysfunction

 

PREFERRED RESPONSE 5

 

Thyroid dysfunction is not one of the clinical entities included in the female athlete triad.

 

The female athlete triad was coined in 1992 by the American College of Sports Medicine as a complex disorder more prevalent in the adolescent and young female

 

 

athlete population including decreased bone mineral density (BMD), menstrual dysfunction, and low energy availability with or without a concomitant eating disorder. Treatment should involve a multidisciplinary approach, including psychological and nutritional counseling for eating behaviors and dietary management, reduction of training intensity to decrease risk of stress fractures, and initiating calcium and vitamin D supplements for osteoporosis.

 

Nazem et al performed a systematic review of articles containing the female athlete triad, reviewing diagnosis via screening during physical examinations as well as laboratory and imaging evaluation for menstrual dysfunction, low energy availability, and low bone mineral density. They state that potential complications including possible infertility, decreased immune function, cardiovascular disease, and irreversible loss of bone mineral density. They concluded that prevention, early recognition, and a multidisciplinary treatment team with a focus on proper nutrition and natural return of menses is vital.

 

Nattiv et al review the position of the American College of Sports Medicine regarding the female athlete triad, including screening for the triad at the pre-participation physical, discouragement of unhealthy weight loss practices. Essential members of the multidisciplinary treatment team include a health-care professional, a registered dietitian, and a mental health practitioner. They endorse that the first aim of treatment for any triad component is to increase energy availability by increasing energy intake and reducing exercise energy expenditure.

 

Answers 1-4: Decreased bone mineral density, menstrual dysfunction, and low energy availability with and without an eating disorder are the clinical entities included in the female athlete triad.

 

(OBQ14.231) Figures A and B are axial and coronal MRI images of a 21-year-old male athlete. He injured his left leg during a hurdling race approximately 1 week ago. What would be the next best step in the management of this injury? Review Topic

 

 

  1. Anti-inflammatory medication and non-weightbearing for 6 weeks

  2. Urgent CT scan

  3. Hip arthroscopy

  4. Open surgical repair

  5. Dynamic stretching and progressive mobilization

 

 

PREFERRED RESPONSE 4

 

Figures A and B show an acute proximal hamstring tendon avulsion. The next best step in management would be open surgical repair of all tendons to their origin at the ischial tuberosity.

 

Athletes participating in sports that require sprinting, jumping, acceleration and deceleration are at increased risk of sustaining a proximal hamstring tendon avulsion. The greatest predictor of this injury is prior hamstring injury. Other risk factors include increasing age, high training demand, increased body mass index and tight hip flexor muscles. MRI is the gold standard imaging to identify these images. Open hamstring tendon repair is recommended in athletes when all of the hamstring tendons have avulsed off their origin or 2 tendons have avulsed and retracted more than 2 cm.

 

Cohen et al. wrote a JAAOS article on acute proximal hamstring rupture. They point out that testing the peroneal branch of the sciatic nerve function is important in the physical examination, as injury to this branch will cause weakness of the short head of the biceps femoris and may slow potential postoperative rehabilitation.

 

Lefevre et al. reviewed the return to sports after surgical repair of acute proximal hamstring ruptures. They performed a prospective observational study that included 34 patients. Patients returned to sports within a mean 5.7 ± 1.6 months, at the same level in 27 patients (79.4 %) and at a lower level in 7 patients (20.6 %). They conclude that surgical repair of acute proximal hamstring ruptures has the potential to significantly improve the functional prognosis of patients with these injuries.

 

Figures A and B shows a significant amount of swelling and hematoma around the hamstring tendon. The whole ischial tuberosity is denuded of tendon, which is consistent with a complete rupture. Illustration A shows a large posterior thigh ecchymosis commonly seen with this injury. The ecchymosis presents approximately 1 week following injury, which is know as latent ecchymosis. Illustration B shows a

schematic and intraoperative image of the open tendon repair of an acute injury.

Incorrect

Answers:

Answers 1 and 5: Nonoperative is recommended in the case of single tendon rupture. Treatment consists of rest, ice, weight bearing as tolerated, NSAIDs, gentle stretching, therapeutic exercise, and gradual return to athletic activity, over approximately 4 to 6 weeks. This patient has ruptured his biceps femoris, semimembranosis, and semitendinosis.

Answer 2: There would be no indication for CT scan. MRI are more sensitive and specific for proximal hamstring ruptures. Answer 3: There is no intra-articular hip pathology. Some case reports have suggested that endoscopic hamstring repair can be used to treat these injuries, however these are low powered studies.

 

 

(SAE13SM.22) What strategy has proven most effective in preventing transmission of methicillin-resistant Staphylococcus aureus among teammates? Review Topic

 

  1. Separate players with infections in a separate locker room or changing area.

  2. Treat teammates of the infected player with prophylactic antibiotics.

  3. Cover any skin lesions with occlusive dressing during sporting activity.

  4. Ban players with infections from any team event.

 

PREFERRED RESPONSE 3

 

Prevention is the key to controlling infections among athletes. Proper hygiene is critical and should mandate showering, hand washing, wearing breathable clothing, and shower sandals. The sharing of towels or athletic equipment should be forbidden. Daily skin surveillance by athletes, trainers, and physicians can allow early recognition and treatment initiation during the early stages of infection, limiting risk for further transmission. Additionally, disinfecting shared equipment, covering lesions with occlusive dressing during sporting activity, and restricting the contact activities of infected athletes can limit risk for an infectious outbreak among teammates.

 

(SAE10SM.69) During a knee arthroscopy on a 38-year-old patient with isolated medial knee pain and no lateral symptoms, a routine examination of the lateral compartment reveals a discoid lateral meniscus. The discoid lateral meniscus is not torn. Based on these findings, what is the most appropriate action? Review Topic

 

  1. Complete lateral meniscectomy

  2. Lateral meniscal repair

  3. Saucerization of the lateral meniscus

  4. Microfracture of the lateral femoral condyle

  5. Do nothing surgically to the lateral meniscus

 

PREFERRED RESPONSE 5

 

The most appropriate action is to note this finding in the surgical report but do nothing surgically in the lateral compartment. Multiple studies have shown that asymptomatic discoid lateral menisci seen on routine knee arthroscopies for other pathology need not be addressed surgically. They do not cause problems later in life and do not need to be treated prophylactically.

 

 

(SAE08AN.8) Figure 6 shows a sagittal oblique MRI scan. The arrow is pointing to what structure? Review Topic

 

 

 

  1. Bucket-handle tear of the medial meniscus

  2. Ligament of Humphrey

  3. Ligament of Wrisberg

  4. Posterior intermeniscal ligament

  5. Partial tear of the posterior cruciate ligament

PREFERRED RESPONSE 2

 

The meniscofemoral ligaments connect the posterior horn of the lateral meniscus to the intercondylar wall of the medial femoral condyle. The ligament of Humphrey (arrow) passes anterior to the posterior cruciate ligament, whereas the ligament of Wrisberg passes posterior to the posterior cruciate ligament. One or the other has been identified in 71% to 100% of cadaver knees, with the ligament of Wrisberg being more common.

 

 

(SBQ07SM.46) A 28-year-old male presents with persistent knee symptoms 6 months following ACL reconstruction. Current radiographs are shown in Figure A. Based on the location of his femoral tunnel, which of the following physical exam findings is likely present? Review Topic

 

 

 

  1. Positive Lachman's exam

  2. Positive external rotation dial test at 30 degrees

  3. Positive anterior drawer sign

  4. Positive posterior drawer sign

  5. Positive pivot shift sign

 

PREFERRED RESPONSE 5

 

The clinical presentation is consistent with a ACL reconstruction with a malpositioned femoral tunnel at the "12 o'clock" position. ACL reconstruction with 12 o'clock femoral fixation would lead to a vertically placed graft and result in continued instability with cutting activities, and a positive pivot shift exam due to failure to reconstruct the posterolateral bundle of the ACL.

 

Current standards for anatomic ACL reconstruction stress the importance of more horizontal graft placement (10:30 in a right knee vs 1:30 in the left knee. This

 

attempts to reconstruct both the anteromedial bundle, which provides anterior-posterior stability, and the posterolateral bundle, which provides rotational stability. Improper femoral graft placement is one of the most common reasons for ACL revision surgery.

 

Scopp et al studied the effect of ACL femoral fixation in the coronal plane for patellar tendon autografts in a biomechanical model. In their study, they review that nonanatomic tunnel positioning of the femoral tunnel accounts for most of all technical failures and continued knee rotational instability. Their results supported that oblique femoral fixation as opposed to vertical femoral fixation of the ACL lead to more normal knee rotational stability.

Incorrect

Answers:

Answers 1 and 3: A positive Lachman or anterior drawer would not be present with a vertically placed femoral tunnel since the graft would still provide resistance to anterior-posterior translation . Answer 2: A positive external rotation dial test at 30 degrees would be present with a posterolateral corner injury. Answer 4: A positive posterior drawer sign would be present with a PCL injury.

 

 

(SAE11OS.85) During right knee anterior cruciate ligament (ACL) reconstruction, after drilling an appropriately positioned and referenced tibial tunnel, the surgeon finds that the transtibial guide is placing the femoral tunnel at 11:30 within the intercondylar notch. Which of the following choices will best enable appropriate graft placement in this clinical scenario? Review Topic

 

  1. Revise the tibial tunnel to be more oblique.

  2. Revise the tibial tunnel to be more posterior.

  3. Convert to a transtibial double-bundle ACL.

  4. Prepare the femoral tunnel via an anteromedial portal or two-incision technique.

  5. Hyperflex the knee and place the femoral tunnel with the transtibial guide.

 

PREFERRED RESPONSE 4

 

Anatomic placement of the femoral tunnel is best achieved in this clinical scenario by drilling the femoral tunnel through the anteromedial portal or via a two-incision technique. Several recent studies have demonstrated the difficulty that may be encountered in restoring true ACL anatomy on the femoral side when placing a femoral tunnel through a transtibial technique. While this is not always the case and this technique may be reasonable and sufficient, it is important for orthopaedic surgeons to critically assess tunnel placement intraoperatively and postoperatively to

 

minimize errant tunnel placement, demonstrated in the literature as the most common cause of ACL failure and need for revision. In this not uncommon clinical scenario, simply converting to a two-incision ACL technique or drilling through the anteromedial portal with the knee hyperflexed will permit accurate femoral tunnel placement and increase the likelihood of an optimal clinical outcome. Femoral tunnel accuracy with these techniques is enhanced by a lower starting point in the intercondylar notch. Familiarity with these techniques is valuable for surgeons performing ACL reconstruction. Revising the tibial tunnel in this scenario would likely lead to bone compromise of the proximal tibia and may interfere with graft fixation and incorporation. Converting to a double-bundle ACL with a transtibial technique would not correct the vertical femoral tunnel. Hyperflexion of the knee may improve femoral tunnel placement to some extent, but is unlikely to allow anatomic placement of a femoral tunnel when the transtibial guide lies in a clearly excessive vertical position.

 

 

(SAE13SM.62) Figure 62 is an arthroscopic view of the intercondylar notch of a right knee from an anterolateral portal. What is the main function of the structure delineated by the black asterisks? Review Topic

 

 

 

  1. Resist anterior translation during knee flexion

  2. Resist posterior translation during knee flexion

  3. Resist rotatory loads during knee flexion

  4. Resist rotatory loads during knee extension

 

PREFERRED RESPONSE 4

 

The structure shown is the posterolateral bundle of the anterior cruciate ligament (ACL). This bundle is optimally positioned in the knee to resist rotatory forces during terminal knee extension. "Resist anterior translation during knee flexion" best describes the anteromedial bundle. "Resist rotatory loads during knee flexion" is unlikely because the posterolateral bundle is tightest during knee extension. The posterior cruciate ligament, not the ACL, functions to resist posterior translation.

 

(SAE11OS.181) A 16-year-old boy has had knee pain for the past 6 months, and activity restrictions have not provided relief. An MRI scan reveals a stable 1.5 cm by 1 cm osteochondritis dissecans on the weight-bearing surface of the lateral femoral condyle. What is the best course of treatment? Review Topic

 

  1. Continued activity restrictions for 6 more months or until asymptomatic

  2. An aggressive physical therapy program that includes closed chain quadriceps strengthening

  3. Arthroscopic drilling of the subchondral bone

  4. Open debridement and screw fixation

  5. Osteochondral autograft transplant procedure

 

PREFERRED RESPONSE 3

 

As a child approaches skeletal maturity, osteochondritis dissecans lesions are unlikely to heal with continued nonsurgical management. Drilling of the lesion has a high success rate. The lesion is stable and an open repair or osteochondral transplant is not needed.

 

 

(OBQ14.81) A 14-year-old male soccer player was seen initially in the emergency room 1 week ago after an acute right hip injury during a soccer tournament. The patient reports that the hip pain has improved, but still requires crutches for long distance ambulation. His radiograph is seen in Figure A. What would be the next most appropriate step in management? Review Topic

 

 

 

  1. Admission for hip aspiration

  2. Admission for closed reduction and percutaneous pinning

  3. Non-weightbearing activities and crutches for 6 weeks total

  4. Weight-bearing as tolerated and crutches for 3 weeks total

  5. Urgent CT of the pelvis

 

PREFERRED RESPONSE 4

 

This patient has an avulsion fracture of the right anterior superior iliac spine. This injury should be treated with crutches and progressive weight-bearing, as tolerated.

 

The diagnosis of an avulsion fracture of the anterior superior iliac spine is made on the basis of: 1. History (sudden contraction of the sartorial and tensor fasciae latae muscle), 2. Physical findings (tenderness over the anterior superior iliac spine [ASIS] and pain with straight-leg raise), 3. Patient's age (most commonly in adolescents or young adults), and 4. Radiographs (confirmed fracture on standard views of the pelvis). Treatment of these injuries is almost always conservative with crutches and progressive weight-bearing activities as tolerated. The relative indications for operative treatment include displacement of the fracture fragment > 3 cm or painful non-union.

 

White et al. defined two types of anterior superior iliac spine avulsion fractures. A sartorius avulsion fracture (Type 1) usually occurs when sprinting. The fracture fragment is usually small and displaced anteriorly. The tensor avulsion fracture (Type 2) usually occurs when twisting the trunk (e.g. swinging a bat). This fragment is usually larger than Type 1 fractures and more likely to be displaced laterally.

 

Holden et al. reviewed pediatric pelvic fractures. They state that avulsion fractures of the anterior superior iliac spine are usually low-energy injures, and are not associated with other life-threatening injuries. They do not require an extensive workup (e.g. CT scanning)

 

Figure A is an antero-posterior view of the pelvis with a small right-sided avulsion fracture of the anterior superior iliac spine.

 

Incorrect Answers:

Answer 1: There is no indication for hip aspiration in an otherwise healthy patient with no infectious symptoms. Answer 2: Admission for closed reduction and percutaneous pinning would not be indicated in this patient. This patient is improving clinically and the fracture fragment is not significantly displaced. In addition, the operative modality of choice for this injury would be open reduction with internal fixation. Answer 3: This injury should not be treated with extended non-weightbearing activity. Answer 5: A multislice-CT is rarely necessary. A relative indication for CT scan would be to measure displacement of the avulsed fragment from the pelvis or to define the origin of the avulsed fragment from either the superior or the inferior iliac spine.

(SAE10SM.87) What is the most likely diagnosis based on the MRI findings shown in Figures 87a and 87b? Review Topic

 

 

 

  1. Anterior cruciate ligament (ACL) tear

  2. Posterior cruciate ligament (PCL) tear

  3. Lateral collateral (LCL) ligament tear

  4. Patellar dislocation

  5. Patellar tendon rupture

 

PREFERRED RESPONSE 4

 

The MRI scans reveal increased signal in the medial facet of the patella and the anterior aspect of the lateral femoral condyle. This pattern is typically seen in patients with acute patellar dislocations. In patients with ACL tears, the bone bruise of the lateral femoral condyle is usually seen in the central portion at the sulcus terminalis and the posterior half of the lateral tibial plateau and is not usually seen in the patella. This pattern of bone bruising is not seen with patellar tendon ruptures, LCL tears, and PCL tears.

 

 

(SAE11OS.114) Which of the following factors is considered to be the strongest predictor of outcome following arthroscopic partial meniscectomy? Review Topic

 

  1. Patient age

  2. Patient body mass index

  3. Amount of meniscal resection

  4. Location of the meniscal tear

  5. Modified Outerbridge cartilage score

PREFERRED RESPONSE 5

 

In a recent evidence-based review of the literature, the only consistent factor predicting outcome after arthroscopic partial meniscectomy was the extent of osteoarthritis as classified by the modified Outerbridge cartilage score at the time of surgery. All other factors listed (ie, location of meniscal tear, patient age, patient BMI, and amount of meniscal resection) were shown to not predict outcome following partial meniscectomy. While not provided as an answer choice, female gender was shown to be a predictor for slower recovery in the short term.

 

 

(SAE08AN.91) Figure 51 shows an arthroscopic view of the patellofemoral joint from an inferolateral portal. The arrow points to which of the following structures? Review Topic

 

 

 

  1. Loose body

  2. Plica

  3. Displaced meniscus tear

  4. Torn retinaculum

  5. Osteochondral defect

 

PREFERRED RESPONSE 2

 

Synovial folds or plicae are the result of incomplete or partial resorption of the synovial membranes during fetal development of the knee. The arthroscopic view shows a medial patellar plica, which has been noted in 5% to 55% of all individuals but becomes symptomatic in only a small number of patients. Symptoms may include crepitus, pain, snapping, and swelling and often respond to nonsurgical management.

 

(SAE13SM.64) What is an example of cognitive rest after concussion? Review Topic

 

  1. Playing chess

  2. Increasing reading

  3. Working online

  4. Limiting video games

 

PREFERRED RESPONSE 4

 

Physical and cognitive rest are recommended as treatment for sports-related concussion. Cognitive rest involves minimizing activities that require concentration and attention such as reading, schoolwork, video games, text messaging, working online, and playing games that require concentration such as chess. Limiting or eliminating video games after concussion is a form of cognitive rest.

 

 

(SAE13BS.56) What factor induces myofibrillar muscle protein synthesis (MPS)? Review Topic

 

  1. Aerobic exercise

  2. Anabolic hormones (growth hormone/testosterone)

  3. Resistance exercise above 60% 1-repetition maximum (RM)

  4. High-repetition exercise at mid intensity (30% 1-RM)

 

PREFERRED RESPONSE 3

 

Resistance exercise induces myofibrillar MPS that drives muscle hypertrophy. Growth hormone/testosterone does not influence MPS in acute response to exercise or adaptive response of muscle hypertrophy to resistance exercise. Recombinant growth hormone administration does not affect MPS. Anabolic steroids do not drive adaptation in humans; an intrinsic autocrine/paracrine factor and mechanotransduction process is involved. Exercise above 60% 1-RM represents anabolic ceiling. There is a sigmoidal dose response to resistance exercise, maximum MPS occurs at > 60% 1-RM. Aerobic-zone exercise does not result in hypertrophy of skeletal muscle, but it does increase oxidative capacity.

 

(SAE08PA.48) A 14-year-old girl reports a 3-week history of anterior thigh pain and a palpable mass after sustaining a soccer-related injury. Examination reveals a tender, firm mass in the midportion of the rectus femoris. MRI scans are shown in Figures 39a through 39c. What is the most appropriate management? Review Topic

 

 

 

  1. Incision and drainage of the abscess

  2. Nonsteroidal anti-inflammatory drugs, physical therapy, and a repeat MRI scan in 6 to 8 weeks

  3. Open biopsy

  4. Hematoma evacuation and musculotendinous repair

  5. Primary wide resection followed by radiation therapy

 

PREFERRED RESPONSE 2

 

The history, examination, and MRI scan findings are consistent with a midsubstance partial rupture of the rectus femoris muscle. This is an injury masquerading as a “pseudo tumor.” The lack of an appreciable mass effect on the T1-weighted MRI scan, the defined fluid signal on the T2-weighted scans, and the lack of significant contrast enhancement after gadolinium are all most consistent with injury rather than a neoplasm. Most of these injuries respond to nonsurgical management; a few will benefit from late debridement and repair if symptoms fail to resolve in 3 to 6 months. The treatment of choice is nonsurgical management with a follow-up MRI scan to verify that the findings are resolving.

 

 

(SAE08AN.56) Figure 33a shows a line drawing of a normal hemipelvis. The anterior acetabular rim is bold. Figure 33b illustrates a hemipelvis with a crossover sign, which is indicative of what acetabular pathology? Review Topic

 

 

  1. Low acetabular index

  2. Excessive acetabular retroversion

  3. Deficient anterior column bone

  4. Labral detachment

  5. Pelvic discontinuity

 

PREFERRED RESPONSE 2

 

In a normal AP pelvis radiograph, the anterior rim of the acetabulum runs medially and distally, diverging from the posterior rim which runs much more vertically. In excessive acetabular retroversion, the anterior rim (bold line in Figure 33b) and posterior rim start laterally, and as these lines progress medially and distally, the anterior line crosses the posterior line. This predisposes to femoral acetabular impingement.

 

 

(SAE13SM.51) What is the most important genetic element that distinguishes community-acquired methicillin-resistant Staphylococcus aureus (CA-MRSA) from hospital-acquired MRSA? Review Topic

 

  1. Beta-lactamase

  2. Penicillin-binding protein 2a

  3. Panton-Valentine leukocidin (PVL)

  4. Staphylococcus cassette chromosome (SCCmec) type I

 

PREFERRED RESPONSE 3

 

PVL is a cytotoxin that defines CA-MRSA and is not typical of hospital-acquired MRSA. PVL has the ability to lyse white blood cells and cause tissue necrosis, allowing for rapid progression of abscess formation. Beta-lactamase is an enzyme that breaks the beta-lactam bond of penicillin and is present in most strains of Staphylococcus aureus today. Synthetic penicillins such as methicillin are resistant to the effects of beta-lactamase. MRSA and CA-MRSA carry the mecA gene, which

 

encodes a penicillin-binding protein with a very low affinity for beta-lactam antibiotics, resulting in methicillin resistance. SCCmec mobile genetic units carry the mecA gene with additional genetic elements that together yield the multidrug-resistant strains found in healthcare environments. SCCmec type IV is specific to CA-MRSA and lacks these additional genetic elements, resulting in less multidrug resistance.

 

 

(OBQ13.265) A 26-year-old elite female swimmer underwent a left medial meniscal allograft transplantation. She returns to clinic 3 years later with knee pain. What is the most likely cause for late presenting knee pain in this patient population? Review Topic

 

  1. Late immune rejection of the meniscal graft

  2. Loss of graft fixation

  3. Osteoarthritis

  4. Graft tear due to acellularity

  5. Late-onset graft infection

 

 

PREFERRED RESPONSE 4

 

The most common long-term complication after meniscal transplantation is meniscal graft tear. Graft failure that results from graft tears is thought to be related to the acellularity of graft tissue.

 

Meniscal allograft transplantation is considered a salvage treatment option for young patients (<50 years old) with symptomatic meniscal deficiencies. The overall complication rate ranges from 4-36%, which include meniscal tearing, acute immune rejection, superficial and deep infection, chronic knee pain, etc. Graft tears make up

>50% of these complications.

 

Rath et al. evaluated 18 of 23 patients who had underwent meniscal allograft transplantation. They showed that 8 of 22 allograft menisci (36%) tore during the 8-year study period. They believe the decreased biologic activity of the graft over time may be a major factor that contributes to the high frequency of graft re-tearing.

 

Sekiya et al. reviewed meniscal allograft transplantation. They concluded that meniscal allograft transplantation may partially restore native meniscal function. Data has also shown that the progression of degenerative arthritic changes in transplanted meniscus-deficient knee compartments is slowed with this procedure.

 

Illustration A shows a basic schematic of meniscal transplantation. Here the graft is secured by anterior + posterior bone plugs and sutures to secure the transplanted meniscus into its native anatomical position.

Incorrect

Answers:

Answer 1: Late immune rejection of the meniscal graft has not been documented in literature. Graft rejection occurs most commonly in the acute phase, at an incidence of

<1%.

Answer 2: Graft incorporation into the surrounding tissue occurs 8-12 weeks after surgery. Loss of fixation would most commonly occur acutely, at an incidence of ~1-2%.

Answer 3: Meniscal transplantation has been shown to be protective against the progression of osteoarthritis in the knee. Answer 5: Chronic infections from cadaveric graft tissue are extremely rare, at an incidence of <1%.

 

 

(SAE10SM.79) What is the primary function of the structure at the tip of the probe in Figure 79? Review Topic

 

 

 

  1. Internal tibial rotation

  2. External tibial rotation

  3. Posterior tibial translation

  4. Anterior tibial translation

  5. Femoral internal rotation

 

PREFERRED RESPONSE 1

 

The structure shown in the figure is the popliteus tendon. This structure is a continuation of the popliteus muscle belly and attaches more proximally through its hiatus in the lateral meniscus onto the lateral femoral epicondyle anterior and distal to the insertion of the lateral collateral ligament. The popliteus is a dynamic internal rotator of the tibia. The popliteus complex reinforces the posterior third of the lateral capsule and plays a major role in the dynamic and static stabilization of the lateral tibia on the femur, including restriction of external tibial rotation, posterior tibial translation, and varus rotation of the tibia.

 

(SAE08AN.59) A 40-year-old man has had hip pain with increased activity over the past year. Examination reveals restriction of motion and tenderness with combined hip flexion, adduction, and internal rotation. An AP radiograph is shown in Figure 34. What is the most likely diagnosis? Review Topic

 

 

 

  1. Developmental dysplasia of the hip

  2. Osteonecrosis

  3. Perthes disease

  4. Pseudogout

  5. Femoral acetabular impingement

 

PREFERRED RESPONSE 5

 

Femoral acetabular impingement (FAI) is a pathologic entity leading to pain, reduced range of motion in flexion and internal rotation, and development of secondary arthritis of the hip. There are two types of FAI: cam impingement and pincher impingement. Cam impingement is seen when a nonspherical femoral head produces a cam effect when the prominent portion to the femoral head rotates into the joint. This mechanism produces shear forces that damage articular cartilage. Radiographs reveal early joint degeneration and flattening of the head neck junction (the so-called “pistol grip deformity”) as seen in this image. The pincher type of impingement involves abnormal contact between the femoral head neck junction and the acetabulum, in the presence of a spherical femoral head.

 

 

(SAE10SM.82) A 22-year-old male soccer player reports left hip and groin pain. He states that symptoms began before a preseason tournament but have worsened steadily for the past 2 weeks. He denies any recent fever or sickness and is otherwise healthy. Examination reveals tenderness over the symphysis pubis and pain with resisted rectus abdominus testing. Radiographs are negative. What is the next step in the proper management of this patient? Review Topic

  1. Rest, nonsteroidal anti-inflammatory drugs, rehabilitation, and gradual return to play

  2. Aspiration of the symphysis pubis followed by an appropriate course of antibiotics

  3. Referral to a general surgeon for hernia evaluation

  4. Rigid plating across the symphysis to address instability

  5. MRI evaluation of the symphysis

 

PREFERRED RESPONSE 1

 

Appropriate management of osteitis pubis includes rest, nonsteroidal anti-inflammatory drugs, directed rehabilitation, and gradual return to sports. Lack of fever or chills excludes osteomyelitis as a source of pain. Examination with tenderness over the symphysis pubis and pain with resisted rectus abdominus testing is consistent with osteitis pubis as opposed to a sports hernia, where a patient would be tender in the abdomen, not the pubis. There is no symphyseal instability that would require symphyseal plating.

 

 

(SAE13SM.76) What do the T2-weighted, fat-saturated MRI scans shown in Figures 76a through 76d reveal? Review Topic

 

 

 

 

 

  1. Posterior cruciate ligament (PCL) tear, isolated

  2. PCL tear and medial meniscus tear

  3. Anterior cruciate ligament (ACL) tear, isolated

  4. ACL tear and medial meniscus tear

 

PREFERRED RESPONSE 4

 

The MRI scans show that edema is noted on the femoral insertion of the ACL consistent with a high-grade or complete ACL tear. The ACL is not visualized on the sagittal view, although the torn meniscus can be seen in the notch. On the coronal image, there is an empty lateral wall sign indicating proximal disruption of the ACL. The medial meniscus images show a disruption of normal meniscus morphology consistent with a bucket handle medial meniscus tear. Note the appearance on the sagittal MRI scan of what appears to be a second soft-tissue density in line with the PCL. This "double PCL" sign is highly indicative of a displaced medial meniscus tear rather than a displaced lateral meniscus tear.

 

 

(SAE08OS.7) What type of medial collateral ligament tear heals the most reliably? Review Topic

 

  1. Proximal

  2. Midsubstance

  3. Distal

  4. Associated with an anterior cruciate ligament tear

  5. Associated with a posterior cruciate ligament tear

PREFERRED RESPONSE 1

 

Proximal medial collateral ligament (MCL) injuries adjacent to the medial epicondyle heal robustly. These proximal injuries are more prone to calcification, characterized clinically with temporarily increased pain and stiffness. The distal MCL, despite its long attachment site on the proximal tibia, heals less well. MCL injuries associated with other ligament injuries heal less reliably.

 

 

(SAE07SM.42) A 12-year-old boy reports the acute onset of pain and a pop over the right side of his pelvis while swinging a baseball bat during a Little League game. Radiographs reveal an avulsion of the anterior superior iliac spine with 2 cm of displacement. Management should consist of Review Topic

 

  1. open reduction and internal fixation of the fragment along with the rectus femoris.

  2. open reduction and internal fixation of the fragment along with the sartorius.

  3. open reduction and internal fixation of the fragment along with the iliopsoas.

  4. rest and protected weight bearing with crutches.

  5. excision of the fragment.

 

PREFERRED RESPONSE 4

 

Anterior superior iliac spine avulsion fractures are caused by sudden, forceful contractions of the sartorius and tensor fascia lata. These injuries occur in young athletes through the growth plate with the hip extended and the knee flexed, such as while sprinting or swinging a baseball bat. The athlete will often report a pop or snap at the time of injury. Displaced fractures usually can be seen on radiographs. CT or MRI can be obtained to confirm the diagnosis. In most patients, nonsurgical management consisting of rest and protected weight bearing yields satisfactory outcomes. Surgery is usually reserved for fractures with displacement of more than 3 cm and painful nonunions.

 

 

(SAE10SM.21) A 23-year-old woman has had a 3-year history of snapping and pain in her left hip. She notes that the snapping started while marathon training and is only problematic about 15 minutes into a run. Examination is consistent with a negative Stinchfield, negative logroll, negative flexion abduction/external rotation test (FABER) of the hip; however, she has a positive Ober test as she has difficulty adducting her hip across the midline in the lateral decubitus position. Management consisting of nonsteroidal anti-inflammatory drugs and stretching has failed to improve her snapping. What is the most reliable surgical treatment? Review Topic

  1. Hip arthroscopy with labral debridement

  2. Hip arthroscopy with femoral acetabular impingement lesion debridement

  3. Release of the iliopsoas tendon

  4. Z-plasty of the iliotibial band

  5. Release of the iliotibial band at Gerdy's tubercle

 

PREFERRED RESPONSE 4

 

The patient has external-type snapping hip (coxa saltans). It is not uncommon for patients to have a very long duration of symptoms that limit running or other sporting activities, and commonly affects the downward leg (usually the left leg when running on the left side of the road). The snapping causes a profound bursitis at the greater trochanter, and occasionally corticosteroid injections may be helpful. Her physical examination does not suggest an intra-articular process, and is not consistent with an internal-type snapping hip, usually caused by the iliopsoas tendon as it moves over the iliopectineal eminence. Stretching is the mainstay of treatment, as testing with a positive Ober signifies a tight iliotibial band as the thigh has difficulty crossing the midline with adduction. Various iliotibial band lengthening procedures have been described, including a Z-plasty near the proximal origin of the iliotibial band. Release at Gerdy's tubercle has not been described.

 

 

(SAE13SM.3) Figure 3 is the clinical photograph of a 20-year-old college soccer player who has a 7-day history of worsening left ankle pain and swelling after being slide-tackled in a game. Radiograph findings of his ankle and foot are normal. He complains of malaise. His history includes a severe ankle sprain 3 months ago. The sprain caused him to miss half the season, but he was able to play in the last 2 games. What is the most appropriate treatment? Review Topic

 

 

 

  1. Incision and drainage

  2. Ice the ankle but don't let him play.

  3. Topical antibiotics for 7 days with an occlusive dressing

  4. Debridement in the training room followed by 5 days of oral antibiotics

PREFERRED RESPONSE 1

 

The clinical photograph shows a skin infection with an appearance consistent with methicillin-resistant Staphylococcus aureus. This infection should be clinically incised and allowed to drain and a course of antibiotics should follow. If this infection is not promptly treated with debridement, it likely will worsen and potentially spread to other teammates. Antibiotics are secondary to surgical debridement but are a necessary adjunct. Although this patient has a history of severe sprain, his malaise and skin appearance do not correlate with a ligament injury or fracture. Debridement in the training room is not appropriate and would likely not fully decompress the fluid collection.

 

 

(SAE07HK.78) A 28-year-old woman who is an avid runner reports pain about the left hip with activities. Nonsurgical management has failed to provide relief. An MRI arthrogram is shown in Figure 47. What is the most likely diagnosis? Review Topic

 

 

 

  1. Osteonecrosis

  2. Transient osteoporosis

  3. Loose chondral fragment

  4. Labral tear

  5. Femoral neck stress fracture

 

PREFERRED RESPONSE 4

 

The MRI arthrogram reveals dye extravasation into the labrum, consistent with a labral tear. The MRI findings are not typical of osteonecrosis, stress fracture, or transient osteoporosis. There is no increase in bone marrow edema in the neck or femoral head.

 

(SAE08AN.88) What fibers of the anterior cruciate ligament tighten with extension of the knee? Review Topic

 

  1. Anterolateral

  2. Anteromedial

  3. Posterolateral

  4. Posteromedial

  5. Posterior oblique

 

PREFERRED RESPONSE 3

 

The anterior cruciate ligament consists of two functional bundles: anteromedial and posterolateral. During extension of the knee, the posterolateral bundle becomes taut. In flexion, the anteromedial bundle is tight and the posterolateral bundle relaxes. Traditionally, anterior cruciate ligament reconstruction primarily recreates the anteromedial bundle. Recently, techniques for double bundle reconstruction have been described to recreate the normal anatomic relationship of the two bundles.

 

 

(OBQ15.11) A 32-year-old recreational basketball player underwent a successful anterior cruciate ligament (ACL) reconstruction using hamstring autograft approximately 15 months ago. His chart notes that he has good functional knee outcome scores as tested by his physical therapist. However, he has not returned to play despite being cleared by his physician 3 months ago. After successful ACL reconstruction, which of the following factors has shown to contribute the greatest influence on a player's decision to return to sport? Review Topic

 

  1. Surgeon's advice not to return

  2. Persistent knee pain

  3. Duration of sport participation

  4. Lifestyle and psychological factors

  5. Autograft harvasting surgical technique

 

PREFERRED RESPONSE 4

 

Athletes who do not return to their preinjury level of sport after primary ACL reconstruction despite having good knee function are largely influenced by lifestyle and psychological factors.

 

The return to their preinjury level of sport is frequently expected within 1 year after anterior cruciate ligament (ACL) reconstruction, yet many athletes do not achieve this milestone. Having a previous ACL reconstruction to either knee, poorer hop-test symmetry and subjective knee function, and more negative psychological responses were associated with not returning to the preinjury level sport. Fear of reinjury is considered one of the most common reasons cited for a postoperative reduction in or

 

 

cessation of sports participation.

 

Tjong et al. conducted a qualitative study of 31 patients, aged 18 to 40 years, to understand the factors influencing a patient's decision to return to his or her preinjury level of sport after ACL reconstruction. They found 3 overarching factors what largely influenced their decision to return to the preinjury sport: fear, lifestyle changes, and innate personality traits. This highlighted the importance of recognizing and addressing the psychological factors and lifestyle changes that significantly contribute to a patient's postoperative decision to return to sport.

 

Ardern et al. investigated the return-to-sport rates at 2 years after ACL reconstruction in athletes. At 2 years after surgery, 66% were playing sport, with 41% playing their preinjury level of sport and 25% playing a lower level of sport. Demographics, physical function, and psychological factors were supported as the most important influencing factors for the return to their preinjury level of sport.

 

Incorrect Answers:

Answer 1: Surgeon advice not to return is considered a weak influence on a competitive athletes decision to return to sport. Answer 2: Persistent knee pain with good functional knee outcomes scores is considered a weak influence on a competitive athletes decision to return to sport. Answer 3: Return to sport across different types of sports has shown to be similar across studies. This is considered a weak influence on a competitive athletes decision to return to sport. Answer 5: The type of autograft used in primary ACL reconstruction has not shown to influence a players decision to return to sport.

 

(SAE07SM.84) Figure 23 shows the postoperative radiograph of a patient who underwent an anterior cruciate ligament (ACL) reconstruction (with bone-patella tendon-bone autograft) that failed. He initially had loss of flexion postoperatively. What is the most likely cause of this failure? Review Topic

 

 

 

  1. Fixation in the tibial tunnel

  2. Fixation in the femoral tunnel

  3. Posterior placement of the tibial tunnel

  4. Anterior placement of the femoral tunnel

  5. Size of the patellar autograft

 

PREFERRED RESPONSE 4

 

The key to this question is the fact that the patient initially lost flexion postoperatively and this relates to anterior placement of the femoral tunnel, thus capturing the knee. The bone plug seen on the radiograph is actually from the tibial tunnel, but this occurred as the patient forced flexion until failure of the ACL graft and pullout of the plug from the tunnel. Although it could be argued that better tibial fixation would have prevented this failure, poor placement of the femoral tunnel led to the failure of this ACL reconstruction.

 

 

(SAE10SM.44) A 13-year-old pitcher is hit in the left intercostal space by a line drive ball. He collapses, is apneic and unresponsive, and his radial pulse is absent. What is the next step in management? Review Topic

 

  1. Protect the airway and use smelling salts

  2. Protect the airway, move to the shade, and place in reverse Trendelenburg

  3. Protect the airway and protect from seizure activity

  4. Protect the airway, start CPR, and prepare to cardiovert

  5. Protect the airway, and move patient slowly to a spine board

 

PREFERRED RESPONSE 4

 

Sudden death in athletes without structural cardiac damage is referred to as commotio cordis. This is an emergency. The immediate priorities are protection of the airway, starting CPR, and early cardioversion as this patient has an arrhythmia. It is hypothesized to occur from apnea, vasovagal reflex, or ventricular arrhythemia as reported by Maron and associates from the direct impact of the baseball during a vulnerable part of the cardiac rhythm. Janda and associates reported that soft-core baseballs may not differ from standard baseballs with regard to the risk of fatal chest-impact injury while playing baseball. High survival rates are associated with rapid treatment.

 

(SAE08AN.85) Figure 49 shows an acute axial MRI scan of a right knee. What is the most likely diagnosis? Review Topic

 

 

 

  1. Patellar tendon rupture

  2. Lateral dislocation of the patella

  3. Quadriceps tendon rupture

  4. Anterior cruciate ligament rupture

  5. Posterior cruciate ligament rupture

 

PREFERRED RESPONSE 2

 

The MRI scan shows bone bruises in the medial aspect of the patella and the lateral aspect of the lateral femoral condyle. Both of these signs are typical for a lateral dislocation of the patella with spontaneous reduction. In addition, there may be associated tearing of the medial retinaculum or distal aspect of the vastus medialis.

 

 

(SAE10SM.90) Which of the following statements best describes labral tears in the hip? Review Topic

 

  1. They are unrelated to degenerative joint disease.

  2. They lead to increased movement of the femur relative to the acetabulum.

  3. They usually result from lesions of the ligamentum teres.

  4. They only occur with abnormal bone morphology.

  5. They commonly occur in the posteroinferior quadrant of the hip.

 

PREFERRED RESPONSE 2

 

Labral and chondral lesions are observed within the anterosuperior quadrant of the acetabulum. Tearing of the labrum markedly reduces resistance to joint motion, leading to instability. The most common associated lesions are chondral injuries. They can occur with or without abnormal bone morphology. The etiology for labral tears can be from traumatic and degenerative causes, structural abnormalities from femoroacetabular impingement, developmental abnormalities, and hip instability.

 

(OBQ13.205) A 26-year-old football player develops tachycardia and hot, dry skin during a game. He is found to have a temperature of 41 degrees C, but is not sweating. Further examination reveals the player is not oriented to time or place, and he soon develops convulsions. Which of the following is the most important next step in treatment? Review Topic

 

  1. Aggressive administration of IV fluids

  2. Administer acetaminophen

  3. Lay him supine with leg elevation

  4. Rapid cooling with ice immersion

  5. Administration of IV antibiotics

 

 

PREFERRED RESPONSE 4

 

The patient in the vignette has heat stroke; this condition is treated with rapid reduction in core body temperature through the use of ice immersion, cooling blankets, and/or internal cooling for a goal temperature below 39 C.

 

Heat stroke is a medical emergency with a high mortality rate. The hallmark features include central nervous system dysfunction and anhidrosis. Other symptoms include behavioral changes, such as confusion, disorientation, and staggering. Seizures and unconsciousness can also develop. The first modality of treatment is rapid reduction in temperature, which can be accomplished through ice water immersion, cooling blankets, or evaporative cooling methods including fans and cold water sprays. The goal in temperature reduction is 0.2 C per minute for a target temperature of 39 C.

 

Casa et al. reviewed current literature regarding the cause and care of exertional heat stroke. They stated that mortality from heat stroke remained significant, with the highest rates from sports existing between 2005 and 2009. They recommended accurate temperature assessment, prompt aggressive treatment using an efficient cooling modality (i.e. cold water or ice water immersion) prior to transport, and medically supervised return to play/duty as essential to preventing mortality.

 

Illustration A shows some of the visible differences between heat stroke and exhaustion, with the key discriminator being mental status changes present in heat stroke.

 

Incorrect Answers:

Answer 1: Administration of IV fluids would volume resuscitate the volume depleted patient, but rapid cooling is the most important next step. Answer 2: Acetaminophen is an antipyretic, which works to reduce fever through blockage of cyclooxygenase. This would not treat the underlying condition of the patient in this vignette. Answer 3: Lying supine with leg elevation and administration of fluids is the treatment for heat syncope, which is defined as a transient loss of consciousness with peripheral vasodilation, decreased cardiac output, and a normal temperature. Lying supine with leg elevation would help with redistribution of circulating volume centrally, thus increasing cardiac output and causing peripheral vasoconstriction (as a response by baroreceptors).

Answer 5: IV antibiotics could help if the elevated temperature was secondary to a systemic infection causing septic shock, but there is no evidence to suggest that the patient has an infection as the cause of his symptoms.

 

 

(SAE10SM.100) An 18-year-old boxer sustained a blow to his right eye in a boxing match. Examination on the sideline reveals hyphema, reduced visual acuity and color vision, and a visual field cut. What is the next step in management? Review Topic

 

  1. Eye patch and ophthalmology evaluation in 2 days

  2. Fluorescein eye stain

  3. Emergent CT

  4. High-dose systemic steroids

  5. Observation

 

PREFERRED RESPONSE 3

 

With the examination demonstrating reduced visual acuity and visual field changes, emergent CT is needed to look for traumatic optic neuropathy from direct or indirect trauma. The most common mechanism is blunt facial trauma (78%), but penetrating trauma is also common (22%). The most common etiologies are sports and motor vehicle accidents. Fluorescein eye stain would only be useful for corneal abrasion or corneal foreign body. CT scans are often helpful for an orbital fracture, optic nerve sheath hemorrhage, optical canal fractures, skull fractures, foreign bodies, nonorbital facial fractures, or associated brain injuries. The presence of a fracture of the optic canal on a CT scan was a poor prognostic sign in a recent series by Goldenberg and associates. The treatment is controversial. Although treatment options include high-dose corticosteroids, retrobulbar steroid injection, optic canal decompression, and optic sheath fenestration, there is no consensus as to the optimum treatment. It has been reported that treatment does not alter the prognosis in children and adolescents. Only 29% to 44% of children and adolescent patients had significant improvement in visual acuity. Hyphema is a collection of free blood in the anterior chamber of the eye. It is the most common intraocular eye injury associated with sports as reported by Denyi and associates, and occurs in 24% of injured eye cases. At the time of injury, it occurs as a haze in the anterior chamber. An eye patch and ophthalmology evaluation in 2 days is inappropriate because timely evaluation in this scenario is important. High-dose steroids are often used for this injury but not before a full evaluation including a CT scan. Observation is not appropriate because the injury needs an urgent evaluation.

 

(OBQ15.231) A 19-year-old football player is taken off the field because of fatigue. Examination reveals a rash shown in Figure A. Oral examination reveals findings shown in Figure B. Posterior cervical glands are palpable. A mass is palpable in the left upper quadrant. Which of the following is true regarding the most likely diagnosis? Review Topic

 

 

 

  1. There is a risk of splenic abscess

  2. Blood-borne transmission results in more severe symptoms

  3. He can return to play 3 weeks after he is asymptomatic

  4. Amoxicillin will allow faster return to play

  5. He should be isolated for 48 hours after symptom onset

 

 

PREFERRED RESPONSE 3

 

This patient has infectious mononucleosis (IM). Return to play should occur 3 weeks after symptom resolution.

 

IM is caused by the Epstein-Barr virus (EBV). Annual incidence is 1-3% in college freshmen. It is characterized by Hoagland's triad (fever, pharyngitis, lymphadenopathy). Some have rash and splenomegaly. Splenic rupture is rare (0.1-0.2% of patients). It is caused by sudden increase in portal venous pressure from a simple Valsalva maneuver or from external trauma. The risk of rupture is highest in the first 3 weeks of illness.

 

Putukian et al. reviewed IM and athletic participation. They recommend return to LIGHT activity after 3 weeks from symptom onset when the athlete is afebrile, has a good energy level, and does not have any significant associated abnormalities. They recommend returning to CONTACT sports after at least 3 weeks when the athlete has no remaining clinical symptoms, is afebrile, and has a normal energy level.

 

Jaworski et al. discussed infectious diseases in athletes. They state that splenic rupture occurs because of lymphocytic infiltration that distorts the support structure of the spleen, leading to fragility. They recommend return to light, non-contact activities once the athlete is afebrile and appropriately hydrated, fatigue has improved, and a minimum period of 3 weeks has passed from symptom onset.

 

Figure A shows a petechial rash, which can be seen in IM. Amoxicillin increases the risk of rash. Figure B shows unilateral exudative pharyngitis. The left tonsil is

covered

by

a

white

exudate/pseudomembrane.

Incorrect

Answers:

Answer 1: There is a risk of splenic rupture with trauma because of splenomegaly. Answer 2: EBV is carried in saliva and transmission is by kissing, sharing utensils, sneezing/coughing.

Answer 4: Amoxicillin/ampicillin is contraindicated because of the risk of rash. Answer 5: Isolation is unnecessary as transmissibility of EBV is low.

 

 

(SAE10PE.99) A 12-year-old girl is seen for left ankle pain. Radiographs reveal osteochondritis dissecans (OCD) involving the talus. What should the parents be told regarding management? Review Topic

 

  1. No treatment is required because spontaneous healing is common.

  2. Nonsurgical management typically relieves pain and results in radiographic healing in less than 12 weeks.

  3. Nonsurgical management frequently relieves pain but often may not result in radiographic healing even 6 months after treatment.

  4. Hyperbaric oxygen treatment is helpful.

  5. Ankle fusion is frequently necessary.

 

PREFERRED RESPONSE 3

 

Nonsurgical management of OCD of the talus in skeletally immature individuals frequently results in a fairly rapid decrease in symptoms, but radiographic abnormalities can frequently be found even 6 months after treatment. Spontaneous resolution of this condition is rare. Hyperbaric oxygen treatment has not been shown to be beneficial for this condition. Progression of the condition to the point of requiring ankle fusion is rare.

 

 

(SAE11OS.153) A 31-year-old high school football coach has right medial knee pain that is made worse with prolonged standing. His knee is minimally painful in the morning but by the end of the school day, he must sit down. The pain often makes sleeping difficult. He states that several years ago he underwent a surgical procedure to "clean out" the cartilage of the knee; however, he only had several months of pain relief. He is noted to be an athletic male (BMI of less than 30). Knee examination is unremarkable except for medial joint line pain that is exacerbated with standing and walking. Radiographs, including a long-leg view, and MRI scans are seen in Figures 153a through 153d. He wishes to remain active and asks whether he would be a candidate for allograft meniscus transplantation. You advise him that Review Topic

 

 

  1. the current literature does not support allograft meniscus transplantation.

  2. allograft meniscus transplantation is a surgical option; however, he is beyond the age where the procedure will provide much lasting benefit.

  3. you would recommend a course of viscosupplementation.

  4. based on his age and limb alignment, you would not recommend an allograft meniscus transplant but would recommend a high tibial osteotomy.

  5. based on his age and limb alignment, you would recommend a high tibial osteotomy and a staged allograft meniscal transplant after the osteotomy has healed.

 

PREFERRED RESPONSE 5

 

The patient's history, physical findings, and MRI scans indicate that a complete medial meniscectomy was performed. The meniscus provides an essential function in dissipating forces to the adjacent articular cartilage. Complete or partial meniscectomy has been shown to result in more rapid clinical and radiographic arthritis than if the meniscus is preserved. Allograft meniscal transplantation has been shown to be effective in the young patient with an absent meniscus, no or correctable limb malalignment, and minimal or correctable articular cartilage damage. His age would be appropriate for an allograft meniscus transplant. Based on the patient's long-leg radiograph, a valgus-producing high tibial osteotomy would be appropriate but alone would not address the absent meniscus in this young patient. Viscosupplementation may provide some temporary relief but is not an appropriate long-term solution. A staged valgus-producing osteotomy followed by an allograft meniscus transplant would be the most appropriate treatment.

 

(SAE08OS.177) Failure of posterolateral corner repair or reconstruction of the knee may be associated with which of the following? Review Topic

 

  1. Valgus malalignment

  2. Varus malalignment

  3. Anterior cruciate ligament reconstruction

  4. Posterior cruciate ligament reconstruction

  5. Medial collateral ligament reconstruction

 

PREFERRED RESPONSE 2

 

Failure to reconstruct a torn anterior cruciate ligament or posterior cruciate ligament at the time of posterolateral corner (PLC) repair or reconstruction dramatically increases the likelihood of PLC failure. Uncorrected varus malalignment places extensive tension on a PLC reconstruction and is a well recognized cause of failure. Valgus malalignment and medial collateral ligament reconstruction are not recognized means of failure.

 

 

(OBQ13.62) A 13-year-old gymnast presents with ongoing knee pain for the past few months. She tried conservative measures including kinesiotaping, physical therapy and rest. On physical exam, she has normal valgus alignment, negative patellar tilt and discomfort with resisted open chain knee extension. A representative radiographs are shown in Figure A-C. What is the most likely clinical diagnosis? Review Topic

 

 

 

  1. Patellofemoral syndrome

  2. Bipartite patella

  3. Stress fracture of the distal femur

  4. Patella alta

  5. Patella baja

PREFERRED RESPONSE 1

 

Based on history, physical examination and radiographic findings this patient has patellofemoral syndrome

 

Evaluation of a patient with patellofemoral pain requires a physical examination and plain radiographs. Appropriate examination of all structures around the knee is critical to rule out other diagnoses. An MRI is useful for evaluating intra-articular or intra-osseous lesions, if clinical suspicion is suggestive of this. Treatment is predominantly conservative, with focus on low impact exercises that maximize aerobic conditioning.

 

Earl et al. review the epidemiology, etiology and management of patellofemoral syndrome. They note that there is no clear cause of this issue, although issues related to the quadriceps and dynamic malalignment may be contributory.

 

Outerbridge et al. describe overuse injuries in the young athletic patient. They provide an overview of diagnosis and management specific to this patient population.

 

Figures A, B and C show AP, lateral and merchant radiographs of a normal knee in a skeletally immature individual. No osseous abnormalities are identified.

Incorrect

Answers:

Answers 2, 4, 5: There no radiographic findings to indicate the presence of a bipartite patella, patella alta or patella baja. Answer 3: Based on the radiographs, there is no evidence of stress fracture

 

 

(SAE10SM.39) Anaerobic weight training has what effect in a prepubescent 10-year-old male athlete? Review Topic

 

  1. It can induce muscle hypertrophy.

  2. It can increase efficiency of muscle action.

  3. It has no effect on muscle performance.

  4. It can cause injury to the growth plate.

  5. It can lead to a higher risk of osteochondritis dissecans.

 

PREFERRED RESPONSE 2

 

Although anaerobic weight training in this age group does not lead to muscle hypertrophy, it can increase the efficiency of muscle action by increasing muscle memory. There is insufficient testosterone in this patient population to allow for muscle hypertrophy. Proper techniques of weight training have been shown to be safe and do not damage the growth plates or joints in these individuals.

 

(SAE11AN.80) Figure 80a shows an arthroscopic view from an infralateral portal of a right knee. Figure 80b shows a coronal MRI scan, and Figures 80c through 80e show consecutive sagittal images of the knee. The images show what anatomic finding? Review Topic

 

 

 

  1. Loose body

  2. Discoid lateral meniscus

  3. Transverse meniscal ligament

  4. Displaced lateral meniscus tear

  5. Displaced medial meniscus tear

 

PREFERRED RESPONSE 2

 

The arthroscopic view and the coronal MRI scan show a discoid lateral meniscus covering almost the entire lateral tibial plateau. The sagittal views show a contiguous meniscus or "bow tie" sign on three consecutive images, pathognomonic for a discoid meniscus. Lateral discoid menisci are much more common than medial. There is no evidence of abnormal signal to indicate meniscal tearing. A transverse meniscal ligament is best seen anterior to the anterior horn of the lateral meniscus on multiple views. There is no evidence of a loose body on the arthroscopic or MRI images.

 

(SAE10SM.41) A 21-year-old female college athlete sustained a stress fracture of the fifth metatarsal 1 year ago which was treated successfully with surgical stabilization and she returned to normal activities. She now has a tension-sided femoral neck fracture. Along with surgical fixation of the fracture, what is the next step in management? Review Topic

 

  1. Obtain a menstrual history.

  2. Advise the athlete never to compete in high level endurance sports again.

  3. Obtain serum calcium levels.

  4. Obtain a psychiatric consultation.

  5. Recommend changes in training intensity.

 

PREFERRED RESPONSE 1

 

Stress fractures can be seen in female athletes who develop the female athletic triad including amenorrhea, osteoporosis, and eating disorders. Any female athlete with a history of stress fractures should undergo a workup for this disorder. Workup should include obtaining a menstrual history, obtaining a nutritional consultation, and obtaining a bone density. When properly counseled, these athletes may return to high endurance sports activities. Although these athletes may require a change in training intensity or psychiatric consultation, it would not be the next step in management. Psychiatric consultation may not be necessary unless an eating disorder has been diagnosed. Serum calcium levels are normal in these patients. Tension-sided stress fractures of the femoral neck require surgical stabilization with internal fixation as opposed to compression-sided stress fractures that can be treated with rest and nonsurgical management.

 

 

(SAE10BS.83) The skeletal muscle length-tension relationship for voluntary force generation is observed with which of the following biomechanical paradigms? Review Topic

 

  1. Isometric

  2. Isotonic

  3. Isokinetic

  4. Concentric

  5. Eccentric

PREFERRED RESPONSE 1

 

Muscle force during isometric contractions (muscle not allowed to shorten) varies with starting length. If the length is too long, the sarcomeres generate little or no active tension. If a muscle is overstretched during certain surgical procedures, then the patient may generate less than 30% maximal force. The force-velocity relationship describes isotonic (constant load) conditions.

 

 

(SAE10SM.43) A 21-year-old collegiate wrestler and rugby player reports an 8-month history of groin pain. Examination reveals a slight Trendlenburg gait, abductor weakness, hip flexion of 90 degrees, and internal rotation of 10 degrees. A radiograph and MRI arthrogram are shown in Figures 43a and 43b. What is the next most appropriate step in management? Review Topic

 

 

 

  1. Abduction orthosis

  2. DEXA scan

  3. Bone scan

  4. Hip osteotomy

  5. Hip arthroscopy and osteoplasty

 

PREFERRED RESPONSE 5

 

The patient has mixed cam and pincer-type femoroacetabular impingement. The radiograph shows an aspherical femoral head, and an acetabulum with increasd lateral coverage. The MRI arthrogram shows a degenerative labral tear and an aspherical femoral head. The triad of MRI-arthrographic findings consists of anterosuperior labral tears, anterosuperior cartilage lesions, and an increased alpha angle as reported by Kassarjian and associates. The pathoanatomy of cam-type impingement is characterized by a pistol-grip deformity of the femoral head-neck junction and a relative retroversion of the femoral head. As a result, there is convexity of this area

 

that causes abutment against the normal hip acetabulum with range of motion. Surgical treatment involves recontouring of the femoral head-neck. It can be performed arthroscopically or as an open procedure. The early results of these procedures are promising. However, Gerdeman and associates have pointed out that available evidence is very limited and long-term studies will be needed as to whether treatment halts the natural history of hip arthritis. In contrast, pincer-type impingement is characterized by retroversion of the acetabulum and abutment of the normal proximal part of the femur against the abnormal acetabulum. An abduction orthosis has no role in the treatment of femoroacetabular impingement. A DEXA scan is unlikely to provide any additional useful information. It is unlikely that a bone scan will add any additional information. A hip osteotomy is not indicated.

 

 

(OBQ09.157) A 27-year-old professional rugby player is sprinting down the field during a game and sustains a twisting injury to his right knee with immediate onset of swelling, pain, and difficulty with ambulation. Imaging of his right knee is demonstrated in Figures A, B, and C. Which of the following structures has most likely been injured? Review Topic

 

 

 

  1. Posterior cruciate ligament

  2. Anterior cruciate ligament

  3. Popliteus

  4. Lateral collateral ligament

  5. Medial collateral ligament

 

PREFERRED RESPONSE 2

 

The rugby player has sustained an injury to his anterior cruciate ligament (ACL), as demonstrated by the Segond fracture on radiograph and bone-bruising pattern on magnetic resonance imaging (MRI).

 

The ACL is the most commonly injured knee ligament, with an incidence of between

 

 

100,000 and 200,000 ruptures per year in the United States. The majority of ACL tears occur from non-contact athletic injuries. Acute ACL tears are associated with lateral meniscal tears. Physical examination findings include positive Lachman and Pivot shift tests. Imaging includes radiographs which are usually normal; however, the presence of a Segond fracture (avulsion fracture of the proximal lateral tibia) is pathognomonic for an ACL rupture and represents bony avulsion by the anterolateral ligament (ALL). MRI demonstrates bone bruising of the middle third of the lateral femoral condyle (sulcus terminalis) and posterior third of the lateral tibial plateau in more than 50% of acute ACL tears.. Bone bruise patterns correlate with the direction of the abnormal anterior translation and abutment of the posterolateral tibia against the middle third of the lateral femur during the injury. Treatment options include nonoperative management with physical therapy and lifestyle modification or operative intervention if failure of conservative measures.

 

Bathala et al present a radiologic case study of a Segond fracture visualized on AP radiograph of the knee of a patient who sustained an ACL rupture. They discuss the critical nature of this finding and the need for further imaging, as it is associated with ACL tears, meniscal tears, and damage to structures in the posterolateral corner.

 

Viskontas et al discuss different patterns of bone bruising demonstrated on MRI in acute anterior cruciate ligament ruptures of 100 patients, 86 with non-contact and 14 with contact injuries. They found that the non-contact mechanism correlates with more severe bone bruising in both the medial and lateral compartments as demonstrated on MRI.

 

Figures:

Figure A - AP radiograph of the knee that demonstrates a Segond fracture Figures B and C - coronal T1 and T2 MRI slices that demonstrate bone bruising of the middle third of the lateral femoral condyle and posterior third of the lateral tibial plateau

 

INCORRECT ANSWERS:

Answers 1, 3, 4, 5 - Segond fracture as well as bone bruising of the middle third of the lateral femoral condyle and posterior third of the lateral tibial plateau are not associated with injuries to the PCL, popliteus, LCL, or MCL.

 

(OBQ14.42) An elite football player has sustained a left knee injury during play. A dynamic imaging analysis is performed on the affected knee, which shows anterior shift and internal rotation of the tibia at low flexion angles. There is also some mild medial translation of the tibia at greater flexion angles. What structure(s) have most likely been injury? Review Topic

  1. Anterior cruciate ligament

  2. Posterior cruciate ligament

  3. Posterior cruciate ligament and medial collateral ligament

  4. Medial collateral ligament

  5. Lateral collateral ligament, popliteal tendon and arcuate ligament

 

 

PREFERRED RESPONSE 1

 

This patient has sustained an anterior cruciate ligament (ACL) rupture.

 

The ACL is the primary restraint to anterior translation of the tibia relative to the femur. It also acts as secondary restraint to tibial rotation and varus/valgus rotation. ACL-deficient knees have been shown to have abnormal knee kinematics, which has been thought to contribute to the osteoarthritis that develops after injury.

 

DeFrate et al. examined the knee joint kinematics of 8 patients with unilateral anterior cruciate ligament rupture using in vivo imaging. They found significant anterior shift and internal rotation of the tibia at low flexion angles in ACL-deficient knees. They also noted some medial translation of the tibia between 15° and 90° of flexion.

 

Illustration A shows the effect of medial tibial translation on tibiofemoral contact in ACL-deficient knees. The medial translation of the tibia causes increased contact between the tibial spine and inner surface of the medial femoral condyle. This might be a contributing factor to the joint degeneration observed in ACL-deficient patients.

 

Incorrect Answers:

Answer 2: Posterior cruciate ligament deficiency would cause a posterior shift of the tibia

Answer 3: Posterior cruciate ligament and medial collateral ligament deficiency would theoretically cause increased internal rotation and posterior translation of the tibia

Answer 4: Medial collateral ligament deficency would cause increased valgus rotation and slightly increased internal rotation. Answer 5: Lateral collateral ligament, popliteal tendon and arcuate ligament injuries collectively make up a posteriolateral corner injury. This would cause increased external rotation of the tibia.

 

(SAE10SM.49) A 38-year-old man is three quarters of the way through the Hawaiian Ironman events run in a temperature of 60 degrees F. He is sweating profusely and suddenly collapses. Prior to this he had been drinking large amounts of bottled water at every water stop. What is the most likely diagnosis? Review Topic

  1. Hypernatremia

  2. Hypothermia

  3. Hyponatremia

  4. Subendocardial myocardial infarction

  5. Ruptured berry aneurysm

 

PREFERRED RESPONSE 3

 

Hyponatremia is often seen in endurance athletes such as triathloners, ultramarathoners, and marathoners after prolonged exertion. It is commonly attributed to excess free water intake that fails to replete massive sodium losses that result from sweating as reported by O'Connor. Exercise-induced hyponatremia is generally asymptomatic, particularly in patients in whom the sodium is only mildy reduced. Up to 10% of ultradistance athletes have a sodium level of 135 mEq/L or less, but those who are symptomatic usually have a sodium level of 125 mEq/L as reported by Noakes and O'Connor. The best way to prevent hyponatremia is to maintain the proper volume and types of fluid intake to ensure fluid balance during exercise. Beverages containing carbohydrates in concentrations of 4% to 8% (ie, "sports drinks") are recommended for athletes participating in exercise lasting more than an hour (eg, marathon runners, etc.) To avert brainstem herniation and death, severe, acute hyponatremia requires rapid correction. Oral rehydration with salty solutions is safe and effective in patients with mild symptoms. Too rapid correction has been reported to cause central pontine myelinolysis; therefore, correction ought to be performed slowly. Hypernatremia, hypothermia, subendocardial myocardial infarction, or ruptured berry aneurysm are unlikely in this scenario.

 

 

(SAE10SM.24) Which of the following is the only nonreversible effect of anabolic steroids? Review Topic

 

  1. Muscle hypertrophy

  2. Alterations in high density lipoprotein (HDL) and low density lipoprotein (LDL) ratios

  3. Alopecia

  4. Personality effects

  5. Acne

 

PREFERRED RESPONSE 3

 

The loss of hair or alopecia, is the only nonreversible effect of anabolic steroid use. Once anabolic steroids are stopped, muscle hypertrophy and training gains are quickly lost and the HDL/LDL ratios return to their preexisting levels. Fortunately, the personality effects and the acute acne are reversible.

 

(OBQ14.112) All of the following conditions are associated with the female athlete triad EXCEPT? Review Topic

 

  1. Amenorrhea

  2. Osteoporosis

  3. Low LDL levels

  4. Decreased immune function

  5. Insufficient caloric intake

 

 

PREFERRED RESPONSE 3

 

All of the following listed are associated with the female athlete triad except for Low LDL cholesterol levels. In fact, these patients often have elevated levels of LDL due to the hypoestrogenism caused by menstrual dysfunction.

 

The female athlete triad is an interrelationship of menstrual dysfunction (i.e., amenorrhea or oligomenorrhea), low energy availability (insufficient caloric intake for demand, with or without an eating disorder) and decreased bone mineral density. It is relatively common among young women participating in sports. More recently, it has been suggested that endothelial dysfunction also results, due to an imbalance between vasodilating and vasoconstricting agents triggered from inappropirate levels of nitric oxide on the microscopic level, which predisposes these women to atherosclerotic changes and increases their risk of cardiovascular disease in the future.

 

Matheson et al. analyzed cases of 320 athletes with bone scan-positive stress fractures (M = 145, F = 175) seen over 3.5 years and assessed the results of conservative management. They found that conservative treatment of stress fractures in athletes is satisfactory in the majority of cases.

 

Constantini et al. evaluated the prevalence of vitamin D insufficiency and deficiency among young athletes and dancers. They found a higher rate of vitamin D insufficiency among participants who practice indoors, during the winter months, and in the presence of iron depletion.

 

Nazem et al. reviewed the major components and health consequences of the female athlete triad as well as strategies for diagnosis and treatment of the conditions. They concluded that treatment requires a multidisciplinary approach involving health care professionals as well as coaches and family members.

 

Yagi et al. followed 230 runners participating in high school running teams for a total of 3 years to report occurrence of medial tibial stress syndrome (MTSS) and stress fracture. Predictors of MTSS and stress fracture were investigated. The authors reported a significant relationship between BMI, internal hip rotation angle and MTSS infemales.

 

Incorrect Answers:

Answer 1, 2, 4 and 5: All of these conditions are associated with the female athlete triad.

(SAE10SM.85) Kinematic testing of patellofemoral motion demonstrates that malalignment that produces increased Q angle causes a shift of the patella laterally in the trochlear groove and is most pronounced during what phase of the flexion arc? Review Topic

 

  1. 0 to 15 degrees

  2. 20 to 30 degrees

  3. 40 to 90 degrees

  4. 100 to 120 degrees

  5. 130 to 140 degrees

 

PREFERRED RESPONSE 3

 

Dynamic patellofemoral joint contact measurements on cadaveric knees with simulated increased Q angle demonstrated that forces shifted to the lateral facet. The lateral shift in the patella was most pronounced from 40 to 90 degrees of flexion. At lower degrees of flexion, the lateral shift was significantly less. At higher degrees of flexion, the continued shift of the patella was not as pronounced.

 

 

(SAE10SM.75) A 20-year-old male military recruit reports a 5-day history of progressive deep groin pain that is made worse with weight-bearing activities and running. His initial coronal T2-weighted MRI scan is shown in Figure 75. His initial treatment should consist of which of the following? Review Topic

 

 

 

  1. Bed rest with skeletal traction (distal femur traction pin)

  2. Calcium supplements

  3. Crutches with protected weight bearing

  4. Open reduction and internal fixation

  5. Pulsed ultrasound treatment

PREFERRED RESPONSE 3

 

The MRI scan findings and patient history demonstrate a compression-sided femoral neck stress fracture. The stress fracture on the coronal MRI scan involves about one third of the width of the femoral neck. Surgical treatment would be recommended for tension-sided fractures of the femoral neck. The most appropriate initial treatment is protected weight bearing, with close examination and imaging follow-up. Skeletal traction is not currently used for nondisplaced femoral neck stress fractures, and there is no indication for open reduction and internal fixation. There is no documented role for pulsed ultrasound or calcium supplements in the acute treatment of stress fractures.

 

 

(SAE07HK.17) A 74-year-old woman has had acute medial right knee pain for the past 3 months. She denies any history of trauma or previous problems. Coronal and sagittal MRI scans are shown in Figures 11a and 11b. What is the most likely diagnosis? Review Topic

 

 

 

  1. Osteoarthritis

  2. Rheumatoid arthritis

  3. Medial meniscal tear

  4. Osteonecrosis

  5. Transient osteoporosis

 

PREFERRED RESPONSE 4

 

Spontaneous osteonecrosis of the medial femoral condyle is seen in the MRI scans, and is most common in women older than age 60 years. Although usually present in the weight-bearing portion of the medial femoral condyle, spontaneous osteonecrosis has also been described involving the lateral femoral condyle and patella. Most patients are seen postcollapse, and the treatment of choice is arthroplasty. Optimal treatment in precollapse stages is controversial.

 

(SAE08OS.175) Endurance training primarily involving aerobic activities results in which of the following muscular adaptations? Review Topic

 

  1. Hypertrophy of individual myofibers

  2. Increases in the muscle cross-sectional area

  3. Increase in the number of motor units recruited

  4. Increase in the capillary density of active muscles

  5. Changes in the activity of glycolytic enzymes

 

PREFERRED RESPONSE 4

 

Endurance training primarily involves aerobic activities and results in different muscular adaptations compared with resistance training. The most important adaptation is a significant increase in the capillary density of active muscles. The respiratory capacity of mitochondria is increased, resulting in greater maximal oxygen consumption; however, there are no significant changes in the activity of glycolytic enzymes. With resistance training, initial gains in strength observed are caused primarily by an increase in the number of motor units recruited to execute a particular movement. After approximately 4 to 6 weeks, individual myofibers begin to exhibit hypertrophy. Other changes that occur with muscular resistance training include an increase in the number of type IIA fibers with a concomitant decrease in type IIB fibers.

 

 

(OBQ14.159) A 17-year-old presents with persistent left knee pain after a twisting injury during a soccer match 24 hours ago. On physical exam he has a mild effusion. He has tenderness to palpation on the medial joint line. Lachman test, anterior drawer test and posterior drawer test are attempted but limited secondary to pain. Dial test reveals a side-to-side external rotation difference of roughly 5 degrees. His MRI images are seen in Figures A-D. These findings would be most consistent with: Review Topic

 

 

 

 

 

  1. ACL tear and medial meniscal tear

  2. Medial mensical tear only

  3. PCL tear and medial meniscal tear

  4. PLC tear and meniscal tear

  5. PCL tear only

 

 

PREFERRED RESPONSE 2

 

The patient has sustained a complex tear involving the posterior horn of the medial meniscus. Localizing joint line tenderness is the most sensitive physical examination finding for this injury.

 

Many provocative tests have been described to aid in the diagnosis of meniscus tears.

  • The Apley test is performed with the patient prone, by axially loading the tibiofemoral joint at 90° of knee flexion. Pain with compression and external rotation (medial meniscus) or internal rotation (lateral meniscus) is considered positive.

  • The Ege's test is performed by having the patient squat with their knees maximally externally rotated or internally rotated. The test is positive when pain and/or a click is felt by the patient.

  • The McMurray test is performed by passively moving the knee from flexion to extension while externally or internally rotating the leg. A palpable click at the joint line with external rotation (medial meniscus) or internal rotation (lateral meniscus) is considered positive.

  • The Thessaly test is performed by supporting the patient as they internally or externally rotate their knee and body while keeping the foot planted, with the knee in 5° of flexion and then 20° of flexion. The test is positive when joint line pain and/or sense of locking/catching is experienced by the patient.

 

Ryzewicz et al. performed a systematic review of prospective cohort studies comparing magnetic resonance imaging (MRI) and clinical examination to arthroscopy in the diagnosis of meniscus tears. The Apley test, Ege's test, McMurray test and Thessaly test at 5° were shown have high specificity but low sensitivity. Joint line tenderness has a higher sensitivity, but lower specificity. The Thessaly test at 20° demonstrated the highest sensitivity and specificity, although there was only one available study evaluating this test.

 

Abdon et al. performed a prospective study looking at 68 clinical parameters to

determine which combinations of symptoms and signs indicated the presence of a meniscus tear. They found that joint-line tenderness and mechanical locking were predictive of a meniscus tear, while the McMurray sign did not prove valuable. Pain at rest, sick leave and medial patellar tenderness all negated the presence of a meniscus tear. The clinical accuracy in diagnosing meniscal tears was 61% in this study.

 

Figures A and B are T1 sagittal MR images demonstrating an intact anterior cruciate ligament (ACL) and posterior cruciate ligament (PCL), respectively. Figures C and D are T2 coronal and sagittal MR images showing a complex tear of the posterior horn of the medial meniscus. Illustration A demonstrates the Ege's test. Illustration B shows the Thessaly test at 5° of flexion.

Incorrect

Answers:

Answer 1: This patient's MR demonstrates an intact ACL (Figure A) and therefore he does not have a concomitant ACL tear. Answer 3 and 5: This patient's MR demonstrates an intact PCL (Figure B) and therefore he does not have a concomitant or isolated PCL tear. Answer 4: This patient's dial test is negative and therefore he likely does not have a concomitant PLC injury.

 

 

(SAE10SM.80) Which of the following findings helps to distinguish between stress fractures of the tibia and shin splints? Review Topic

 

  1. With shin splints, a bone scan shows the posterior tibial cortex in a diffuse, longitudinal orientation.

  2. With tibial shin splints, the bone scan is more intense.

  3. A more diffuse area of tenderness is seen in tibial stress fractures.

  4. A three-phase bone scan is positive in all phases with shin splints, but only positive in delayed images with tibial stress fractures.

  5. After activity, pain persists longer with tibial stress fractures.

 

PREFERRED RESPONSE 1

 

Anterior tibial pain can often be difficult to diagnose. A bone scan showing the tibial cortex in a diffuse, longitudinal orientation is consistent with shin splints compared to a more discreet, localized uptake more commonly seen with a stress fracture. Bone stress injuries are due to cyclical overuse of the bone. They are relatively common in athletes and military recruits but are also seem in otherwise healthy people who have recently started new or intensive physical activity. Diagnosis of bone stress injuries is based on the patient's history of increased physical activity and on imaging findings.

 

The general symptom of a bone stress injury is stress-related pain. Bone stress injuries are difficult to diagnose based only on a clinical examination because the clinical symptoms may vary depending on the phase of the pathophysiological spectrum in the bone stress injury. Imaging studies are needed to ensure an early and exact diagnosis. If the diagnosis is made early, most bone stress injuries heal well without complications.

 

 

 

(SAE11OS.25) Performing reconstruction of the anterior cruciate ligament by drilling the femoral tunnel via an anteromedial portal, in contrast to transtibial drilling, affords what theoretical benefit? Review Topic

 

  1. Longer femoral tunnel

  2. More anatomic graft placement

  3. A more vertically oriented graft

  4. Diminished risk of posterior tunnel wall violation ("blowout")

  5. Diminished risk to lateral femoral articular cartilage and subchondral bone posteriorly

 

PREFERRED RESPONSE 2

 

Recent trends in anterior cruciate ligament reconstruction include an emphasis on anatomic rather than isometric reconstruction of the ligament. According to some studies, this more effectively restores knee kinematics and with this, rotatory stability. Transtibial drilling affords limited access to the lateral intercondylar wall and has been associated with vertical graft orientation. The anteromedial portal, in contrast, allows independent femoral tunnel drilling and more anatomic positioning of the graft. A more anatomically positioned tunnel established via an anteromedial portal may afford increased tunnel and graft obliquity. This has been suggested to resolve rotatory instability. Knee flexion angle during the course of reaming has been studied to assess favorable and negative tunnel characteristics and hazards to regional anatomic structures. When compared with transtibial drilling, the anteromedial portal is associated with shorter femoral tunnels, posterior tunnel wall integrity compromise, and increased risk to lateral femoral articular cartilage and subchondral bone posteriorly.

 

(SAE10SM.76) A college athlete has a knee injury requiring surgery. He has acne, gynecomastia, and well-developed muscles related to the use of anabolic steroids. What association with steroid use is concerning for surgery and anesthesia? Review Topic

 

  1. Fluid and electrolyte imbalance

  2. Increased bleeding time

  3. Impaired liver function

  4. Lowered oxygen requirements

  5. Splenomegaly

 

PREFERRED RESPONSE 1

 

Anabolic steroids increase procoagulant factors VII and IX and thromboxane, all of which lead to hypercoagulability which would decrease bleeding time. Liver function is usually upregulated as oral steroids induce hepatic enzymes and patients are therefore less sensitive to anesthetic agents. Anabolic steroids have a mineralocorticoid effect and users frequently use diuretics to mask this effect. Both can lead to fluid and electrolyte imbalances. Cardiovascular effects include hypertension, left ventricular hypertrophy, impaired diastolic filling, and thrombosis. Large muscle mass and high calorie intake lead to high ventilatory requirements caused by increased oxygen consumption and carbon dioxide production. Anabolic steroids have no effect on the spleen.

 

 

(SAE13SM.100) Figure 100 is the MRI scan of a 52-year-old runner who has right knee pain that has been occurring 10 minutes into her run for 2 months. On examination, she has tenderness over the lateral epicondyle. Her Ober test result is positive. What is the most appropriate initial treatment? Review Topic

 

 

 

  1. Iliotibial band bursectomy

  2. Z-lengthening of the iliotibial band

  3. Partial excision of the iliotibial band

  4. Home stretching program and cross training

 

PREFERRED RESPONSE 4

 

Iliotibial band syndrome (ITBS) is a common cause of lateral knee pain in runners. Potential etiologies for the pain include repetitive friction, compression, and bursal inflammation. An Ober test is used to assess iliotibial band tightness. With the patient lying on the unaffected side, the affected leg is abducted and extended. The test result is positive if the examiner is unable to adduct the leg from this position. An MRI scan can be helpful in making the diagnosis, but a negative MRI scan does not rule out ITBS. Studies have reported increased signal intensity on T2-weighted images deep to the iliotibial band adjacent to the lateral epicondyle, with thickening of the iliotibial band. Nonsurgical treatment is most appropriate initially and involves activity modification, ice, anti-inflammatory medications, and stretching. Corticosteroid injection to the iliotibial bursa is also an option to treat acute pain. After the initial inflammation improves, a strengthening program is started. Multiple surgical procedures have been described for recalcitrant cases, including iliotibial band excision, Z-lengthening, and iliotibial band bursectomy.

 

 

(SAE13SM.35) A 24-year-old former high school wrestler had anterior cruciate ligament (ACL) reconstruction with hamstring autograft 6 years ago. He now experiences daily instability of his knee with routine activities including walking. Examination reveals a grade 3+ Lachman with a soft endpoint, varus laxity at 30 degrees, and a positive dial test at 30 degrees that dissipates at 90 degrees of knee flexion. He has mild medial joint line tenderness. When walking, there is a slight varus thrust. What treatment is most likely to lead to a successful outcome? Review Topic

 

  1. Hamstring autograft

  2. Revision ACL reconstruction and posterior cruciate ligament (PCL) reconstruction

  3. Revision ACL reconstruction and posteromedial corner reconstruction

  4. Revision ACL reconstruction and posterolateral corner reconstruction

PREFERRED RESPONSE 4

 

This patient underwent an ACL reconstruction that has now failed. Based on his examination, he also has a posterolateral corner injury. Because this concomitant injury was not treated, the patient had undue strain on his graft, resulting in ultimate failure. Hamstring grafts are as effective as other graft types for ACL reconstruction. The medial meniscus provides secondary stabilization to the knee; however, this patient has a missed lateral ligamentous injury, and meniscus tears do not result in the development of a varus thrust. An unrecognized PCL tear likely results in mild-to-moderate medial and patellofemoral osteoarthritis without significant lateral laxity and thrust.

 

 

(SBQ12SP.35) A 16-year-old male football player is tackled into the ground during a game. He comes to the sideline with unilateral upper extremity pain, burning dysesthesias, and muscle weakness. Several minutes later, his symptoms resolve. In this scenario, what muscles are most likely to be affected? Review Topic

 

  1. Deltoid and biceps

  2. Triceps

  3. Wrist flexors

  4. Finger flexors

  5. Interossei

 

PREFERRED RESPONSE 1

 

This patient presents with a 'burner' or 'stinger,' a transient unilateral neurapraxia that most commonly involves the biceps, deltoid and rotator cuff muscles.

 

Hypotheses for the mechanism of neurapraxia in stingers include nerve root compression in the neural foramen as a result of extension and compression, traction on the brachial plexus or a direct blow to the plexus. C5-C6 is the most commonly involved level, indicating root injury or upper trunk brachial plexus injury. Unilateral and transient symptoms are required for diagnosis. Bilateral 'stinger' symptoms of numbness, tingling, and weakness are consistent with cervical spinal cord neurapraxia. Return to play in contact sports is contraindicated with bilateral symptoms prior to MRI evaluation of the cervical spine.

 

Shannon et al. review burner syndrome in athletes. They emphasize that criteria for return to play are based on a complete and thorough neurologic and physical exam. Normal strength and sensation in both upper extremities as well as a normal cervical spine exam (including range of motion, compression, Spurling's, Adson's, and resistive head pressures) are required for return to play. Any positive findings mandate withdrawal of the player from competition.

 

Illustration A shows hypothesized areas of involvement by mechanism according to

 

Shannon et al. A, Extension-ipsilateral compression. B, Flexion-contralateral flexion (traction) or direct trauma.

Incorrect Answer Answer Answer

2:

3:

4:

Wrist Finger

Triceps

flexors flexors

answers:

C7 C7 C8

Answer 5: Interossei C8-T1

 

 

(OBQ07.17) ACL rehabilitation often includes exercises with the distal end of the extremity fixed in place with axial loading and co-contracture of muscle groups to help stabilize the joint. This type of exercise is more commonly referred to as: Review Topic

 

  1. Eccentric contraction

  2. Closed chain exercise

  3. Open chain exercise

  4. Proprioceptive neuromuscular facilitation

  5. Isometric contraction

 

PREFERRED RESPONSE 2

 

A "closed chain exercise" is one in which the distal end of the extremity is fixed, allowing axial loading of the extremity with co-contracture of muscles decreasing stress across that joint.

 

Closed chain exercises are preferred for any rehabilitation protocol that wants to minimize stress across a potentially unstable joint. After ACL reconstruction it is important to begin motion and strengthening but not stress the reconstructed ligament which may lead to loss of stability or failure.

 

An example of a closed chain exercise of the quadriceps is a squat or leg press in which the foot is fixed against the floor/plate and both the quadriceps and hamstrings can contract together keeping the knee joint stable and preventing excess stress across the reconstructed ACL. An example of an open chain exercise of the quadriceps is a seated leg extension in which the foot is not fixed and the quadriceps contract in isolation. This creates a strong anterior pull on the tibia which can lead to excess stress on the ACL graft.

 

Beynnon et al. present a review on the behavior of ACL grafts during rehabilitation. They found that exercises that produce the least amount of stress across an ACL graft are either dominated by hamstring muscle contraction, involve quadriceps muscle activity with the knee flexed at 60° or greater, or involve active knee motion between

 

35°

and

90°

of

flexion.

Illustration A is an example of a squat, which is a closed chain exercise. Notice the feet are fixed in place against the floor. Illustration B is an example of a seated leg extension, which is an open chain exercise. Notice the feet are not fixed in place and no axial loading or co-contracture can occur.

Incorrect

Answers:

Answer A: Eccentric contractions are when a muscle is lengthening during the contraction.

Answer C: Open chain exercises involve the distal end of the extremity being freely mobile in space and a muscle group acts in isolation. Answer D: Proprioceptive neuromuscular facilitation (PNF) is the use of muscle stretch to augment contraction. Answer E: An isometric contraction is when the muscle length remains constant as it contract.

 

 

(SAE10SM.83) Closed chain kinetic exercises are differentiated from open chain exercises by which of the following? Review Topic

 

  1. Increased joint shear

  2. Maximally rehabilitate individual muscles

  3. Achieve normal motion in all the joints of the kinetic chain

  4. Maximize joint distraction

  5. The compressive nature of applied loads

 

PREFERRED RESPONSE 5

 

Closed chain kinetic exercises confer a margin of safety and are protective of healing or repaired tissues by the compressive nature of the applied forces. Closed chain kinetic exercise is associated with decreased shear, translation, and distraction of the joints within the chain. Because of patterns of motion with closed chain kinetic exercises, individual muscles may not be maximally strengthened or all joint motion returned to normal. Closed chain kinetic exercises may be used earlier in the rehabilitation process.

 

(SAE11AN.15) During establishment of an anterior portal for hip arthroscopy, what structure is at greatest risk for injury? Review Topic

 

  1. Lateral femoral cutaneous nerve

  2. Femoral nerve

  3. Femoral artery

  4. Superior gluteal nerve

  5. Sciatic nerve

 

PREFERRED RESPONSE 1

 

The anterior portal for hip arthroscopy is approximately 6 cm distal to the anterior superior iliac spine, penetrating the muscle belly of the sartorius and the rectus femoris before entering through the anterior capsule. The lateral femoral cutaneous nerve is divided into three or more branches at the level of this portal and may be injured during portal placement. The femoral nerve and artery are more medial and at less risk. The superior gluteal and sciatic nerves are posterior and not at risk with an anterior portal.

 

 

(OBQ13.275) A patient has persistent instability symptoms one year after ACL reconstruction. Radiographs and MRI show an intact graft with a femoral tunnel that enters the notch at the 12 o'clock position. These clinical findings have been associated with which of the following? Review Topic

 

  1. Lachman 2+, negative pivot shift and higher Lysholm scores

  2. Lachman 2+, positive pivot shift and no change in Lysholm scores

  3. Positive pivot shift and lower Lysholm scores

  4. Lachman 1+, negative pivot shift and lower Lysholm scores

  5. Lachman 1+, negative pivot shift and no change in Lysholm scores

 

PREFERRED RESPONSE 3

 

The clinical presentation is consistent with a mal-positioned femoral tunnel leading to a vertical graft. Vertical grafts are associated with persistently positive pivot shift and lower Lysholm satisfaction scores. The Lysholm is a commonly used brief subjective questionnaire.

 

Early arthroscopic single-incision transtibial ACL reconstruction often resulted in femoral tunnels which were at the top of the notch (12 o:clock position), rather than at the anatomic origin on the wall. The resulting vertical graft often improved anteroposterior laxity (as tested with the Lachman) but was less able to provide rotational stability (as tested with the Pivot shift).

 

Lee et al. found a significant association between vertically positioned grafts and residual (postoperative) positive pivot shift tests. They also found patients with a vertically positioned graft had lower Lysholm satisfaction scores. They conclude more oblique positioning of the graft may have advantages in rotational stability, which in turn increase subjective patient satisfaction.

 

Yasuda et al. noted that vertical non-anatomic tunnel placement cannot completely restore normal rotatory stability in laboratory studies.

Incorrect

Answers:

Answer 2. An intact vertical graft is less likely to have a positive Lachman with no endpoint.

Answer 3-5. An excessively vertical graft in a patient with persistent instability is likely to have a positive pivot shift.

 

 

(SAE13BS.96) A 19-year-old man sustains a thigh contusion over his quadriceps muscle while skateboarding. He is treated with several days of immobilization followed by gradually increasing range of motion and activity. Histologic analysis of a biopsy of the healing muscle in the mobilization state at 4 weeks from the date of injury would be characterized by Review Topic

 

  1. a decreased density of nerve fibrils.

  2. lymphocytes and macrophages infiltration.

  3. parallel penetration of muscle fiber through scar tissue.

  4. lengthening and narrowing of myotubes.

 

PREFERRED RESPONSE 3

 

Skeletal muscle contusions are a common cause of morbidity from sports-related injuries. The healing response involves a complex balance among muscle repair, regeneration, and scar-tissue formation. Animal models of muscle contusion have demonstrated a time-dependent histological response to postinjury protocols of immobilization and remobilization. Mobilization initiated after a brief period of immobilization resulted in better penetration of regenerative muscle through limited connective tissue scar in line with native surrounding muscle. Early mobilization in an in vivo muscle contusion model resulted in a progressive increase in myotubule, early nerve regeneration, and reduced inflammation. Immobilization results in disorganized penetration of muscle fiber through dense connective scar tissue with immobilization.

 

(SAE11AN.9) When performing hip arthroscopy, the hip should be placed in neutral to slight internal rotation to protect which of the following structures? Review Topic

 

  1. Femoral nerve

  2. Lateral femoral cutaneous nerve

  3. Ascending lateral femoral circumflex artery

  4. Ascending medial femoral circumflex artery

  5. Sciatic nerve

 

PREFERRED RESPONSE 5

 

The sciatic nerve is at greatest risk for injury during hip arthroscopy with placement of a posterolateral (posterior paratrochanteric portal). It can be within 3 cm of this portal. Advancing the trocar with the hip in neutral to slight internal rotation helps to protect the sciatic nerve from iatrogenic injury. The two structures in closest proximity with placement of arthroscopy portals are the lateral femoral cutaneous nerve (anterior portal) and the ascending branch of the lateral femoral circumflex artery (mid-anterior portal). The femoral nerve and medial femoral circumflex arteries are located medial to these anterior portals. Rotation of the hip has not been associated with increased risk of injury to any of these additional structures.

 

 

(OBQ14.23) Figure A shows intraoperative images of a right knee in an 8-year-old boy after he sustained an injury. Which of the following is the most common indication for performing this procedure? Review Topic

 

 

 

  1. Prevention of knee hyperextension

  2. Correct of valgus instability

  3. Correction of varus malalignment

  4. Treatment of symptomatic meniscal tear

  5. Treatment of an asymptomatic abnormally shaped meniscus

 

PREFERRED RESPONSE 4

 

This patient has undergone arthroscopic saucerization of his discoid meniscus. The indication for this procedure is treatment of a symptomatic meniscal tear.

 

Arthroscopic treatment of lateral discoid meniscus injuries has the advantages of reducing trauma, precise resection or repair of the meniscus and saucerization of the remaining discoid meniscus. Operative treatment is usually limited to patients with pain and mechanical symptoms that are undergoing partial meniscectomy or repair. Asymptomatic discoid meniscus without tears are not considered a surgical indication for routine saucerization.

 

Kramer et al. reviewed the diagnosis and treatment of traumatic discoid meniscal tears in children. They report that knee shape, size and skeletal maturity must be considered when determining the optimal method of repair. However, all symptomatic torn discoid menisci are best treated with saucerization and repair.

 

Good et al. looked at the arthroscopic techniques of discoid meniscus repair. Arthroscopic saucerization was successful in 28 of 30 knees and meniscal repair in 23 of 30 knees. At final follow-up, all patients exhibited full knee flexion beyond 135 degrees. In 3 of 30 patients there was residual knee pain, and four reported intermittent mechanical symptoms.

 

Figure A shows a series of arthroscopic images of the right knee lateral compartment. There is a sequential saucerization of the discoid meniscus.

 

Incorrect Answers:

Answer 1: A displaced torn discoid meniscus may block extension. Answer 2: Valgus instability is not an indication for saucerization of the discoid meniscus.

Answer 3: Saucerization does not prevent knee malalignment. Answer 5: An intact discoid meniscus seen on MRI is not an indication for saucerization.

 

(SBQ13PE.83) 8-year-old boy complains of intermittent painless clicking in his knee. His physical examination is normal. His family doctor orders an MRI, which reveals an incomplete lateral discoid meniscus without evidence of tear. What is the most appropriate treatment? Review Topic

  1. Observation only

  2. Diagnostic arthroscopy

  3. Saucerization of meniscus

  4. Saucerization of meniscus and microfracture

  5. Saucerization and stabilization of the mensicus

 

 

PREFERRED RESPONSE 1

 

The clinical presentation is consistent with a asymptomatic discoid meniscus. The most appropriate treatment at this time would be observation only.

 

MRI scans of the knee are very sensitive for identifying discoid menisci. Diagnosis is usually made when there are 3 or more 5mm sagittal MRI images showing meniscal continuity. Treatment is mostly focused on conservative modalities. Asymptomatic tears are usually treated with observation only. Saucerization is indicated for recurrent locking, swelling, persistent pain, or radiographic evidence of a meniscal tear.

 

Watanabe et al. described three types of discoid lateral menisci based on arthroscopic appearance. In this classification, discoid menisci with normal peripheral attachments are either type I (complete) or type II (incomplete). Type III discoid menisci, the so-called Wrisberg ligament type, are lacking posterior capsular attachments with the exception of the posterior meniscofemoral ligament.

 

Kramer et al. looked at the presentation of pediatric knee pain and discoid meniscus. They showed that the lateral meniscus is more commonly affected than the medial meniscus. The majority of discoid tears occur in the posterior or middle aspect of the meniscus.

 

Illustration A shows the cross-section of normal meniscus. Illustration B shows the Watanabe classification.

 

Incorrect Answers:

Answer 2: Diagnostic arthroscopy would not be indicated in the patient is asymptomatic and without evidence of a tear Answer 3: Saucerization of meniscus would be indicated with a Type II discoid if there was an associated tear or it was discovered incidently during knee arthroscopy. Answer 4: Saucerization of meniscus and microfracture may be considered if there was an associated tear, and an osteochondral lesion. Answer 5: Type III discoid meniscus would require stabilization along with saucerization.

(SAE10SM.28) An 18-year-old collegiate football player injures his right shoulder during a tackle. He reports pain and numbness in the shoulder and numbness radiating to his fingers. His symptoms improve within 15 minutes and he has no residual symptoms. This condition is best known as Review Topic

 

  1. acute and transient spinal cord injury.

  2. central cord syndrome.

  3. nerve root avulsion.

  4. Guillain-Barre syndrome.

  5. stinger/burner.

 

PREFERRED RESPONSE 5

 

The condition described in this case is known as a stinger or burner. It is caused by stretching the upper trunk of the brachial plexus in the C5 and C6 nerve roots. The symptoms are temporary and last 15 to 20 minutes. There are no residual deficits, unless the patient has had multiple repetitive injuries. Once motor and sensory examination findings and reflexes have normalized, the athlete can return to play. Acute spinal cord injury may cause temporary complete paralysis in the upper and lower extremities with resolution of symptoms within 24 hours. Central cord syndrome affects the upper more than lower extermities and affects mostly elderly patients. Nerve root avulsions lead to permanent deficits and have a poor prognosis for return of function. Guillain-Barre syndrome is an autoimmune disease that presents as an ascending paralysis with weakness in the legs that spreads to the upper limbs and the face along with complete loss of deep tendon reflexes.

 

 

(SAE07SM.8) When performing an inside-out lateral meniscal repair, capsule exposure is provided by developing the Review Topic

 

  1. iliotibial band and biceps tendon interval, then retracting the lateral head of the gastrocnemius anteriorly.

  2. iliotibial band and biceps tendon interval, then retracting the lateral head of the gastrocnemius posteriorly.

  3. iliotibial band and biceps tendon interval, then retracting the lateral collateral ligament posteriorly.

  4. iliotibial band and biceps tendon interval, then splitting the lateral head of the gastrocnemius.

  5. lateral head of the gastrocnemius and biceps tendon interval, then retracting the biceps tendon anteriorly.

PREFERRED RESPONSE 2

 

Capsular exposure for an inside-out lateral meniscal repair is performed by developing the interval between the iliotibial band and biceps tendon. Posterior retraction of the biceps tendon exposes the lateral head of the gastrocnemius. Posterior retraction of the gastrocnemius provides access to the posterolateral capsule.

 

 

(SAE07SM.45) What is the most common behavioral effect of anabolic steroid use in athletes? Review Topic

 

  1. Increased aggression

  2. Psychosis

  3. Drug dependence

  4. Depression

  5. Mania

 

PREFERRED RESPONSE 1

 

Users of anabolic steroids often display increased feelings of hostility and aggression. Although reports of psychotic, depressive, and manic behavior have been reported with the use of steroids, they are rare. Drug dependence, such as seen with narcotics, is not a feature of steroid use.

 

 

(SAE10BS.35) An active 23-year-old man has right groin pain that increases with sports activity. Examination reveals decreased internal rotation of the affected hip. He has a positive impingement test and radiographs reveal no crossover sign. An MRI scan is most likely to reveal which of the following? Review Topic

 

  1. Abnormal alpha angle and a chondrolabral tear

  2. Acetabular retroversion

  3. Heterotopic ossification

  4. Ankylosing spondylitis

  5. Coxa varum

PREFERRED RESPONSE 1

 

Young patients with hip pain and a positive impingement test are likely to have femoroacetabular impingement. The triad seen in these patients is a reduced concavity at the femoral head-neck junction, which leads to an increase in alpha angle and a chondrolabral tear. MR-arthrogram is the cross-sectional imaging modality of choice. These patients usually have reduced internal rotation and a positive impingement sign. The other findings, though possible, are not the most likely scenario in this young and active patient.

 

 

(OBQ15.154) Figure A is the lateral view of a left knee cadaveric specimen that has the extensor mechanism removed. Which two structures labeled provide the most rotational stability in knee flexion? Review Topic

 

 

 

  1. A + C

  2. B + C

  3. B + D

  4. C + D

  5. C + E

 

PREFERRED RESPONSE 4

 

The politeus tendon (C) and the popliteofibular ligament (D) are the lateral knee structures that provide the most rotational stability in knee flexion. The lateral collateral ligament (B) provides varus stability and rotational stability in knee

 

 

extension.

 

The lateral knee is stabilized by 28 unique static and dynamic structures. These structures work in concert to resist varus gapping and rotational knee instability. The three primary stabilizers include the fibular or lateral collateral ligament (LCL), the popliteus tendon (PT) and the popliteofibular ligament (PFL). Injury to these ligaments results in increased forces seen on ACL and PCL ligament reconstruction grafts with combined posterolateral corner (PLC) injuries, which can result in graft failure if the PLC is not reconstructed as well.

 

James et al reviewed the anatomy and biomechanics of the lateral knee stabilizers, detailing the specific functional contributions of the individual structures. The LCL is the primary restraint to varus instability, the highest forces occurring at knee extension (0°-30°). The PT and PFL serve as restraints to rotational instability, the highest forces occurring at knee flexion (60°-90°).

 

LaPrade et al reviewed the anatomy and biomechanics of the medial knee stabilizers. The three key static stabilizers are the superficial medial collateral ligament (sMCL), the deep medial collateral ligament (dMCL), and the posterior oblique ligament (POL). The sMCL is the primary restraint to valgus instability (proximal sMCL) and secondary restraint to rotational instability (distal sMCL) at all knee flexion angles. The dMCL is a secondary restraint to valgus and rotational instability. The POL is the primary restraint to rotational instability.

 

Figure A depicts the lateral knee structures in a cadaver specimen (A = lateral meniscus, B = LCL, C = PT, D = PFL, E = lateral head of gastrocnemius). Illustration A is a labeled picture of the lateral knee structures. Illustration B is a table from James et al demonstrating the change in forces experienced by the different lateral stabilizers at varying degrees of knee flexion. You see the LCL resists rotation at 0°-30° (extension) while the PT and PFL resist rotation at 60°-90° (flexion).

 

Incorrect Responses:

Answer 1: The lateral meniscus (A) is a secondary restraint to anterior translation and does not provide significant rotational stability. Answers 2 + 3: The LCL (B) provides minimal rotational stability in knee flexion, as compared to the PT and PFL. Answer 5: The lateral head of the gastrocnemius (E) does not provide significant rotational stability.

 

 

(SAE10SM.58) Which of the following is considered an advantage of the tibial inlay fixation compared to transtibial tunnel technique when used in posterior cruciate ligament reconstruction? Review Topic

  1. Less invasive

  2. Superior published clinical results

  3. Decreased surgical time

  4. Elimination of the critical 90-degree turn at the tibial aperture of the tunnel

  5. Improved cosmesis

 

PREFERRED RESPONSE 4

 

One of the most difficult aspects of posterior cruciate ligament reconstruction is placement of the tibial tunnel and passing of the graft through this tunnel. The tibial inlay technique requires a posteromedial approach to the tibia whereby the graft is directly fixed to the posterior aspect of the tibia. This obviates the need for a tibial tunnel. This technique has never been shown to be less invasive, more cosmetic, or require decreased surgical time. It has also never been shown in a published level I study to have superior clinical results. However, it does eliminate the need for the 90-degree critical "killer" turn and passing of the tibial graft through the tibial tunnel which may lead to graft failure.

 

 

(SAE07SM.15) A 21-year-old professional ballet dancer reports a painful popping sensation over her right hip joint. Examination reveals that symptoms are reproduced with hip flexion and external rotation. Which of the following studies will best confirm the diagnosis? Review Topic

 

  1. Radiographs

  2. Bone scan

  3. CT

  4. Ultrasonography

  5. MRI

 

PREFERRED RESPONSE 4

 

The patient has snapping hip syndrome of the internal type, which is more common in ballet dancers. It is caused by the iliopsoas tendon gliding over the iliopectineal line or the femoral head. The diagnosis usually can be made by the history and physical examination. Snapping is reproduced by hip flexion and extension or flexion with external rotation and abduction. Conventional and dynamic ultrasonography will confirm the snapping structure. Radiographs occasionally show calcifications near the lesser trochanter. MRI can be used to rule out other diagnoses that can simulate snapping hip.

 

(SAE10SM.30) In the anterior cruciate ligament-deficient knee, what structure provides an important secondary restraint to anterior tibial translation? Review Topic

 

  1. Anterior horn of the lateral meniscus

  2. Posterior cruciate ligament

  3. Posterior horn of the medial meniscus

  4. Popliteus tendon

  5. Quadriceps muscle

 

PREFERRED RESPONSE 3

 

Cadaveric studies have demonstrated the important role of the posterior horn of the medial meniscus in stabilizing the anterior cruciate ligament-deficient knee with significantly greater resultant force in the medial meniscus when subjected to anterior tibial loads. The posterior horn of the medial meniscus is thought to limit anterior tibial translation by acting as a buttress by wedging against the posterior aspect of the medial femoral condyle. The other soft tissues mentioned do not play any significant role in prevention of anterior tibial translation in the anterior cruciate ligament-deficient knee.

 

 

(SAE07SM.46) What is the effect on knee kinematics following placement of an anterior cruciate ligament (ACL) graft at the 12 o’clock position? Review Topic

 

  1. Decreased rotational stability

  2. Decreased anterior-posterior stability

  3. Decreased flexion

  4. Decreased extension

  5. Graft failure secondary to impingement

 

PREFERRED RESPONSE 1

 

Endoscopic ACL reconstructive techniques may result in a vertical graft placement. The reconstructed ligament will resist anterior translation of the tibia but the graft will not restore rotatory stability. Decreased flexion and extension are caused by placement of the femoral tunnel too anterior and posterior, respectively. Impingement of the graft on the femoral notch is caused by anterior placement of the tibial tunnel or inadequate notchplasty.

 

(OBQ14.146) An 18-year-old male presents with ongoing knee pain that limits his ability to participate in athletics. He was diagnosed with Osgood-Schlatter at the age of 13, and has symptoms since that time despite periods of rest, and use of NSAIDs, ice, and strapping for activities. A radiograph of his knee is seen in Figure A. Which of the following is TRUE regarding surgical treatment of his knee? Review Topic

 

 

 

  1. Surgery for Osgood-Schlatter is never indicated

  2. Prednisolone injection should be attempted prior to considering surgical intervention

  3. Surgical treatment must include hamstring lengthening for optimal outcome

  4. Ossicle excision and tubercleplasty will improve symptoms but return to sport is unlikely

  5. Ossicle excision and tubercleplasty will relieve symptoms and return to sport is expected

 

PREFERRED RESPONSE 5

 

Osgood Schlatter Disease (OSD) should be considered for surgical treatment if severe symptoms continue despite non-operative treatments and the patient is skeletally maturity. While not a commonly needed procedure, ossicle excision with tubercleplasty has been shown to provide relief of symptoms and return to full activities in most patients.

 

OSD is a traction apophysitis of the tibial tubercle, believed to be secondary to the repetitive pull of the patellar tendon. The mainstay of treatment for OSD is nonoperative treatment with rest, ice, anti-inflammatories, activity modification, and physical therapy. Symptoms typically resolve when the apophysis closes at skeletal maturity. Indications for surgery are persistent symptoms (pain) localized to the tibial tubercle that limit activities, have failed nonoperative treatment, and persist into skeletal maturity. Surgery usually consists of ossicle resection with or without tubercleplasty (resection of the tubercle prominence), and has been shown to be effective at relieving symptoms.

 

Pihlajamäki et al. reviewed 178 patients (all military recruits) who had surgically treated OSD for persistent symptoms. They performed ossicle excision and tubercleplasty. They report that at ten year follow-up 85% of patients reported no pain at the operative site, and 75% had returned to full preoperative activity level.

 

Mital et al. present a review of 16 knees that underwent ossicle resection and tubercleplasty for persistent OSD. They report only one patient did not return to their sport. They recommend combining the ossicle excision with tubercleplasty for best results.

 

Figure A is a skeletally mature patient with radiographic evidence of OSD. Illustration A is a skeletally immature patient with radiographic evidence of OSD.

Incorrect

Answers:

Answers 1: Osgood-Schlatter is treated non-operatively in most patients and resolves at skeletal maturity. However, refractory cases that have failed non-operative treatments and persist past skeletal maturity should be considered for surgical intervention.

Answer 2: Prednisolone injection is not used to treat Osgood-Schlatter. Answer 3: Hamstring lengthening is not indicated for treatment of Osgood Schlatter. Answer 4: Ossicle resection and tubercleplasty does provide relief of symptoms, and return to sport is expected for most patients.

 

 

(SAE10SM.33) A 20-year-old basketball player sustains a knee injury during a game and is seen in the orthopaedic clinic 3 days after injury. Examination reveals a positive Lachman, pivot shift, joint line tenderness, and moderate effusion. Which of the following tissue injuries is most likely causing the jointline tenderness? Review Topic

 

  1. Medial meniscus tear

  2. Popliteus tendon rupture

  3. Lateral meniscus tear

  4. Proximal tibia-fibula disruption

  5. Pes anserine bursitis

 

PREFERRED RESPONSE 3

 

The physical examination findings are consistent with an acute anterior cruciate ligament tear. In the acute setting, a lateral meniscus tear is a more common secondary injury than a medial meniscus tear. In one study of acute anterior cruciate ligament tears in alpine skiers, the incidence of lateral meniscus tears was over four times that of medial meniscus tears. Medial meniscus tears are more common in the chronic setting, most likely secondary to its role as a secondary restraint.

 

 

(SAE10SM.36) An 18-year-old woman injures her left knee playing soccer. At the time of anterior cruciate ligament (ACL) reconstruction, she was noted to have an irreparable posterior horn medial meniscus tear. Partial meniscectomy will have what primary effect? Review Topic

 

  1. Increase medial femoral-tibial peak contact loads

  2. Increase medial compartment contact area

  3. Decrease in situ forces in the ACL graft

  4. Decrease anterior tibial translation

  5. Increase posterior tibial translation

 

PREFERRED RESPONSE 1

 

The medial meniscus distributes force through the medial compartment. Peak loads in the affected compartment are increased by partial and complete meniscectomy. The posterior horn of the medial meniscus is also an important secondary restraint to anterior tibial translation in the ACL-deficient knee. In situ forces in the reconstructed ACL are increased with loss of the posterior horn of the medial meniscus.

 

 

(SAE11OS.176) Which of the following rehabilitation techniques is appropriate for initial nonsurgical management of an isolated grade 2 posterior cruciate ligament injury? Review Topic

 

  1. Immobilization in full extension for 4 weeks

  2. Immobilization in 30 degrees of flexion for 4 weeks

  3. Relative protection for 10 to 14 days, then range of motion with progressive plyometric exercises

  4. Relative protection for 10 to 14 days, then range of motion with gentle open-chain hamstring strengthening

  5. Relative protection for 10 to 14 days, then range of motion with gentle closed-chain quadriceps strengthening

PREFERRED RESPONSE 5

 

Treatment should consist of relative protection for 10 to 14 days followed by early range of motion and gentle closed-chain quadriceps strengthening. Isolated grade 1 and grade 2 posterior cruciate ligament injuries can be successfully managed nonsurgically. Progression to global knee strengthening can begin 4 to 6 weeks after the injury, with return to functional activity when full range of motion and strength is established. Plyometric exercises involve rapid alteration of contraction and loading of a muscle and should not be used in the early rehabilitation of a ligament injury of the knee because it risks further injury to the ligament. Hamstring strengthening should be avoided until the ligament has healed (4 to 6 weeks) because the posterior force on the tibia will stress the injured posterior cruciate ligament. Immobilization may be used for a short time to allow swelling and pain to subside, but early range of motion is preferred to avoid unnecessary stiffness following the stable injury.

 

 

(SAE07SM.100) Second impact syndrome (SIS) after head injury is characterized by which of the following? Review Topic

 

  1. Gradual progression of neurologic symptoms

  2. Preventable by restricting return to play until symptom-free

  3. Excellent prognosis for full recovery

  4. Rarely involves brain stem compromise

  5. CT rarely shows brain edema

 

PREFERRED RESPONSE 2

 

SIS is a devastating but preventable complication of head injury. It occurs when return to activities is allowed prior to complete resolution of the symptoms of the first head injury. A second, sometimes trivial, head injury can lead to a devastating series of events that can result in sudden death. The symptoms tend to progress rapidly and often involve the brain stem. The prognosis is poor.

 

(SAE13SM.4) A 23-year-old otherwise healthy 6-ft, 4-in basketball player complains of pain in his knees. An examination reveals localized tenderness to palpation over the inferior pole of the patella. The patient notes a significant exacerbation of his pain when the examiner takes the knee from flexion to extension. Review Topic

 

  1. Semimembranosis tendonitis

  2. Patellar tendonitis

  3. Iliotibial band friction syndrome

  4. Quadriceps tendonitis

 

PREFERRED RESPONSE 2

 

Patellar tendonitis is common in jumping sports such as basketball and volleyball. The pain is localized to the inferior border of the patella and is exacerbated by extension of the knee. Treatment for the vast majority of patients is nonsurgical and includes nonsteroidal anti-inflammatory drugs, physical therapy, and orthoses (patella tendon strap). Iliotibial band friction most commonly occurs in cyclists and runners (especially those who run up hills) and is a result of abrasion between the iliotibial band and the lateral femoral condyle. Localized tenderness with the knee flexed at 30 degrees is common. The Ober test may be helpful in making the diagnosis. Semimembranosis tendonitis most commonly occurs in male athletes during their fourth decade of life. The diagnosis is usually made with an MRI scan or nuclear imaging. Quadriceps tendonitis is similar to patellar tendonitis but is much less common. The pain may be associated with clicking and is localized to the superior border of the patella.

 

 

(SAE10SM.23) A collegiate division I football player ruptures his anterior cruciate ligament (ACL). After counseling him, you agree to perform a double-bundle ACL reconstruction. Which of the following is a correct statement for this technique? Review Topic

 

  1. The anteromedial (AM) bundle limits translation and the posterolateral (PL) bundle controls rotation.

  2. The PL bundle limits translation and the AM bundle controls rotation.

  3. The anterolateral (AL) bundle limits translation and the posteromedial (PM) bundle controls rotation.

  4. Both the AL and the PM control rotation equally.

  5. The AL bundle controls rotation and the PM bundle limits translation.

PREFERRED RESPONSE 1

 

The ACL is composed of two anatomic bundles: the anteromedial (AM) and the posterolateral (PL). They are both considered important to the stability of the knee. Although they work in concert, the AM bundle controls translation, especially in flexion, whereas the PL bundle prevents rotation.

 

 

(SAE08OS.22) Figures A and B show routine postoperative radiographs obtained 2 weeks after anterior cruciate ligament (ACL) reconstruction with autologous patellar tendon graft. Based on these findings, what is the next most appropriate action? Review Topic

 

 

 

  1. CT

  2. Routine ACL rehabilitation

  3. Modified ACL rehabilitation to limit weight bearing

  4. Modified ACL rehabilitation to limit flexion

  5. Revision ACL surgery

 

PREFERRED RESPONSE 5

 

The radiographs reveal an intra-articular position of the femoral bone plug; therefore, revision ACL surgery is indicated. Recognized early, this graft may be suitable to use for the revision, but an alternate should be available.

 

(SAE08AN.67) The posterior horn of the medial meniscus receives its primary blood supply from what artery? Review Topic

 

  1. Middle genicular

  2. Medial inferior genicular

  3. Medial superior genicular

  4. Lateral superior genicular

  5. Inferior lateral genicular

 

PREFERRED RESPONSE 1

 

The middle genicular artery supplies the posterior capsule and intracapsular structures (anterior cruciate ligament, posterior cruciate ligament, posterior horns of the meniscus). The medial and lateral inferior geniculates anastomose anteriorly to form a capillary network to supply the fat pad, synovial cavity, and patellar tendon. The lateral superior and inferior genicular arteries supply the lateral retinaculum.

 

 

(OBQ15.244) An 18-year-old male soccer player sustains a knee injury during a game. Examination is notable for a positive pivot shift test. What other physical examination finding is most likely to be present? Review Topic

 

  1. Medial joint line tenderness

  2. Lateral joint line tenderness

  3. Positive dial test at 30° of knee flexion

  4. Varus laxity at 30° of knee flexion

  5. Positive posterior drawer test

 

PREFERRED RESPONSE 2

 

The patient has sustained a tear of his anterior cruciate ligament (ACL), as demonstrated by the positive pivot shift test; therefore, he would most likely exhibit lateral joint line tenderness indicative of a lateral meniscus tear, the most common intraarticular injury associated with an ACL tear.

 

ACL tears usually occur as a result of a non-contact pivoting injury. Abnormal anterior translation results in bone contusions of mid-lateral femoral condyle and posterolateral tibia, which can be seen on MRI. Other concomitant intraarticular injuries include meniscal tears (lateral > medial), chondral damage and other ligamentous injury (MCL, LCL, PLC) usually found in cases of higher energy trauma such as a knee dislocation.

 

 

 

Piasecki et al prospectively analyzed intraarticular injuries associated with ACL tears in high school athletes by gender and sport. There was no significant difference in mechanism of injury between sexes. Female basketball and soccer players had fewer intraarticular injuries (medial femoral condyle lesions, medial and lateral meniscus tears) compared to male athletes. The authors hypothesized that women may therefore enjoy a better prognosis following reconstruction.

 

Spindler et al performed a prospective cohort study investigating concomitant intraarticular injuries in patients who underwent ACL reconstruction. Eighty percent of patients had a bone bruise on MRI, 68% involving the lateral condyle. At time of arthroscopic reconstruction, meniscal tears were identified in 56% of lateral menisci and 37% of medial menisci.

 

Incorrect Responses:

Answer 1: Medial joint line tenderness is suggestive of a medial meniscus tear, which is a common injury associated with ACL tears but is less common than a lateral meniscus tear in cases of acute ACL rupture. Medial meniscus tears are more common in chronic ACL tears. Answer 3: Positive dial test at 30° of knee flexion is suggestive of a posterolateral corner injury, which is less common than a lateral meniscus tear in patients with an ACL tear.

Answer 4: Varus laxity at 30° of knee flexion is suggestive of a lateral collateral ligament injury, which is less common than a lateral meniscus tear in patients with an ACL tear.

Answer 5: Positive posterior drawer test is suggestive of a posterior cruciate ligament injury, which is less common than a lateral meniscus tear in patients with an ACL tear.

 

(SAE07SM.73) A 20-year-old soccer player who collapsed after a goal kick reports weakness and nausea. He appears slightly confused. Examination reveals that he is not sweating. His skin is warm and dry. The outdoor temperature is 80 degrees F (26.6 degrees C) with a relative humidity of 80%. Management should consist of Review Topic

 

  1. a drink of water.

  2. a sports drink with electrolytes.

  3. placement in the reverse Trendelenburg position in a shaded area.

  4. immersion in a warm water bath.

  5. transportation to the emergency department.

PREFERRED RESPONSE 5

 

There is a spectrum of heat-related conditions. Heat cramps are the mildest form of heat illness. In heat exhaustion, cramps are associated with headache and weakness, and the skin is pale and moist. Treatment of heat cramps or heat exhaustion consists of removing and loosening excess clothing, applying ice to the axilla and groin, ingestion of cool water, and cool water sprays. This patient demonstrates symptoms of heat stroke which is a medical emergency. The core body temperature may be as high as 106 to 110 degrees F (41.1 to 43.3 degrees C). In heat stroke, the patient may no longer be sweating, and the skin may be hot and red. The athlete is usually confused, weak, nauseated, and may have seizure activity. Central nervous system depression has been called the most important marker of heat stroke, and progresses from confusion and bizarre behavior to collapse, delirium, and coma. Bizarre behavior is often the first sign of heat stroke. The patient needs to be treated and moved to a medical facility rapidly. During transfer, IV fluids and cooling of the athlete should be initiated. The best treatment of heat-related illness appears to be prevention with adequate hydration and monitoring of conditions (temperature and humidity), with cancellation of competition when conditions do not comply with guidelines.

 

 

(OBQ15.136) A radiologist's report of a lateral knee radiograph comments that the height of the patella is 4 cm, and the length of the patellar tendon is 3 cm. Which of the following may lead to this measurement? Review Topic

 

  1. Osteochondritis dissecans

  2. Quadriceps tendon rupture

  3. Inferior pole patellar sleeve avulsion

  4. MPFL disruption

  5. Nail-patella syndrome

 

PREFERRED RESPONSE 2

 

The question describes a knee with patella baja, which would likely be seen in the presence of a quadriceps tendon rupture.

 

The measurements described allow you measure an Insall-Salvati Index (ISI). The ISI is the ratio of the patellar tendon length to the patellar height (distance from the superior pole to the inferior pole). These measurements give a ratio of 0.75 which indicates the presence of patella baja. Normal values range from 0.8 to 1.2. A lower ratio is diagnostic of patella baja, while a higher ratio is diagnostic of patella alta. The ISI as well as some other measures of the patella's position within the trochlea are important when evaluating conditions such as patellar instability or patellofemoral

 

 

pain.

 

Insall and Salvati present their original paper describing the anatomy of the knee with respect to patellar positioning. They note that previous methods of determining patellar positioning was either too complicated, or too subjective. They found the length of the patellar tendon and the height of the patella to be approximately equal, and saw no variations greater than 20%. Thus they proposed the ISI with "normal" values ranging from 0.8 to 1.2.

 

Shabshin et al. present a study making similar calculations as the ISI but done on sagittal MRI rather than lateral radiographs. They found a slightly higher degree of variation, and defined "normal" as ranging from 0.79 to 1.52. They also noted females tended to have higher ratios on average compared to males.

 

Aglietti et al. compared various measurements of the patellofemoral joint in a group of asymptomatic knees compared to knees with recurrent patellar subluxation. The normal knees were found to have an ISI of about 1, average congruence angle of -8, and an average Q angle of 15°. The subluxating knees had higher ISIs (average 1.23), higher congruence angles (average +16), and a higher Q angles (average 20°).

 

Illustration A shows how to measure and calculate the ISI. Illustration B shows measurements of the patellofemoral joint that may help identify presence of trochlear dysplasia and patellar maltracking.

 

Incorrect Answers:

Answer 1: Osteochondritis dissecans does not lead to alterations of the ISI and thus would not cause patella baja. Answer 3: An inferior pole patellar sleeve avulsion would lead to patella alta and an ISI greater than 1.2.

Answer 4: A disruption of the medial patellofemoral ligament (MPFL) would lead to patellar instability and abnormalities of other measures, such as the patellar tilt. Answer 5: Nail-patella syndrome is a genetic disorder often associated with small or absent patellae. The measurements given are not diagnostic of this condition.

 

(SAE13SM.43) An otherwise healthy 15-year-old wrestler has a 6-cm cutaneous lesion on the posterior aspect of his right elbow that he reports as a spider bite. What is the most likely diagnosis? Review Topic

 

  1. Psoriasis

  2. Tinea corporis

  3. Herpes simplex virus

  4. Community-acquired methicillin-resistant Staphylococcus aureus (CA-MRSA)

PREFERRED RESPONSE 4

 

Patients who have skin and soft-tissue infections caused by CA-MRSA often describe the lesion as a spider bite. The cytotoxin Panton-Valentine leukocidin that is produced by many strains of CA-MRSA causes tissue necrosis, resulting in rapid development of an abscess and the appearance of a spider bite. Patients with psoriasis have thick, red skin with flaky, silver-white patches. Tinea corporis is a general term for a cutaneous fungal infection. The lesion appears as a well-demarcated erythematous plaque with a raised border and central hypopigmentation, giving it a ring-like appearance. Primary infection with herpes simplex virus can produce constitutional symptoms with burning, tingling, or stinging at the site. Grouped vesicles with clear fluid 1 to 2 mm in size form on an erythematous base and then rupture, leaving moist ulcers or crusted plaques.

 

 

(SBQ07SM.48) Carbohydrate loading is a common practice among endurance athletes that works by what mechanism? Review Topic

 

  1. Increasing serum fructose levels

  2. Maximizing stored muscle glycogen for endurance after 90 minutes of exercise

  3. Diuresis

  4. Decreasing serum glucose levels

  5. Maximizing stored liver glycogen for endurance after 60 minutes of exercise

 

PREFERRED RESPONSE 2

 

Carbohydrate loading is a strategy that involves changes to training and nutrition which can maximize muscle glycogen stores prior to endurance exercise lasting longer than 90 minutes.

 

There continues to be varying scientific opinions as to whether athletes should eat low vs. high glycemic index foods, and how this will affect the amount that the serum glucose levels rise. Fructose is able to be converted to glycogen in the liver but not muscle, therefore high-fructose containing foods are suboptimal. This practice is recommended for athletes participating in events that are longer than 90 minutes (after the bodies normal supply of glycogen is depleted).

 

Coyle et al. analyzed 10 trained cyclists who performed two bicycle ergometer exercise tests 1 wk apart. They found that blood glucose concentration was 20-40% higher during the exercise after carbohydrate ingestion than during the exercise without carbohydrate feeding. The exercise-induced decrease in plasma insulin was prevented by carbohydrate feeding. Ultimately, fatigue was postponed by carbohydrate feeding during exercise in seven out of the ten subjects.

 

Costill et al. examined the effect of carbohydrates on muscle glycogen resynthesis in trained runners and found that muscle glycogen levels could be normalized with the proper use of carbohydrates after strenuous activity.

 

Sherman et al. used muscle biopsies to determine the effects of moderate- or high-carbohydrate diets on muscle glycogen and performance in runners and cyclists over 7 consecutive days of training. They found that a high carbohydrate diet restored muscle glycogen to more normal levels than a moderate carbohydrate diet but did not find any difference in the effect on performance.

 

Illustration A is a graph depicting the typical carbohydrate depletion/loading regimen that endurance athletes utilize to maximize muscle glycogen stores prior to endurance exercise. This typically involves a hard workout followed by three days of a low-carb diet, another hard workout, and another three days of a high-carb diet.

Incorrect

Answers:

Answer 1, 3, 4 & 5: None of these mechanisms describe the manner in which carbohydrate loading helps endurance athletes.

 

 

(SAE13BS.12) Delayed-onset muscle soreness (DOMS) is initially evident at the muscle tendon junction and can spread throughout the entire muscle. It is primarily associated with what type of exercise? Review Topic

 

  1. Concentric

  2. Isometric

  3. Eccentric

  4. Plyometric

 

PREFERRED RESPONSE 3

 

DOMS is primarily associated with eccentric exercise because fiber lengthening occurs during muscle contraction. Muscle injury occurs very early during eccentric exercise, with structural disruption of the cytoskeleton. A significant increase in creatine kinase levels occurs 24 to 48 hours after exercise and may peak between 3 to 6 days after exercise. All other choices are not strongly associated with delayed muscle soreness.

 

 

(OBQ13.257) A 38-year-old man is being considered for medial meniscus transplantation following an arthroscopic subtotal meniscectomy performed at the time of ACL reconstruction. His body mass index (BMI) is 28kg/m2. Laboratory tests are shown in Figure A. Standing long-leg radiographs reveal a 4 degree valgus deformity compared with the contralateral side, with the weightbearing line running through the lateral tibial spine. His arthroscopic photos also revealed a 1.7cm wide Outerbridge II chondral lesion over the lateral femoral condyle and synovitis. What factor in this patient is an absolute contraindication to meniscal transplantation? Review Topic

 

 

 

  1. Rheumatoid arthritis

  2. Previous anterior cruciate ligament reconstruction with allograft tissue

  3. Malalignment

  4. Chondral defect

  5. Body mass index

 

PREFERRED RESPONSE 1

 

This patient has rheumatoid arthritis. Inflammatory arthritis is an absolute contraindication to meniscal transplantation.

 

Besides inflammatory arthritis, other absolute contraindications include diffuse arthritis, Outerbridge grade IV changes, untreated tibiofemoral subluxation, synovial disease, previous joint infection, skeletal immaturity, or marked obesity.

 

Lee et al. reviewed meniscal transplantation. They recommend meniscal transplantation only after nonoperative measures (unloading braces, weight loss, activity modification, analgesia) have failed. Indications include young patients (<50yo) with symptoms referable to a meniscus-deficient tibiofemoral compartment, with a stable knee with normal alignment, with intact articular surfaces (grade I or II).

 

Figure A is a table showing this patient's lab results. He has positive RF and anti-CCP antibody. In addition, he has synovial changes on arthroscopy. These findings are significant for active RA.

 

Incorrect Answers:

Answer 2: Previous cruciate ligament reconstruction is not a contraindication.

 

Meniscus transplantation can be attempted if cruciate ligament deficiency was addressed, or there are plans for concurrent or staged reconstruction. Answer 3: Varus/valgus malalignment is not an absolute contraindication although concurrent/staged treatment is necessary to correct alignment. Answer 4: Outerbridge Grade I and II chondral defects are not contraindications. Knees with Outerbridge III lesions may still be considered if defects are focal and concomitant treatment is performed. Answer 5: Marked obesity is a contraindication. This is not present in this individual.

 

 

(SAE10SM.48) Which of the following pieces of equipment currently offers the greatest opportunity for lowering the number of equestrian injuries? Review Topic

 

  1. Knee pads

  2. Wrist guards

  3. Boots

  4. Helmets

  5. Quick release stirrups

 

PREFERRED RESPONSE 4

 

Ball and associates reported that "horseback riding was more dangerous than motorcycle riding." In a 10-year study of major traumatic injuries, they reported that 151 (2%) of 7,941 trauma patients had major equestrian injuries (injury severity score

> or = 12). Injuries included the chest (54%), head (48%), abdomen (22%), and extremities (17%). Only 9% of riders wore helmets, and 64% believed the accident was preventable. The authors noted that "helmet and vest use will be targeted in future injury prevention strategies." In another study, Frankel and associates noted that helmet use was only documented in 34% of riders. Although orthopaedic injuries are common, knee pads, wrist guards, boots, and quick release stirrups would most likely have less impact on injury prevention.

 

 

(SAE08OS.80) When obtaining grafts from a trochlear donor site during an osteochondral autograft transplantation procedure, where on the trochlea are the lowest contact pressures found? Review Topic

 

  1. Proximal/medial

  2. Proximal/lateral

  3. Central

  4. Distal/medial

  5. Intercondylar notch

PREFERRED RESPONSE 4

 

Patellofemoral contact pressure studies have shown that the lowest contact pressures during a functional range of motion of the knee (0 degrees to 110 degrees) are located on the distal/medial surface of the trochlea. However, before a final donor site location is chosen, additional considerations include the size of the defect (the distal/medial location may not provide sufficient surface area for larger lesions) and the curvature of the recipient surface (the distal/medial surface is convex whereas the intercondylar notch is saddle-shaped).

 

 

(SAE13PE.10) Figures 10a and 10b are the sagittal and coronal MRI scans of a 5-year-old boy who noticed “clicking” in his right knee. His family denied any trauma, but admitted that the child was active and fell frequently. Birth and developmental history were unremarkable, and specifically negative for other musculoskeletal conditions. On physical examination, there was no warmth, tenderness, or erythema, or effusion. The child had an audible and palpable clunk when the knee was taken from a position of extreme flexion to full extension. There was no anterior, posterior, medial, or lateral instability on examination or medial or lateral joint line tenderness. The child had not been systemically ill. Radiographs were unrevealing. What is the most likely diagnosis? Review Topic

 

 

 

  1. Discoid lateral meniscus

  2. Congenital absence of the anterior cruciate ligament

  3. Torn medial meniscus

  4. Osteomyelitis of the distal femur

PREFERRED RESPONSE 1

 

A discoid lateral meniscus is probably the most common cause of a symptomatic clicking or clunking in the knee in a young child. This is a congenital problem that can become symptomatic as soon as a child ambulates, or the condition may remain asymptomatic for several years. The meniscus develops from a cartilaginous anlage and at no point in its development is it discoid. The MRI scans reveal a band of meniscal tissue filling the joint's lateral compartment on both sagittal and coronal images. A medial meniscal tear is usually accompanied by a history of injury and an effusion, which are not present in this child. There is also no joint line tenderness, which makes this diagnosis less likely. Congenital absence of the anterior cruciate ligament may be found in children born with congenital knee hyperextension, which is ruled out in this case by normal history and examination findings. Children with osteomyelitis are often systemically ill. On examination, they may have warmth and tenderness. MRI scans will often show an area of increased signal on T1-weighted images.

 

 

(SAE10SM.99) Which of the following can be seen in the heart of a well-conditioned athlete? Review Topic

 

  1. Decreased stroke volume

  2. Decreased cardiac output

  3. Decreased resting heart rate

  4. Decreased ventricular wall thickness

  5. Decreased vagal tone

 

PREFERRED RESPONSE 3

 

The well-conditioned heart of an athlete leads to increased ventricular wall thickness which in turn increases the amount of blood ejected from the heart per given stroke (stroke volume). The increased parasympathetic (vagal) tone also leads to a lower (decreased) resting heart rate. Cardiac output is equal to stroke volume X heart rate and is increased during exercise in a well-conditioned athlete.

 

(OBQ14.3) A 22-year-old ballet dancer undergoes hip arthroscopy for increasing hip pain and popping with activity. She experiences complete resolution of signs and symptoms post-operatively. Her pre- and post-operative magnetic resonance sagittal images shown in Figure A (left, pre-operative; right, post-operative). Which of the following pre-operative physical examination findings may have been positive? Review Topic

 

 

 

  1. Pain with internal and external rotation of her hip with her hip and knee in an extended position

  2. Limited motion when moving the hip from flexion-abduction-external rotation to flexion-adduction-internal rotation

  3. Moving from hip flexion-abduction-external rotation to neutral triggers a popping sensation

  4. Limitation in active hip range of motion with catching, locking and grinding noted on passive motion

  5. Pain with a half sit-up and tenderness at the pubic ramus

 

PREFERRED RESPONSE 3

 

This patient has internal snapping hip (coxa saltans), which is caused by the psoas tendon sliding over femoral head, iliopectineal ridge, lesser trochanter exostoses, or iliopsoas bursa.

 

Snapping hip exists in 3 forms: (1) external snapping hip, which is caused by the iliotibial band (ITB) sliding over the greater trochanter, (2) internal snapping hip, and

(3) intraarticular snapping hip, which is caused by loose bodies (traumatic, or from synovial chondromatoses) or labral tears. While painless snapping hip requires no treatment, painful snapping hip may be addressed with activity modification, physical therapy, steroid injections. Surgical release (ITB z-plasty or psoas tenotomy) is indicated if nonoperative management is unsuccessful.

 

Ilizaliturri et al. evaluated the results of endoscopic iliopsoas tendon release at the lesser trochanter (10 patients) vs endoscopic transcapsular psoas release from the peripheral compartment (9 patients). There were improvements in WOMAC scores in both groups, and no difference between groups. They conclude that both techniques are equally effective.

 

Marquez Arabia et al. evaluated if the psoas tendon regenerates after tenotomy in 27 patients. At 23 months, they found that tendon regeneration occurred in all patients, to

 

a mean circumference of 84% of the original. One patient had persistent pain, but all had 5/5 hip flexion strength. They hypothesize that the bulk of iliopsoas muscle fibers attaches directly to the proximal femoral shaft without a tendon, preventing retraction and allowing regeneration to occur easily.

 

Figure A shows pre- and post-operative arthroscopic psoas tenotomy magnetic resonance sagittal images. Illustration A shows the psoas tendon (white arrows) prior to transection. Illustration B shows the psoas tendon after transection (green arrows, proximal tendon segment; yellow arrows, distal segment). Illustration C and D are diagrams showing release at the level of the lesser trochanter and hip joint respectively.

Incorrect

Answers:

Answer 1: This is consistent with pincer-type femoroacetabular impingement. Additional findings may include internal rotation only to neutral with her hip flexed to

90 degrees, but full external rotation. Answer 2: Decreased internal rotation and a positive impingement test (forced

flexion, adduction, femoroacetabular Answer 4: These

and internal

rotation) are classic findings

findings may

be found with intra-articular

for cam-type impingement loose bodies.

Answer 5: These signs are typical for athletic pubalgia.

 

 

(SAE08AN.35) A 16-year-old boy sustains a twisting injury to the left knee while wrestling. MRI scans are shown in Figures 22a through 22c. What is the most likely diagnosis? Review Topic

 

 

 

  1. Anterior cruciate ligament rupture

  2. Posterior cruciate ligament rupture

  3. Bucket-handle medial meniscus tear

  4. Lateral meniscus tear

  5. Osteochondral lesion

PREFERRED RESPONSE 3

 

The MRI scans show a displaced bucket-handle medial meniscus tear that can be visualized on coronal, sagittal, and axial views. The sagittal view shows the typical “double posterior cruciate ligament sign,” in which the low-signal bucket-handle fragment parallels the normal low-signal posterior cruciate ligament. The coronal and axial images both show the displaced medial meniscus in the notch.

 

 

(SAE08AN.9) An 18-year-old woman sustains a twisting injury of the knee while skiing. Figures 7a and 7b show the radiograph and coronal MRI scan of the knee. In addition to the injury shown, what is the most likely associated injury? Review Topic

 

 

 

  1. Medial collateral ligament rupture

  2. Patellar dislocation

  3. Patellar tendon rupture

  4. Anterior cruciate ligament rupture

  5. Posterior cruciate ligament rupture

 

PREFERRED RESPONSE 4

 

The MRI scan shows a Segond fracture, which is a small avulsion of the lateral joint capsule from the anterolateral aspect of the proximal tibia. It is almost always associated with anterior cruciate ligament rupture and often with a tear of either the medial or lateral meniscus.

 

(OBQ12.257) The function of which of the following structures is to resist internal tibial rotation with the knee in full extension? Review Topic

 

  1. Anterior cruciate ligament

  2. Iliotibial band

  3. Popliteus tendon

  4. Popliteofibular ligament

  5. Posterior oblique ligament

 

 

PREFERRED RESPONSE 5

 

The primary function of the posterior oblique ligament is to resist internal tibial rotation with the knee in full extension.

 

The posterior oblique ligament is a structure within the posteromedial corner of the knee, with attachments proximally to the adductor tubercle of the femur and distally to the tibia/posterior knee capsule. The posterior oblique ligament and posteromedial capsule play a significant role in the prevention of additional posterior tibial translation in the knee in the setting of posterior cruciate ligament injury. They also act to resist internal tibial rotation with the knee in full extension.

 

Griffith et al. reports that the posterior oblique ligament provides significant resistance to valgus and internal rotation forces with knee extension. They used a cadaver model and demonstrated that the superficial MCL resists valgus and external rotation forces more than the posterior oblique ligament, while the posterior oblique ligament is more involved in resisting internal rotation.

 

Tibor et al. reviews the anatomy of the posteromedial corner of the knee. They report that failing to recognize injury to these structures may cause failure of cruciate ligament reconstruction surgery, and that reconstruction or repair of the posteromedial corner may be indicated in the face of multiple ligament injuries.

 

Illustration A shows the posteromedial corner of the knee, including the posterior oblique ligament.

 

Incorrect answers:

1-4: These structures are not primary restraints to internal tibial rotation in full extension.

 

 

(OBQ15.184) You perform an ACL reconstruction with bone-patella tendon-bone (BTB) autograft and are explaining the postoperative rehabilitation protocol to your patient. Which of the following is associated with increased quadriceps volume and improved quadriceps strength at 1 year without a higher risk of knee instability? Review Topic

  1. Use of a hinged knee brace for 6 weeks postoperatively

  2. Use of a continuous passive motion device for 2 weeks postoperatively

  3. Eccentric strengthening starting at week #3

  4. Concentric strengthening starting at week #3

  5. Eccentric strengthening starting at week #12

 

 

PREFERRED RESPONSE 3

 

Early eccentric strengthening following ACL reconstruction has been associated with increased quadriceps volume and improvement in strength at 1 year without a higher risk of knee laxity.

 

Proper rehabilitation following ACL reconstruction is vital to a successful outcome. The cornerstone of ACL rehabilitation is range of motion, strengthening and functional exercises without risk of destabilizing the knee. There are a variety of rehabilitation protocols and recently an effort has been made to standardize the approach to postoperative ACL care. An accelerated protocol starting at 3 weeks postoperatively has been deemed safe and may enhance the speed and safety with which an athlete returns to play.

 

Kruse et al completed a meta-analysis of 29 Level I and II studies focused on the ACL rehabilitation process. The authors concluded that immediate postoperative weightbearing and knee range of motion 0-90 is safe. Early eccentric strengthening appears to accelerate and improve strength gains at 1 year as compared to delayed eccentric strengthening. Home-based rehabilitation programs can be as effective as formal physical therapy. Postoperative bracing and CPM use is neither necessary nor beneficial.

 

Van Grinsven et al conducted a systematic review of ACL rehabilitation programs and physical therapy modalities to develop an evidence-based rehabilitation protocol. The authors demonstrated that an accelerated protocol without postoperative bracing focused on reduction of pain, swelling and inflammation and regaining range of motion, strength and neuromuscular control has significant advantages and does not lead to stability problems.

 

Gerber et al performed a randomized trial comparing two accelerated postoperative rehabilitation protocols – early eccentric (ECC) rehabilitation versus traditional (TRAD) rehabilitation starting at 3 weeks. There were no significant differences in knee pain, effusion or stability. However, quadriceps strength, hopping distance and activity level improved by a significantly greater amount in the ECC group versus the TRAD group at 26 weeks postoperatively. In a separate analysis of these patients, quadriceps and gluteus muscle cross-sectional area and volume in ECC patients were more than twice those in the TRAD group.

 

Illustration A depicts two eccentric ergometers that can be used in postoperative ACL rehabilitation.

 

Incorrect Responses:

Answer 1: No brace or length of brace wear has demonstrated an advantage over

another Answer

type of brace,

2:

There

duration is

of

bracing or no bracing at all.

no

advantage

to

CPM

use.

Answers 4 and 5: Accelerated concentric strengthening at 3 weeks and delayed strengthening at 12 weeks has been shown to result in inferior strength gains and activity levels as compared to early eccentric strengthening at 3 weeks.

 

 

(OBQ15.42) A 17-year-old male lacrosse player sustains an ACL tear. Imaging reveals closed physes and you recommend a transphyseal ACL reconstruction. His mother asks whether a “cadaver tendon” can be used to reconstruct his ACL instead of using his own tendon. What is the most appropriate response regarding the use of allograft compared to autograft for ACL reconstruction in an active adolescent? Review Topic

 

  1. There is a significantly higher risk of infection.

  2. There is a significantly higher risk of growth disturbance.

  3. There is a significantly higher risk of arthrofibrosis and loss of knee motion.

  4. There is a significantly higher risk of graft failure and need for revision surgery.

  5. There is a significantly higher risk of graft failure in low-dose (<2 Mrad) gamma-irradiated allografts only.

 

PREFERRED RESPONSE 4

 

In an active adolescent, anterior cruciate ligament reconstruction (ACLR) with allograft has a significantly higher risk of graft failure and need for revision surgery compared to ACLR with autograft.

 

The incidence of anterior cruciate ligament (ACL) injuries in adolescent athletes has significantly increased over the recent years, now comprising 24.5% of all ligamentous knee injuries in high school athletes. In skeletally mature adolescents, transphyseal ACLR is often performed, similar to adult patients. Proposed advantages of allograft reconstruction in patients of all ages include lack of donor-site morbidity, absence of size limitation, preservation of knee flexor/extensor mechanism, less risk of postoperative knee stiffness/pain and cosmetic appearance. Benefits of autografts include strong structural and fixation properties as well as optimal biologic incorporation.

 

Kraeutler et al. performed a meta-analysis comparing bone-patellar tendon-bone (BPTB) autograft to allograft for ACLR. Patients who underwent ACLR with BPTB autograft demonstrated lower rates of graft rupture, lower levels of knee laxity, improved single-legged hop test results and were more satisfied postoperatively compared to ACLR with BPTB allograft. The authors therefore recommended BPTB

 

autograft

ACLR,

particularly

in

young

active

patients.

Engelman et al. performed a case-control study comparing ACLR in an adolescent cohort using autograft or allograft. Postoperative knee laxity and use of allograft were significantly related to graft failure and need for revision surgery. There was no difference in functional outcome scores, knee range of motion, infection or growth disturbance. There was no difference in graft survival between low-dose (<2 Mrad) gamma-irradiated allografts and nonirradiated allografts.

 

Pallis et al reported a prospective cohort study of 122 ACLR performed in cadets prior to matriculation at the United States Military Academy (USMA). Cadets who entered the USMA with an allograft ACLR were 7.7 times more likely to experience graft failure compared to BPTB and hamstring (HS) autograft groups. There was no significant difference in failure between the BPTB and HS autograft groups. The authors recommend autograft ACLR for young, active individuals.

Incorrect

Responses:

Answers 1-3: There is no significant difference between ACLR with allograft or autograft with regards to risk of infection, growth disturbance or arthrofibrosis and postoperative knee range of motion. Answer 5: There is no significant difference between low-dose (<2 Mrad) gamma-irradiated allografts and nonirradiated allografts with regards to graft failure.

 

 

(SAE13SM.68) A 42-year-old man has increasing pain and, to a lesser extent, some occasional left knee instability. Several years earlier he sustained a noncontact twisting injury to his knee. He had some initial soreness and pain but was able to resume his normal activities while avoiding sports. On examination, the patient has medial joint line pain, a grade 2+ Lachman, and a slight varus thrust. His radiographs reveal mild-to-moderate medial compartment osteoarthritis with varus alignment. What surgical treatment strategy likely will alleviate his pain? Review Topic

 

  1. Distal femoral osteotomy

  2. Unicompartmental knee replacement

  3. High tibial osteotomy (HTO), lateral closing wedge

  4. HTO, medial opening wedge with decreased tibial slope

 

PREFERRED RESPONSE 4

 

This patient had a previous anterior cruciate ligament (ACL) and posterolateral complex injury. With chronic instability and osteoarthritis, the best option is HTO with a decrease in the tibial slope to reduce anterior laxity. Distal femoral osteotomy is better suited to address valgus malalignment. The lateral closing-wedge osteotomy would not allow for adequate correction of the tibial slope. Unicompartmental knee replacement is not indicated when there is ligament instability. If the patient continues to experience instability following correction of the varus malalignment, reconstruction of the ACL and posterolateral corner would be appropriate at that time.

 

(SAE07SM.31) An 18-year-old lacrosse player is diagnosed with infectious mononucleosis. What is the recommendation for return to play? Review Topic

 

  1. Full participation once symptoms resolve

  2. Full participation once the splenomegaly resolves

  3. Full participation 4 weeks after the onset of symptoms regardless of the size of the spleen

  4. Full participation 4 weeks after both the onset of illness and findings of a normal-sized spleen

  5. No participation for 8 weeks

 

PREFERRED RESPONSE 4

 

Infectious mononucleosis commonly affects adolescents and young adults. It is a febrile illness accompanied by acute pharyngitis. Splenomegaly may occur and predispose the athlete to splenic rupture. Splenic rupture has been reported in nonathletes as well as in patients with normal-sized spleens. Clinical evidence supports a return to all sports 4 weeks after the onset of symptoms provided that the spleen has returned to normal size.

 

 

(SAE08OS.77) Which of the following bones is most frequently involved in stress fractures in athletes? Review Topic

 

  1. Femur

  2. Tibia

  3. Fibula

  4. Navicular

  5. Fifth metatarsal

 

PREFERRED RESPONSE 2

 

The tibia is the most frequent stress fracture location in most series in both athletes and modern military training. The anterior midshaft region of the tibia may be at higher risk secondary to tensile forces and a relative paucity of blood supply.

 

(SAE10PE.57) The parents of a 14-year-old female soccer player are concerned about any future injury. They have been advised that she has the potential to play for the US Olympic team. They are especially concerned about the anterior cruciate ligament (ACL). What should you advise them? Review Topic

 

  1. ACL injuries are more common in men younger than 30 years of age.

  2. ACL injuries are more common in women younger than 30 years of age.

  3. ACL injuries are usually the result of contact sports.

  4. The incidence of ACL injuries can be decreased by a neuromuscular training program.

  5. ACL injuries are rarely associated with meniscal injury.

 

PREFERRED RESPONSE 2

 

ACL injuries are five to eight times more common in young women. The highest incidence is associated with basketball and soccer. These sports require rapid directional and rotational changes. Use of neuromuscular training programs has not been associated with a decrease in ACL injuries. It is recommended that there be more frequent rests. ACL injuries are commonly associated with meniscal injury.

 

 

(SAE13BS.11) After sustaining a muscle contusion injury, prolonged immobilization leads to Review Topic

 

  1. increased tensile stiffness.

  2. increased granulation tissue production.

  3. improved recovery of tensile strength.

  4. reduced incidence of myositis ossificans.

 

PREFERRED RESPONSE 2

 

Increased granulation tissue production occurs with prolonged immobilization following contusion injury; this condition may lead to myositis ossificans. Prolonged immobilization can lead to contraction of scar, poor structural organization of the regenerating muscle, and, ultimately, scar tissue. In a study by Jā—»rvinen and associates, muscles immobilized following contusion injury were pulled to failure at tensile strengths much lower than tolerated by mobilized limbs. In addition, there is a decrease in tensile stiffness.

 

(SAE10HK.33) A 22-year-old patient has had severe groin pain for many months and is unable to engage in any physical activity. The AP radiograph of the pelvis shows minimal arthritis. The lateral radiograph of the hip is shown in Figure 33a. An MR-arthrogram is shown in Figure 33b. What is the most appropriate treatment at this stage? Review Topic

 

 

 

  1. Hip arthroscopy and labral debridement

  2. Femoroacetabular osteoplasty and labral repair

  3. Femoral osteotomy

  4. Hemiarthroplasty

  5. Total hip arthroplasty

 

PREFERRED RESPONSE 2

 

The patient has femoroacetabular impingement. The prominence on the femoral neck has resulted in a labral tear and detachment. An MR-arthrogram is the most appropriate modality for diagnosis of a labral tear. The diagnosis of a labral tear per se is not an indication for surgical intervention because the natural incidence of this condition is not known. Labral debridement without addressing the underlying anatomic abnormality is likely to result in a suboptimal outcome. The most appropriate treatment, when indicated, is shaving down of the femoral neck to remove the bony prominence and attachment of the labrum. Femoral osteotomy has no role in the treatment of this condition. The patient has minimal arthritis; therefore, arthroplasty is not indicated.

 

 

(SAE07SM.61) A 20-year-old man reports painless snapping about the lateral aspect of the right hip. He denies any history of trauma. Examination reveals no limp or tenderness. Hip range of motion is full, and there is good strength. Radiographs are normal. What anatomic structure is most likely causing these symptoms? Review Topic

  1. Acetabular labrum

  2. Iliopsoas

  3. Loose body

  4. Piriformis

  5. Iliotibial band

 

PREFERRED RESPONSE 5

 

Coxa saltans (snapping hip syndrome) can occur in two forms: external/lateral or interior/medial/anterior. This patient has the external/lateral form. The external/lateral form involves the iliotibial band, tensor fascia, or gluteus medius, which snaps over the greater trochanter. The external form usually can be treated with physical therapy alone; however, several recent studies report satisfactory results with surgical treatment. Faraj and associates reported good results from surgical Z-plasty in a series of 10 patients. White and associates reported good results in a series of 16 patients with 17 hips who underwent surgical release of an external snapping hip. The interior/medial/anterior form can involve the iliopsoas tendon, acetabular labrum, subluxation of the hip, and loose bodies.

 

 

(SAE10BS.91) In the absence of developmental dysplasia of the hip, what is the most common cause of osteoarthritis? Review Topic

 

  1. Legg-Calve-Perthes disease

  2. Traumatic labral tear

  3. Repetitive high-impact activity such as football

  4. Femoroacetabular impingement

  5. Hip injury resulting from a direct impact such as a knee hitting a dashboard

 

PREFERRED RESPONSE 4

 

Femoroacetabular impingement is a mechanism for the development of early osteoarthritis for most nondysplastic hips. Early surgical intervention for treatment of femoroacetabular impingement, besides providing relief of symptoms, may decelerate the progression of the degenerative process for this group of young patients. There are two general types of femoroacetabular impingement. In cam impingement, the femoral deformity is usually a bump on the head-and-neck junction that impinges on the acetabular rim. The pincer type of impingement is caused by deformity on the acetabular side such as a deep socket or acetabular overcoverage due to retroversion. Both mechanisms create an obstacle for flexion and internal rotation.

 

(SAE08OS.110) Successful short-term outcomes following microfracture of the knee are associated with Review Topic

 

  1. a high fill-rate of the defect on MRI.

  2. a high body mass index.

  3. a long duration of preoperative symptoms.

  4. microfracture of the patella.

  5. anterior cruciate ligament insufficiency.

 

PREFERRED RESPONSE 1

 

Short-term outcome studies of microfracture show good success associated with three variables. These are high fill-rate on follow-up MRI of the defect, low body mass index, and a short duration of preoperative symptoms. Generally patellar microfracture results have been poor.

 

 

(SAE13SM.46) Chronic traumatic encephalopathy (CTE) is a neurodegenerative disease that is characterized by Review Topic

 

  1. onset most often by age 30.

  2. a temporary state of neuronal and axonal derangement.

  3. manifestations of affect such as apathy, irritability, and suicidal ideation.

  4. absence of gross pathological brain changes upon autopsy.

 

PREFERRED RESPONSE 3

 

CTE is a neurodegenerative disease that occurs years or decades after recovery from acute or postacute effects of head trauma. The exact relationship between concussion and CTE is not entirely clear; however, early behavioral manifestations of CTE have been described by family and providers to include apathy, irritability, and suicidal ideation. For some patients, cognitive difficulty such as poor episodic memory and executive function may be the first signs of CTE. Onset most often occurs in midlife after athletes have completed their sports careers, with mean age of onset at 42 years. The effects on the brain are degenerative, leading to a permanent state of derangement. Autopsy findings demonstrate multiple gross pathological findings. The condition is more common among contact athletes.